You are on page 1of 225

AC CIRCUITS

This Free Quality Manual is


Found only at POWERLINE

POWERLINE REVIEW CENTER


M EZZAN INE FLOOR, DONA AM PARO BUILDING
CO RNER ESPANA G. M. TOLENT1NO STREETS
SAM PALOC, M ANILA
TEL. NOS. 735-73-02 & 733-21-18

TEL. NOS. (03)261-2244 & (032)261-8452


1

POWERLINE ENGINEERING REVIEW CENTER, INC.

AC CIRCUIT -1 REVIEW LECTURE

Cycle - a complete change in value and direction of an alternating quantity


1 cycle ~ 360 electrical degrees
Frequency ( f ) - no. of cycles per second (f) Hz.

Periodic time ( T ) - Time taken to complete one cycle


T = — seconds.
/
Instantaneous value - the value*of alternating quantity at any instant.

Maximum value - the maximum value attained during positive or negative half cycle.

It vernal''\n{(ui'-iw t. ori a^i live ntgfailtaneous values oVnaft cycle.

Area,..under.. .the...curve
in general: Ave...value -
Base
2
For sine wave: Ave...value = —* Max. ..value = 0.637 * Max....value
n
Effective value - is that which when applied to a given circuit for a given time produces the same
expenditure of energy as when dc is applied to the same circuit for the same
interval time,
also called "Root Mean Square" RMS value.

j A rea.. .under. .Jhe.. .square...curve


In general: RMS...value
v Base

For sine wave : RMS...value = —~ * Max...value - 0.707 * Max...value

Form factor ~ =1.11 (for sine wave)


Ave...value

_ , , Max...value . .. . .
Peak factor - ------- --------= 1.414 (for sine wave)
RMS., .value

Phase Difference - the angular difference or angular displacement between alternating


quantities
- also called “phase angle•”
In phase - if two alternating quantities attains their maximum & minimum values at the same
time.
Out o f phase- does not attains their maximum and minimum values at the same time.

Wave ;
2

SING LE PHASE CIRCUIT

POWER FORMULAS :

True, real or active power (P)


P = E I cos 0 = I2R in watts.

Reactive or Idle power (Q)


Q ~ E I sin 0 = I2X in vars.
Where : + capacitive (or leading) vars } Based from voltage
~ inductive (or lagging) vars } conjugate method

Apparent Power (S)


S = EI = I2Z = J p 2 + Q 2 in va
Where : 0 ~ angle between E & I

DETERMINATION OF POWER (In Complex form)

1. Component method :
Let E = Ej + j E2 P = EI
I - I, + jl2 P = B, I, + E2I2

2. Conjugate method:
Let E I ~ conjugate of E & I respective

a) Voltage conjugate method : b.) Current Conjugate method :


S = E I w.r t the horizontal axis. S = E 1 w.r.t. the horizontal axis
.P i j Q J p 2 i Q2 /.; 0 !> i j o V />2 ■' (P LU1
where: -t* capacitive (or leading ) vars where: - capacitive ( or leading ) vars
- indcutive ( or lagging) vars + inductive ( or lagging ) vars

POWER in terms o f MAGNITUDE:

P - I2 R - E I (p.f.)
Q = I" X E 1 (r.f.)
s = ij z = e i = ij p 1 +e2
u * a R p watts ■
where : p.f. = cos 8 = — = — = ------- = power factor
r 2 S VA
r.f. = sin 0 = Q var^ = reactive factor
S ~VA

R ESO N A N CE
Is that condition existing in a circuit containing at least one resistor, an inductor and
capacitor where in the current behaves as if it is purely resistive.
3

Characteristics o f Resonant C ircuit:

1. The total current is in phase with the impressed voltage.


2. The power factor of the circuit is unity.
3 The total reactance power is zero.
4. The imaginary component o f the total impedance ( or admittance) in complex form is zero

I j Series Resonance Condition : | II | Para lie! Resonance Conditions

1. XL= Xc P l “ Pc
2: Z ~ R 2. Z = L/RC
3. I is maximum, Z is minimum 3. I is minimum, Z is maximum

I *
V
=R ' = VCR

4. I in phase with V 4. I is in phase with V


5. p.f. = 1.0 5. p.f. = 1.0

RESONANCE FREQUENCY:
fr =
1 Hz ( for series)
iKyfLC
1
2. f, = 2ns^ L±_
C
f£_
[}
Hz (for parallel resonant circuit)

1
3. /, Hz
In-JlC i L -(C/fe

Quality Factor (O Factor)


Max...Stored...Energy [
Q = 2n{
[ Energy...Dissipated / cycle \
InfjJL _ 1 1
Q= (for series)
Rs ~ 2nfrCRsRs V C

Q= = 2rfrCRp = RpJ— (for parallel)


2nfrL

Frequency at half-nowar points / Baad-width

f t = "f *r +4zrZ,
R bw = / 2- / , = |
4ttL ' '*

Maximum Power Transfer:


Condition : R LOad S

4/?
POWERLIne r e v ie w c e n te r Inc.
H e IQtimate E l Review Center
AG CIRCUITS (*1*|>) REVIEW QUESTIONS
1. The instantaneous magnitude of a voltage is given as 100 sin 377t volts,
What is the rms value of the voltage?
A. 100 volts d 70.7 volts
B. 63.7 volts D. 50 volts
2. What is the rms value of a half wave rectified sinusoidal wave with 20 ampere
amplitude?
A. 14.14 Amp C; 10 Amp
B. 12.74 Amp D. 6.37 Amp
3. Which of the following waves has the highest value of peak factor?
A. square wave C. sine wave
B) half-wave rectified sine wave D. triangular
4. The instantaneous value of the voltage is given by its equation v(t) = 105 sin
cot - 25 sin 3cot +15 sin (5cot - 30 degrees). What is its Rms and average
voltage?
A. 77 & 74 volts c. 77 & 48 volts
B. 145 & 72.5 volts D. 145 & 48 volts
5. In a pure resistive circuit, the instantaneous voltage and current are given by:
v(t) =200 sin 377t volts and i(t)= 10 sin (377t-60° )amperes. What is the
average power?
A. 250 W C. 1000W
B.
V -/ 500 W D. 2000 W
6. What is the instantaneous expression for a given rms current 160-j75 amp.?
A. i = V2 250 sin (wt + 25degrees) C. /' = 250 sin (wt + 25 degrees)
B. / = V2 250 sin (wt - 25degrees) D. / = 250 sin (wt - 25 degrees)
7. An inductive load operating at phase angle of 53° draw 1400W from 120 volt
line. Find the current drawn.
A. 19.6A C. 19.4 A
B. 11.7A D. 14.6 A
8. A single phase induction motor is rated 5 hp, 220 V, 75%pf and 85%
efficiency. What is its rated current?
A. 26.6 Amp C. 22.6 Amp
B. 16.95 Amp D. 15.36 Amp
9. A coil is connected to 12 volts dc draws 1 ampere. If the same coil is
connected to 24 volts 60Hz, it draws the same amperes. What is the value of
L?
A. 40 mH C. 62 mH
B. 55 mH D. 70 mH
10. A lightning protector circuit contains 55.7 mH coil in series with a 6 ohms
resistor. What current will flow when it is tested with a 110 volt, 60 Hz voltage?
A. 4.93 A C. 6.57 A
B. 5.04 A D. 4.05 A
11 A coil having a Q of 25 draws 20mA when connected to a 12 volts 1kHz power
supply. What is the inductance?
A )96 mH C. 78 mH
B. 88 mH D. 58 mH
12. At what frequency will the current lead the voltage by 30° in a series circuit
with R= 8 ohms & C=30/ / F.
A. 1149 Hz C. 7216 Hz
B. 2298 Hz D. 796 Hz
13. For a series RLC circuit with elements R = 50 Q, L = 8 mH and C = 2.22 jjF.
What is the current of the circuit if its supply is 200 volts, 796 Hz?
A. 2 - j2 Amp C) 2 + j2 Amp
B. 5 +j13A m p D. 5 -j1 3 A m p
14. A source of 100 Volts rms is supplying a resistance 80 ohms in parallel with a
200 mH inductor. To what angle is the current lag or lead?
A) 125W & 46.7 deg. lag C. 1.25W & 46.7 deg. lead
B. 125W & 43.3 deg. lag D. 1 25W & 43.3 deg. lead
1.5. A load of 6000 voit-amp, 0.6 pf lagging is supplied at 200 volts 60 Hz. It is
desired to raise the pf to 0.95 lagging using capacitor. What is the value of
capacitor?
_/y 239.8 micro-farad C. 23.98 micro-farad
B. 2.398 micro-farad D. 2,398 micro-farad
16. A 10-kVA induction motor operating at 80% power factor lagging and a 5 kVA
synchronous motor operating at 70% power factor leading are connected in
parallel. Find the total real power.
A. 3.5 kW C. 11.5kW
B. 15 kW D. 8 Kw
17. A plant draws 2000 kVA from 240 volts line at a power factor of 0.7 lagging.
Find the kVA required of a capacitor bank in parallel with the plant for the
overall power factor to be 0.9 leading.
A. 1071 C. 750
B. 1878 D. 2106
18.Two impedances, Z1 = 0.8 + j5.6 and Z2 = 8 —j16 and a single-phase motor
are connected in parallel across 200 Vrms, 60 Hz supply. The motor draws 5
kVA at 80% power factor lagging. Find the new line current when a capacitor
is connected across the load to improve the power factor to unity.
A. 20A C. 65A
B. 50A Dj 30A A
19. A series resonant circuit has resistance of 4 ohms, 100mH inductance and
unknown capacitance. It is supplied from a 120 volts, 60 Hz. Calculate the
voltage across capacitor.
^ 1131 V C. 80 V
B. 120 V D. 100 V
20. A series RL branch in an ac circuit has 8 ohms resistance and 10 ohms
inductive reactance. A capacitor is connected across the branch. What should
be the reactance of the capacitor if the unit is to have power factor of unity?
A. 17.2ohms C. 16.7 ohms
B. ;16.4ohms D. 15.8 ohms
21 .What is the capacitance for a series RLC circuit with inductance of 1mH that
will resonate at 4MHz.
A. 1.15 pico-farad C. 1.58 pico-farad
B. 1.45 pico-farad D. 1.75 pico-farad
22. A coil of resistance 2 ohms is connected is series with a capacitor of 50 micro­
farad resonate at 500 rad/sec. What is the Q factor of the coil.
A .25 C. 40
B„20 D. 100
23. What is the bandwidth of a series resonant circuit consisting of 2.4 ohm
resistance, inductor and 30 micro-farad capacitance. .The resonant frequency
is 1000 rad/sec.
A. 43 C. 82
B. 68 D. 72
24. A parallel resonant RLC circuit with R=100 ohm, L=150mH and unknown
capacitor is connected to a source voltage with co =2000rad/sec. Find
capacitance C in micro farad
A. 2 C. 1.5
B. 3.5 D. 5
25. A coil of 40 ohms resistance has an inductance of 100 mH and is connected in
parallel with a 30p.F capacitor .What is the resonance frequency of the circuit ?
A. 23.7 Hz C. 60 Hz
B. 66.3 Hz D. 79.6 Hz
26. At parallel resonance, the circuit draws a current of 2 mA. If the Q-factor of the
circuit is 100. What is the current through capacitor?
A. 2 mA C. 1mA
B. 200 mA D. 100 mA
27. What is the dynamic impedance of a parallel resonant circuit if C= 1 ju F , R=1
ohmandL=1H?
A .1 M Q C. 100 kQ
B. 10 kQ D. 1000 Q
28. A 220 volt 20 A inductions motor draws 3 kW of power. A 4 kVA capacitive
load is placed in parallel to adjust the power factor to unity. What must be the
power factor of the capacitive load?
A. 69.2% C. 77.9%
B. 47.8% D. 59.4%
29.A (6+j10) ohms coil is supplied by a y(t)=30 sin10t volts ac source connected
in series with 12 volts battery. The internal impedance of the generator is
(0.1 +j2) ohms and that of the battery is negligible. Determine the current
through the coil by superposition.
A. 2.23 sin(10t-63°)+1-97 Amp. C. 1.58 sin(10t-63°)+1.97 Amp.
B. 4.33 sin(10t-63°)+2 Amp. D. 2.23 sin(10t+63°)+1.97 Amp.
30. A 5:1 ideal transformer is used to couple a load to a generator whose internal
impedance is 500+j253U ohms. Determine the load impedance that will
transfer maximum power.
A. 20+j20 C)20-j100
B. 100+j100 D. 100-j100
AC Circuits (lcp) Supplementary Problems

1. What is the period of a sinusoidal waveform having 100 MHz frequency?


A. 10ns C. 10 ps
B. 10ms D. 10 s
2. What is the angular frequency of a waveform having a period of 1 msec?
A. 2000pi rad/s C. 2000/pi rad/s
B. 10OOpi rad/s D: 1000/pi rad/s
3. The complex expression of voltage is 300 cis 90 deg. What is its instantaneous equation?
A. 424.4 sin(cot+90°) C. 424.4 cos (cot+90°)
300 sin (cot+90°) D. 300 cos (cot+90°)
4. The instantaneous of a voltage is given as 100 sin 314t volts, What is its average value?
A. 70.7 volts C. 100 volts
B 63.7 volts D. 50 volts
5. The average value of a sinusoidal current is 100 ampere, What is its rms value?
A. 63.7 A C. 70.7 A
B. 141.4 A D. 111 A
6. What is the peak factor of half-wave rectified ac?
A. 1.57 . C. 2
B. 1.11 D. 1.414
7. What is the average value of a half wave rectified sinusoidal wave with 70.7 ampere
amplitude?
A. 22.5 Amp C. 50 Amp
B. 135.35 Amp D. 31.6 Amp
8. if a half wave rectified sine wave has an RMS value of 20 A, what is the average value ?
A. 20 Amp C. 14.14 Amp
B. 12.74 Amp D. 28.3 Amp
9. The phase shift between the voltage and current vectors is due to the following loads
EXCEPT one. Which one is this ?
A. Magnet coils C. Electric flat iron
B. Power capacitors D.Fluorescent lamps
10. A VAR meter depends on the following values EXCEPT one. Which one is this?
A. voltage C. Sine of the phase angle
B. cosine of the phase angle D. current
11. A 120 volt, 60 Hz power line voltage is applied across resistance of 10 ohms. What is the
phase angle in degrees between the current and the voltage?
A. 0 C. 45
B. 30 D. 15
12. A single phase circuit is supplied from a 220 volts 60 Hz source draws 2000 watts of
power and 10 amperes current. What is the power factor of the circuit?
A. 72.5% C. 82.6%
B. 90.9% D. 100%
13. A load of 20 + j 35 ohms is connected across a 220 volts source. Determine the power
A. 1042 watts C. 596 watts
B. 975 watts D. 1087 watts
14. A 10 ohms R,, 30 ohms XL and 60 ohms XL are connected in parallel across a 220 volts
60 Hz source. What is the current in R, in ampere?
A. 7.33 C. 22
B. 3.7 D. 24.6
15. A resistance of 30 ohms and an inductive reactance of 60 ohms are in series across
120 volts, 60 cycles source. Find the power factor in percent of the circuit?
A 45 C. 87
B .67 D. 75
16. A current is represented by its equation 10 sin(cot + 30°) and 5 sin (3cot - 30°). The
magnitudes of the components are instantaneous values. What is the Rms value of the
current?
A. 13.22 amperes C. 1.1 amperes
B. 7.9 amperes D. 10.6 amperes
17. A load draws current from a voltage source. The equation of the voltage and current are
1=10 sin(cot+30°) and V-220 cos (cot-60°) What is the power?
A. 0 watt C. VJxi 000 watts
B. 1100 watts D. 2200 watts
18. A load draws current from a voltage source. The equation of current and voltage are I
=10 cos cot & V=220 sin cot. What is the Power?
A. ■0 watt C. 2,200 watts
B. 1556 watts D. 1320 watts
19. The instantaneous voltage and current equations of an ac circuit are v(t)= 200 sin 377t
volts and i(t)= 8 sin (377t-30°) amp. respectively. Determine the apparent power in
complex form.
A. 296.8+j400 C. 692.8+j400
B. 962.8+j400 D. 692.8+j200
20. An inductor draws 5A of current at 110 volts, 60 Hz. Determine the inductance.
A. 58.36 mH C. 56.38 mH
B. 53.68 mH D. 38.56 mH
21. A capacitor draws 2A of current at 120 volts, 60 Hz. Determine the capacitance in micro­
farad.
A. 44.2 C. 36.4
B. 20.8 D. 55.6
22. A single phase induction motor is rated 5 hp, 75% power factor and 220-volt. What is the
full load current?
A. 22.5 amperes C. 16.5 amperes
B. 20.5 amperes D. 18.5 amperes
23. An impedance draws a current I = 10cos (cat - 30°) Amp from a voltage V = 220 sin cot
volts. What is the power ?
A. 2200 Watts C. 190.5 Watts
B. 1100 Watts D. 1320 Watts
24. The resistor of 6 ohms and unknown impedance coil in series draw 12 Amp from a 120V,
60 Hz line. If the power taken from the line is 1152 Watts, What is the coil inductance ?
A. 15.9 mH C. 10 mH
B. 20 mH D. 1.59 mH
25. A 50 mH inductance is connected in series with a 20 ohm resistor. If the voltage is 220
volts, what power is drawn?
A. 560 watts C. 802 watts
& j 1280 watts D. 680 watts
26. A 25Q resistor is connected inseries with a coil of 50Q resistance and 150 mH
inductance. What is the equationof the current if the source voltage is 200sin(wt-
30deg)?
A. 2.16sin(<»t +7°) C. 1.52sin(cot-67°)
B. 2.13sin(cot -67°) D. 1 52sin(cot -30°)
27. In a series RC circuit, the voltage across the capacitor and the resistor are 60 and 80 volt
respectively, the total voltage is______
A. 70 V C .100 V
B 140 V D. 52.9 V
28. A coil is supplied from a 200 volt and takes a current of 2 amperes at 0.707 power factor
lagging. The quality factor of the coil is__________
A. 25 C. 1
B. 10 D. 100
29. A 50 and 100 microfarad capacitors are connected in series and across a 100 sin (wt +
30) voltage. Write the equation of the current.
A. 1.26 sin (cot+120) C. 5.65 sin (cot+120)
B. 1.26 sin (cot +90) D. 5.65 (rot+90)
30. A 160 microfarad capacitor is connected in series with a 10 ohm resistor. Write the
equation of the current when the voltage is 220 volts?
A. I = 9.8 sin (377 t + 52 degrees) C. I = 7.2 sin (377 t + 46 degrees)
B. I = 8.9 sin (3771- 46 degrees) D. I = 11.4 sin (3771 + 59 degrees)
31'. A 40 micro-farad capacitor is connected in series with a 40 ohm resistor. If the voltage is
220-V, what is the current?
A. 8.9 amperes C. 4.21 amperes
B. 2.84 amperes D. 7.26 amperes
32. A series circuit with 2 amperes flow through a 20 ohm resistor, 50 ohm inductive
reactance and 60 ohm capacitive reactance. What is the supply voltage?
A. 46.9 volts C. 50.5 volts
B. 40.5 volts D. 44.7 volts
33. A 25 ohm resistor connected in series with a coil of 50 ohm resistance and 150 mH
inductance. What is the power factor of the circuit?
A. 85% C. 90%
B. 80% D. 75%
34. A 40 micro-farad capacitor, 100 mH inductance and a 40 ohm resistors are connected in
series. The voltage is 220 volts, what is the current?
A. 6.25 amperes C. 7.23 amperes
B. 5.92 amperes D. 4.47 amperes
35. What is the reactance of a 60 cycle a-c circuit containing an inductance of 1.5 H in series
with a capacitance af 40 micro farad?
A. 499 ohms inductive C. 565 ohms inductive
B. 499 ohms capacitive D. 66.3 ohms capacitive
36. A series circuit has R of 500 ohms, a 10 micro-henry inductance L and a 4 micro-farad
capacitance C across a 60 Hz ,120-volt power source. Compute the load current.
A. 0.144 A C. 0.120 A
B. 0.100 A D. 0.150 A -
37. A 50 micro-farad is connected in series with a coil having 50 ohm resistance and 150 mH
inductance. The source voltage is 100 sin (cot-30°)V. What is the power?
A. 198 watts C. 212 watts
B. 147 watts D. 165 watts
38. A 25-kVA load with a power factor 0.80 lagging has a group of resistive beating units
added at unity power factor. How many kW do these units takes, if the new overall power
factor is 0.85 lagging?
A. 1.25 C. 2.4
B. 4.2 D. 2.75
39. A capacitor of 80 micro-farad is connected in parallel with 20 ohm resistor. What power is
drawn when source voltage is V=220 sin cot volts?
A. 1825 watts C. 1010 watts
B. 1210 watts D. 2420 watts
40. A coil of a 50 ohm resistance and of 150 mH inductance is connected in parallel with a 50
uf capacitor. What is the power factor of the circuit?
A. 80% C. 70%
B. 50% D. 60%
41. A series RLC circuit is excited by a 100 volts 79.6 Hz source. Resistance R is 100 ohms,
inductance L is 1 H and capacitance C is 5 micro farad. Calculate the voltage across C.
A. 353.5/45° C. 7Q.7/-450
B. 282.8/-1350 D. 332.6/135°
42. A 10 ohms resistor is connected in parallel to Z=3+j4 ohms. What is the power factor?
A. 0.6 C. 0.609
B. 0.8 D. 0.809
43. A source of 100 Volts rms is supplying a resistance 80 ohms in parallel with a 200 mH
inductor. To what angle is the current lag or lead?
A. 46.7 deg. lag C. 46.7 deg. lead
B. 43.3 deg. lag D. 43.3 deg. lead
44. A coil of 50 ohm resistance and of 150 mH inductance is connected is parallel with a 50
micro-farad capacitor. The source voltage is 100 sin (cot +30deg). What is the equation
of the line current?
A. 1.91 sin (cot+ 52.5deg) C. 1.82 sin (cot - 62deg)
B. 1.25 sin (cot+ 75.5deg) D. 1.32 sin (cot -75.5deg)
45. A resistor of 50 ohms and an impedance of 100 +j 150 ohms are connected in parallel
across 220 volts. What is the power factor of the load?
A. 96% C. 98%
B. 99% D. 95%
46. The two circuits has the following values: ^= 5 ohms X^= 3 ohms inductive, R2= 7 ohms
and X2= 4 ohms capacitive. If the terminal voltage is 120 volts, what is the total current?
A. 20.6 C. 30.8
B. 14.9 D. 35.5
47. Two impedances Z1=2+j4 and Z2=R+jO, are connected in parallel. Determine R so that
the pf of the circuit is 0.9 lagging.
A. 3.2 ohms C. 2.4 ohms
B. 4.2 ohms D. 6.4 ohms
48. An impedance of Z!=4+j4 ohms is connected in parallel with an impedance Z2=12+j6.
The input reactive power is 2500 var lagging. What is the total active power?
A. 3025 W C. 2502 W
B. 4025 W D. 5045 W
49. Two current, 5+j2 Amp. and 3-j2 Amp. enter a junction. What is the out going current?
A. 2+j4A C. 2A
B. 8A D. 8-j4A

50. A single phase induction motor draws 10A at 75% pf. The voltage is 220V. A capacitor
is 50 uF is installed to improve power factor, What is the resulting power factor?
A. 99% C. 97%
B. 90% D. 95%
51. A factory has an average demand of 176 kw at 70% power factor. What is the minimum
size of capacitor required to raise the power factor to 95%?
A. 122 kvar C. 165kvar
B. 82kvar D. 157 kvar
52. The power factor of a load on a 120 volt, 60Hz source is raised from 0.707 to 0.866
lagging by connecting a 53 micro-farad capacitor across the load. What is the true power
of the load in watts?
A. 286 ' C. 862
B. 682 D. 268
53. A single phase induction motor is rated 5hp, 75% power factor and 220 volts. What
approximate size of capacitor is necessary to raise the power factor to about 95%?
A. 3 kvar C. 2.5 kvar
B. 2 kvar D. 3.5 kvar
54. A load of 10,000 KVA, 85% power factor lagging is connected to a 13,200 volt line. How
much reactive power is needed to correct the power factor to 97% lagging?
A. 5,156 KVAR V C) 3,138 KVAR
B. 2,547 KVAR ' D. 4,753 KVAR
55. A tank circuit is supplied by a current source whose source resistance is 56 k Q . The
tank circuit is composed of a 56 nF capacitor in parallel with a coil of inductance 35 mH
and resistance 80 ohms. Determine the resonant frequency.
A. 3kHz C. 3.6 kHz
B. 6.2 kHz D. 4.6 kHz
56. A parallel resonant RLC circuit with G=10mS, C=1 /u F and unknown L is connected to a
source voltage with a> =2000rad/sec. Find inductance L in milli-Henry
A .2 C .10
B. 30 D. 5
57. What capacitance must be placed in series with an inductance of 0.05 H so that at 100
Hz, the impedance becomes equal to the ohmic resistance?
A. 50 uF C. 35.5 uF
B. 70.7 uF D. 87 uF
58. One leg of a radio tuned circuit has a capacitance of one times ten to the minus nine
farad. It is turned at 200 KHz, what is the inductance of the other leg in henry?
A. 6.33x1 O'4 C. 20 x 10‘3
B. 8.25x1 O'5 D. 120x1 O'3
59. A coil of 40 ohms resistance has an inductance of 100 mH and is connected in parallel
with a 40 |uF capacitor .What is the resonance frequency of the circuit ?
A. 23.7 Hz C. 60 Hz
B. 47.75 Hz D. 79.6 Hz
60. A 5 mH pure inductance is connected in parallel with 1. 0 pi f capacitor. At what frequency
shall the circuit be in anti-resonance.
A. 2250 hertz C, 60 hertz
B. 1020 hertz D. 100 hertz
61. A coil has a 50 ohms resistance and a 70 ohm reactance. A capacitor is connected in
parallel to produce resonance. The source voltage is 120 V. What is the power drawn by
the circuit.
A. 162 watts C. 132 watts
B. 97 watts D. 52 watts
62. A 100mh pure inductance is connected in parallel with a 40 ohms resistor. What is the
power drawn if the voltage is 220volts?
A. 1010 watts C. 1210 watts
B. 1720 watts D. 3018 watts
63. A 15 ohm resistor connected in parallel with an impedance of 8 - j2 ohm across an AC
source. The entire circuit takes a total average power of 2000 Watts. What is the average
power in the 15 ohm resistor
A. 2000 W C. 1277 W
B. 723 W D. 1500 W
64. A coil has resistance of 10 ohms and draws a current of 5A when connected across 100
V, 60 Hz source. Determine the reactive power
A. 433 VAR C. 250 VAR
B. 500 VAR D. 300 VAR
65. What is the reactance offered by a 100 // H inductance are connected to a 100 volt
source of frequency 3MHz?
A. 767 ohm C. 4647 ohm
B. 1885 ohm D. 1260 ohm
66. Determine the power angle in the series circuit which consist of R=25 ohms and L=0.2 H
across a power supply of 200 volts, 30 Hertz.
A. 36.4 deg. C. 52.4 deg.
B. 46.4 deg D. 56.4 deg.
67. An electronic instrument that visually shows the magnitude and shape of an alternating
current signal.
A. Electroscope C. Optical instrument
B. Synchroscope D. Oscilloscope
68. Convert 75/-260 in rectangular form.
A. 67.4-j32.9 C. 75-j26
B. 32.9-j67.4 D. 26-j75
69. A series circuit is composed of j4, j6, -j9,-j2 reactances. What is the total reactance?
A. j C.j21
B. -j D.—j21 A
70. V12 is equal to 100 cis 30 degrees and V23 is equal to 50 cis 120 degrees.
Evaluate V12-V23.
A. 62 cis 6.2 degrees C. 111.8 cis 3.4 degrees
B. 127. 3 cis 39.9degrees D. 62 cis (-6.2 degrees)

PRACTICE MAKES PERFECT!!!


AC MACHINES
(ALTERNATORS)

This Free Quality Manual is


Found Only at POWERLINE

96
iP
P O W E R L IN E
powerlinereviewcenter@vahoo.com

M E Z Z A N IN E F L O O R , D O N A A M P A R O B U IL D IN G
C O R N E R E S P A N A & G. M. T O L E N T IN O S T R E E T S
S A M P A L O C , M A N IL A
T E L N O S. 7 3 5 -7 3 -0 2 & 7 3 3 -2 1 -1 8

CEERS- POWERLINE REVIEW CENTER


TEL. NOS. <032)261-2244 & (0 3 2 )2 6 1 -8 4 5 2
POWERLINE ENGINEERING REVIEW CENTER, INC.

AC MACHINES ( ALTERNATOR) REVIEW LECTURE

ALTERNATORS
An electrical machine converting mechanical energy to AC electrical energy.

GENERAL TYPES OF ALTERNATOR:

A) Synchronous generator - a generator called synchronous because it is driven at constant speed


(synchronous speed ) and it is used in almost all type o f applications.

B) Induction generator - it is an induction motor which run as a generator with a speed above
synchronous speed. Its p.f. is usually leading and connected in parallel
with a synchronous generator in order to supply power for lighting loads.

C) Induction alternator - it generates voltage at higher frequency ( 500 Hz to 10,000 Hz ) it is used to


supply power to induction furnace in order to heat and melt the metal.

TWO TYPES OF ALTERNATOR :


1. Revolving Armature Type
2. Revolving Field Type
a. Stator - Stationary armature
b. Rotor - Rotating field poles

A d v a n ta g e s o f S ta tio n a r y ,A r m a tu r e
1. Output current is delivered to the load without passing it through brush contacts
2. Easier to insulate stationary armature winding for high ac voltage
2. Armature winding can be easily braced to prevent deformation which could be produced
by mechanical stresses.
A. Easily insulated slip rings which transfers low voltage, low power dc field circuit.

Two types of rotor used :

a. Salient pole construction - used for low speed alternator, large diameter than axial length ,
having more poles ( from 6 to 40 poles)

b. Sm ooth cylindrical construction- used for high speed alternator, diameter is less than axial
length, having less number of poles (2 or 4 pole)

PRIMEMOVERS FOR ALTERNATOR

A) F or large AC g en erato r B) For Sm all generator

1. Steam turbine ' 1. Use internal combustion engines


2. Gas turbine
3. Hydraulic turbine
4. Internal Combustion Engine

F re q ue ncy o lG e n e ra te d E m f

PN
f- — Hz Where : P = number o f field poles
120
Ns - synchronous speed, rpm
Gmenited-EniLEQuo Uqr (verphasg)

E(j) =r 4.44 ^)p N f KpKd x 10'8 volts ( rms) (if <J)p is in lines or maxwells )
or E<j> = 4.44 <j)P N f KpKd volts (rms) ( if 4>P is in weber)
or E(j) = 2.22 (|)p Z f K PKd volts (rms) (if <j)P is in weber)
or E(j> = Z Ez KpKa volts (rms)

Notations:

E(j> = generated em f per phase; ” mis voltage per conductor


4>p = flux per pole in weber or maxwell
N ~ number o f turns per phase
Z - number of conductors per phase. \; p {,
f = frequency in Hz.
Kp = pitch factor or coil span factor
Kj - distribution or breadth factor
P IT C H F A C T O R O R C O IL SPAN FA C T O R , Kf)
Vector sum o f induced em f p er coil
kf =
Arithmetic sum o f induced em f p e r coil

( ( po\
a
K P = cos —
= sin
2
\ 2 / \ .j
a° = 1 8 0 ° -p ° a
*
where : p • ■ C° '' ^ >1 S 0 °________
F rC R xm °
Slot pitch Fu.C.P.

D IST R IB U TIO N O R BR EA D TH F A C T O R , Kd
i l

Vector sum oj coil e m f em f with distributed winding


Arithmetic sum o f coil em f em f with concentrated winding

. n /r '' !
nWJ
sin
2 180
where : (3°
. —
P° slots i pole
n sm
2
p ° = distance in electrical degree between coil sides
gc ° = required distance in electrical degree to make it a full pitch coil.
(3 0 = distance in electrical degree between adjacent slots,
n = number o f slots per pole per phase

For instance:
Consider a 36 sjkris, 4pole, 3 phase alternator

I0 \ IIpitch(9 slots/pole)

' || 12 13 14 15 16 17 1 8 -

U H

Fractional pitch ( 8 / 9 pitch)


Alternator Equivalent Circuit:

From
Exciter
Circuit

For star connected armature windings:

Ej, —J?> E* I* = I,. = I

For Delta connected armature windings:

El = E* la “ I* v+= vL
S

t_JL
E<t>= K+ 7„ (Zs) where : Zs = Rc + j

Equivalent Circuit Per Phase Vector Diagram

/------------------- >— 1
1
la t
1
/ va
\
4
Notations:
R* = effective armature (stator) resistance per phase
X s = synchronous reactance per phase = Xar + XL
Zs = synchronous impedance per phase
X L - leakage reactance per phase
Xar = reactance due to armature reaction
EcJ) = generated or no-load voltage per phase
E l - generated or no-load line-to-line voltage
V L —Line-to-line terminal voltage
IL = Line current
lo ~ Ia ~ armature current or phase current
V f - exciter voltage
R f = field resistance
If = field current

ALTERNATOR VOLTAGE REGULATION :


E - V
X 100% = - t ---- t-x 100%
Ka

Alternator on load, the variation of terminal voltage is due to the following reasons:

1. Voltage drop due to armature resistance


2. Voltage drop due to armature leakage reactance
3. Voltage drop due to armature reaction ( major reaso n )

Vector diagram:

( a ) if lagging (b )i Heading (c ) if unity

in com plex fo rm
E<t> = Vj>+ Ia ( Z s )
in m agnitude

Where: © is lag’g. p.f. 0 f o r leading p.f. and 0 = 0 for unity


Alternator Equivalent Circuit:

From
Exciter
Circuit

For star connected armature windings:

El —%/3 E* I(|) = I,. = I,


S

For Delta connected armature windings:


h_ ; V,
E l = E* I, = I* “
S

E<P= P+ 7„ (Zs) where : Zs - Rc + j X s

Equivalent Circuit Per Phase Vector Diaarorr}

lo
T"
Zs & i
i
va
\
i
jL
4
Notations:
R* = effective armature (stator) resistance per phase
X s = synchronous reactance per phase = Xar 4- XL
Z s ~ synchronous impedance per phase
X L = leakage reactance per phase
Xar = reactance due to armature reaction
Ecj) = generated or no-load voltage per phase
E l “ generated or no-load line-to-line voltage
V L = Line-to-line terminal voltage
IL = Line current
la? ~ h = armature current or phase current
Vf = exciter voltage
R f = field resistance
If ~ field current

ALTERNATOR VOLTAGE REGULATION :

% VR= ....VrL x 1 0 0 % = 100%

Alternator on load, the variation of terminal voltage is due to the following reasons:

1. Voltage drop due to armature resistance


2. Voltage drop due to armature leakage reactance
3. Voltage drop due to armature reaction ( major reason )

Vector diagram:

@ i f lagging (b ) if leading ( c ) if unity

in com plex fo rm
E<j> = V 4 + I a ( Z , )
in m agnitude

Where: © is lag’g. p.f. 0 f o r leading p.f. and 0 = 0 for unity


5
LABORATORY TEST FOR ALTERNATORS :

a. DC Resistance Test
While Alternator is at rest, and dc field is open, measure the dc resistance
between each terminals. The average o f three sets o f resistance is called R(.

p, " I ----------------------------- f x

Field is /
Open / Rf
- o- _ /?, H- R2 + R2
R,

b. Open Circuit Test or No load Test


The alternator is driven at synchronous speed, the field current (excitation) is adjusted
from a low value up to that sufficient for voltage reasonably beyond the rated voltage.

E o c - open ckt. e m f per phase


c __ V .
&

c. Short Circuit Test


The alternator is driven at synchronous speed (rated frequency), the field current
(excitation) is adjusted to give 150 % to 200 % o f rated current to How.

I sc ~ short ckt. current per phase

is c -- ! i + l2 ± h -

Open circuit voltage per phase _ ^ o c


zs =
Short circuit current per phase I sc
For wye-connected stator windings

1 Sc

For delta-connected Stator windings

Z s= = - j— R .-1 .5 Rdc R jc|«.


Isc ISC 2.
Js &

Where:
R t = ohmic resistance between terminals (measured by ohmmeter )
Rdc = ohmic resistance per phase
Re - effective resistance per phase
V = voltmeter reading or open circuit voltage between lines
Eoc = open circuit voltage per phase
Lc - short circuit line current

ALTERNATOR EFFICIENCY :

n - fjOlL x \ 00% = -----^ ---- - A i 00% = —■ — A i 00%


P P 4- P P
1 in 1 out 1 loss ai

ALTERNATOR LOSSES

Ratal ion losses 3. Stray load loss


a. Friction and windage loss
b. B ru s h fr ic t io n 4. Exciter Ioss
c. Ventilation lpss
d. Core loss

Electrical losses
a. Field circuit copper loss
b. Armature winding copper loss
c. Brush contact loss

PARALLEL OPERATION OF ALTERNATORS .

The requirements of connecting two alternators in parallel or connecting an alternator to the bus ar

1. The two effective values of voltages are equal at the terminals being connected.
2. The two frequency are the same
3. The tw o^oltages are exactly opposite in phase (in the local circuit o f the two machine).
4. The phase sequence o f the voltages (for polyphase machine) are the same.
5. The two voltages have the same waveforms.

The operation of properly connected synchronous generator in parallel with a system is called
"sy n c h r o n i z i n g If any of the above requirements are not met, there will be cross-current between
alternator winding through the com mon bus-bars, with this cross-current, the generator can not be
synchronize.
MAXIMUM POWER OUTPUT E£R ALTERNATOR

n EV watts . , .
P„,ax = —-------— If Rc IS neglected
A, <t>
Where :

E = em f generated per phase-

V = terminal voltage per phase

• Xs = synchronous reactance per phase

Zs = synchronous impedance per phase

SYNCHRONIZING POWER (P*™) of ALTERNATOR :

Because o f some disturbance on the system, the power angle changes by ij/ (as shown Va = V0 ), w a
the machine to developed an additional power, thereby keeping in synchronism. This additional power is knowa
‘\svnch\ onizingpow er *.

Consider the vector diagram with lagging p.f. load

From the vector diagram


E / a - V J 0° i 0 , ‘■ w #, '
----- —-----Ec / a - (j) - V, / - (]) J
z sr t
1 j
I a - ~ ~ [ Eo cos ( a - (|)) - cos i|) ] + —— [ E0 sin ( a - <|>) + V* sin (|> ]
Zs

Pcj = ( Rectangular ) o f V0 Ia * after simplifying :

pd= _ [ £ o cos ^ _ V^cos ( <|) ~ a ) ]


Zs
When a becomes a + S :

Pd = Ejl [ Ec cos <j> — cos ( (|) + a + 8 ) ]


Zs
A n d since PSYn = ?d “ Pd
n ~ Eoy<
f> r c • /j. . \ , o / j N . v S , watts
P s y n --------— I sin o sin (cj) + a ) + 2 cos ( (j) + a ) sin ‘ —] —-----
2 <t>
EV, waits
/% POWERLINEREVIEWCENTERInc.
/ r o wk Riise \
The Ultimate E.E. Review Center
ALTERNATOR REVIEW PROBLEMS
1. A 60 cycle alternator has 2 poles. What is the speed of alternator?
A. 3600 rpm C. 1500 rpm
B. 1800 rpm D. 1200 rpm

2. A 60 cycle alternator has a speed of 120 rpm. How many poles has it?
A. 24 poles C. 36 pole
B. 20 pole D. 60 pole

3. The effective voltage for a 5-turn coil on a 6 pole generator is 10.6 volts. The flux
per pole is 0.00795 weber. Calculate the frequency of the machine.
A. 30 Hz C. 60 Hz
B. 50 Hz D 40 Hz

4. A 3-phase, 4 pole machine has 36 slots. The winding is double layered and short
rause pitched being made up to 6 turns per coil whose pitch is 140° electrical.
Determine the winding factor.
0.902 C. 0.94
B. 0.96 D. 0.965

5. A 72 slots three phase stator armature is wound for 6 poles, using double layer
lap coils having 20 turns per coil with a 5/6 pitch. The flux per pole is 4.8x10
lines, and the rotor speed is 1200 rpm. Calculate the generated EMF per phase.
A. 6000 volts C. 6480 volts
B. 5680 volts D. 6000 volts

6. A generator is rated 100 MW, 13.8 kV and 90 % power factor. The effective
. resistance to ohmic resistance is 1.5. The ohmic resistance is obtained by
connecting two terminals to a 6 volts d.c. source. If the current 43.8 Amperes,
what is the percent resistance per phase ?
A. 1.5% C. 4.5%
B. 6% D. 6.9%

7. A 150 kVA, 1000 volts three phase wye-connected alternator has an open circuit
' emf of 1000 volts. When an alternator is short circuited at the same excitation, the
armature current is 460 amperes. What is the synchronous impedance?
A. 1.26 ohms C. 2.17 ohms
B. 2.23 ohm£ D. 3.23 ohms

8. A 3-phase, 60 Hz wye connected round rotor synchronous generator rated 10


kVA, 230 volts has a synchronous reactance of 1.2 ohm per phase and an
armature resistance of 0.5 ohm per phase. Calculate the percent regulation at full
load with 0.8 lagging power factor.
A. 12.8% C. 21.8%
B. 28.1% D. 18.2%
9. A 100 kva, 400 volts, wye connected salient pole synchronous generator runs at
full load and 80% power factor leading. If Xd=2Xq=1.1 ohms per phase and Ra is
negligible, calculate the voltage regulation.
A . -26.4% C. -24.6%
B. -64.2% D. -246%

10. A 30 kVA, 220 volt, wye connected, 3-phase, salient pole synchronous generator
supplies rated load at 0.707 lagging power factor. The reactance per phase are
Xd=2Xq=4 ohms. Determine the percent voltage regulation.
A. 100% C. 224%
B. 600% D. 10%

11. A generator at no-load operates 61.2 Hz, slope 1mHz/W. Load 1= 1200W and
load 2=900W. Find the frequency before and after adding the load 2.
A. 60 Hz & 59 Hz C. 61.2 Hz & 59.2 Hz
B) 60 Hz & 59.1 Hz D. 61.3 Hz & 59.3 Hz

12. A 250 MW, 60 Hz turbine generator set has a speed regulation of 5% based on its
own rating. The generator frequency decreases from 60 Hz to a steady vaiue of
59.7 Hz. Determine the increase in the turbine power output.
A. 25 MW C. 50MW
B. 12.5 MW D. 20 MW

13.Two generating units rated 250 MW and 400 MW have governor speed regulation
of 6% and 6.4% resp. from no load to full load. They are operating in parallel and
share a load of 500MW. Determine the load shared by each unit.
A. 190M W & 310 MW C. 220 MW & 280 MW
B. 225MW & 275 MW D. 240 MW & 260 MW

14.A 3<|) Y-connected , 1000 kVA, 6.6 kV turbo-alternator has a synchronous


reactance of 8.7 ohms per phase but negligible resistance . While it is
supplying full load at 80% power factor lagging to a large power system, the
excitation is reduced by 5 % . What is the new value of operating power factor?
A. 87.5 % C. 77.2 %
B. 93.1 % D. 9 5 . 2 %

15. A 60 MVA, 69.3-kV, three phase synchronous generator has a synchronous


reactance of 15 ohms per phase and negligible armature resistance. The
generator is delivering rated power at 80% power factor leading at rated terminal
voltage to an infinite busbar, what is the maximum three phase power that the
generator can deliver before loosing its synchronism.
A. 288 MW C. 96 MW
AC Machines (Alternator) Supplementary Problems

1. In large capacity ac generator, the moving part ja


A. Brushes { C) Poles
B. Armature D. Frame
1
2. Calculate the electrical displacement between adjacent slots of a 16 pole alternator
with 144 slots.
A. 30° C. 60°
B. 20° D. 180°

3. The induced em f in each coil side of a certain alternator winding is calculated as 10


volts, if the measured voltage across the coil is 19.32 volts, the winding is short by
how many electrical degrees?
A. 30° C. 60°
B. 50° D. 20°

4. For a 3-phase winding with 6 slots per pole per phase and with coil span of 15 s lo t,
the value of pitch factor is
A. 0.851 C. 0.951
B. 0.966 D. 1.05

5. Damper or “Amortisseur” windings are used in alternators t o ......


A. achieve synchronism C. reduced windage losses
B.prevent hunting D start up

6. What is the frequency of the generated emf of a 10 pole alternator driven at 720 rpm?
A. 50 hz C. 60 hz
B. 55 hz D. 25 hz

7. A 6 pole, three phase AC generator has 72 slots, the coil span is 12 slots. What is the
pitch factor ?
(jy - 100 C. 0.985
B' 0.939 D. 0.966

8. A 3-<t», 60 hz , 10-pole alternator has 120 slots and 4 turns in every coil. The coil pitch
is short by 2 slots. The winding distribution factor of the alternator is ......
A. 0.925 C. 0.987
B. 0.967 ; 0.957

9. ■A three phase alternator has three armature coils each rated 1200 volts and 120
amperes. W hat is the kVA rating of this unit?
A. 288 . - C. 432
B. 249.4 /f*h 144

10. An alternator oft open circuit generates 360 Volts at 60 Hz when the field current is 3.6
Amp: Neglecting saturation, determine the generated emf at a frequency of 30 Hz and ‘
a field current of 3.0 Amp.
A. 240 Volts C. 160 Volts
B. 300 Volts D. 150 Volts.
11 Determine the distribution factor for a 3- phase winding in which there are 18 slots per
pole.
A. 0.966 -CL 0.958
B. 0.957 D. 0.956

12. Calculate generated emf per phase of a 10 pole, 3-phase, 60 Hz alternator with 5 slot
per pole per phase and 4 conductors per slots in two layers. The winding is short by 3
slots & the flux per pole is 0.12 W eber
A. 2908 Volts C. 720 Volts
B. 1000 Volts D. 3200 volts

13. A 100 MVA, 13.8 kV, 3<j>, Y-connected alternator will have a per phase nominal
impedance o f ......
A. 7.5 ohms d ) 1.9 ohms
B. 2.9 ohms u. 3.5 ohms

14. A generator is rated 100 MW, 13.8 kV and 90 % power factor. The effective resistance
to ohmic resistance is 1.5. The ohmic resistance is obtained by connecting two
terminals to a d.c.source. The current & voltage are 87.6 Amperes and 6 Volts
respectively . W hat is the percent resistance?
Ai 3 C. 4.2
B. 2 D. 1

15. A wye-connected alternator was tested for its effective resistance. The ratio of the
effective resistance to ohmic resistance was previously determine to be 1.35. A 12-V
battery was connected across two terminals and the ammeter read 120 amperes.
What is the per phase effective resistance of thf«.alternator?
A. 0.135 ohms 0.0675 ohms
B. 0.117 ohms D. 0.0234 ohms

16. Dirt accumulation in generator can cause all of the following except
A. low power factor C. poor voltage regulation
B. flashover D. overheating

.17. A generator is rated 100 MW, 13.8 kV and 90 % power factor. The effective resistance
to ohmic resistance is 1.5. The ohmic resistance is obtained by connecting two
terminals to a d.c. source. The current & voltage are 87.6 Amperes and 6 Volts
respectively . What is the resistance per phase ?
A. 0.0513 ohm C. 0.0685 ohm
B. 0.342 ohm D. 0.0617 ohm

18. A generator is rated 100 MW at 22 kV, and 90% power factor. The effective ac
resistance to ohmic resistance is 1.35. The resistance is obtain by connecting two
terminals to a 12 volts dc source. If the current is 120 Amperes, What is the percent
effective resistance per phase ?
A. 1.5% C. 4.5%
B. 6% D. 6.9%

19. A dc resistance test is conducted on a 3-phase 200 kva, 240 volts, delta connected
generator. When the two terminals are connected from 6 volts dc source, the ammeter
inserted in the line reads 150 amperes. The effective resistance to ohmic resistance is
1.25. What is the percent resistance?
A. 7.5% C. 26%
20. Find the synchronous impedance of a 3-phase synchronous generator in which a
given field current produces an armature current of 200 amperes on short circuit and a
generated emf of 1050 volts on open circuit.
A. 1.72'ohms C. 9.1 ohms
B. 5.25 ohms D> 3.0 ohms

21. Synchronous impedance test is taken on a 3-phase generator. Under short circuit
condition, the currents in the three lines are 26.2 A, 23.7 A, and 27.4 A. What
current should be used for the test ?
A. 26.3 A C. 24.6 A
E| 25.7 A D. 25.4 A

22. If in a 3-phase alternator, field current of 50 A produces a full load current of 200 A on
short circuit and 1730 V on open circuit, then its synchronous impedance i s .........
ohm,
A. 8.66 C. 4
B. 5 • D. 34.6

23. A 100 MW, 13,800 volts 90% power factor synchronous generator has an ohmic
resistance of 0.1 ohm and synchronous reactance of 0.42 ohm is operating at rated
condition. Find the angle between the voltages.
A. 9.55 deg C. 6.72 deg.
B. 7.45 deg. D. 3.25 deg.

24. A single phase generator with full load current of 450 amperes at 2400 volts has 0.08
ohms resistance and synchronous reactance is 2.8 ohms. Determine the voltage
regulation in percent at 75% power factor lagging.
A . 55 C 64
B. 41 D. 75
V —*
25. A three phase, 318.75 kVA, 2300 V alternator has an armature resistance of 0.35
ohms per phase and a synchronous reactance of 1.2 ohms per phase. Determine the
vojtage regulation at full load 0.6 power factor leading.
-A .;-4.33% C. 4.33%
B. 6.12% D. -6.12%

26. The power factor of an alternator is determined by its


A. load C. excitation
B. speed D. voltage rating

27. A 3-phase, 1000 kVA, 2,300 volts alternator is short circuited and open circuited and
is operated at rated speed to produce rated current. With the short circuit removed
and the same excitation, the voltage between stator terminals is 860 volts. The
effective ac resistance between stator terminals is 1.2 ohms.. Determine the percent
' voltage regulation at 0.9 power factor lagging
A. 29 C. 32
B. 15 D. 26.7
*

28. A 500 kVA, 3,600 Volt, 3-phase, 60 Hz alternator has an effective armature
resistance per phase of 0.40 ohm. Mechanical and iron losses is 10 kW, field current
at full load unity pf is 70 Ampere. The exciter voltage is constant at 120 Volts. What
is its efficiency when supplying incandescent lamp at rated load?
A. 93.4% C. 92.3%
B. 89.7% D. 95.0 %
29 A 25 kVA 220 volt three phase alternator delivers rated kva at 0.84 power factor. The
effective ac resistance between the armature winding terminals is 0.18. The field takes
9.3 amperes at 115 volts. If the friction and windage loss is 460 watts and the core
loss is 610 watts. What is the Hp delivered by the prime mover?
A. 32.6 C. 28.15
B. 31.54 D. 33.23

30. In a large alternator, dampers


A. reduce frequency fluctuations C. increase stability
B. reduce voltage fluctuation D. none of the above

31. A generator is being synchronized to a large system. The actual system voltage and
frequency are 13.7 kV and 60 Hz respectively. The generator voltage and frequency
are 13.9 kV and 60 Hz respectively when the generator is switched to the system.
Choose which one happens.
A. Generator delivers both MW and MVAR (C. .Generator delivers MVAR
B. Generator delivers MW u. Generator takes in MW

32. It is never advisable to connect a stationary alternator to live bus-bars because


A. It is likely to run as synchronous motor
It will get short circuited
C. It will decrease bus bar voltage though momentarilly
D. It will disturb generated emf of other alternator connected in parallel

33. An alternator is operating with 100 Amp field current. If the field current is increased
to 125 Amp with the same electrical load on the machines, it will
A. be less likely to go out of synchronism C.j operates at new power factor angle
B. be more likely to go out of synchronism ''”D. overheat

34. A generator is being synchronized to a large system. The actual system voltage and
frequency are 13.8 kV and 60 Hz respectively. The generator voltage and frequency
are 13.8 kV and 60.1 Hz respectively when the generator is switched to the system.
Choose which one happens.
■A. Generator delivers both MW and MVAR C. Generator delivers MVAR
B. Generator delivers MW D. Generator takes in MW

35. The driving power from the prime mover driving an alternator is lost but the alternator
remain connected to the supply (i.e. its power circuit breaker failed to trip) and its
excitation is on. The alternator will ....
A. behaves as an induction motor
B. stop & get burnt
C. behaves as synchronous motor but will rotate in reverse direction.
D. behaves as synchronous motor and rotate at same direction .

36. Increasing the speed of alternator, what will happen?


A Decrease MW C. Increase MVAR
B. Increase frequency D. Decrease MVAR

37. The power factor of an alternator is 75 %. The operator is ordered to increase the
power factor to 80 %. What shall he do ?
A. increase the voltage C. operate the governor
B. increase the excitation D. decrease the excitation
38. Two 1500 kVA ac generators running in parallel supplies the total load of 2,250 kW at
0.90 p.f. lagging . One machine is loaded to 1000 kW at 0.85 p.f. lagging. What is the
kVA and p.f. of the other machine?
A. 1750 & 0.8 lagging 1335 & 0.94 lagging
B. 470 & 0.96 leading >• _ D. 1250 & 0.78 leading

39. Two 60Hz alternators are driven by shunt motors. The shunt motors have speed-load
droop characteristics of 3 % and 4 % respectively. The alternators are in parallel and
each carrying 50 kW. There is no automatic speed-load control. An additional 50 kW
load is switched-on. What are the resulting loads of the alternators assuming that the
speed-load control of eachis not adjusted?
A. 78.57 k W & 71.43 kW C. 78 kW & 75kW
B. 82.51 k W & 67.49 kW D. 80.31 kW & 69.69 kW

40. Two identical 2000 kW alternator operates in parallel. The governor of the first
machine is such that the frequency drops uniformly from 50 Hz at no-load to 48 Hz at
full-load. The corresponding uniform speed drop of the second machine is 50 Hz to
47.5 Hz, W hat is the maximum load that can be delivered without overloading either
machine ?
A. 3600 kW C. 6300 kW
B. 4000 kW D. 3800 kW

41. Two-3cj) alternators operating in parallel delivers power at a line potential of 2200 Volts
to an inductive load of 150 kW at 80% p.f. If the armature currents of the two
alternators be equal one be operating at unity p.f. How much power does each
alternator supply to the load ?
A. 117.2 kW for unity p.f.& 32.8 kW C. 112.7 kW for unity p.f. & 38.2 kW
B both 75 kW one at unity p.f. D. 100 kW for unity &50 kW

42. A wye-connected turbo alternator having s synchronous reactance of 0.5 and


negligible resistance is connected to a large power system having a busbar voltage of
13.8 kV supplying a load of 15000 kVA at 0 8 lagging power factor. If the steam supply
is cut-off, what current will the machine carry assuming negligible losses?
A. 256.55 A C. 525.62 A
•B. 384.26 A D. 627.55 A

43. An ac generator 1500 kVA capacity has a full load efficiency of 91% at 100% power
factor and 86% efficiency at 80% power factor lagging. What engine drive in Hp is
required if the genset is to supply entirely incandescent lamp?
A. 3125 C. 1980
B. 2525 . D. 2210

44. A 50 MVA, 13.8 kV, 3<j), Y-connected alternator will have a per phase nominal
impedance o f ......
A. 7.5 ohms C. 3.8 ohms
B. 2.9 ohm 9- D. 15 ohms

45. A three phase salient pole, 440 volts, wye connected synchronous generator operates
at a power angle of 20° while developing power of 36 kW. The machine constant are
Xd=2Xq=5 ohms and Ra is negligible. Calculate the percent voltagfe regulation
A. 70% C. 78%
B. 144% D. 54%
46. A generator at no-load operates 61 Hz, slope 1MW/Hz. Load 1= 1.0 MW and load
2=0.8MW are connected. Find the frequency before and after adding the load 2.
A. 60 Hz & 59 Hz C. 60 Hz & 59.2 Hz
B. 60 Hz & 59.1 Hz D. 60 Hz & 59.3 Hz

47. Out of the following conditions, the one which does not have to be met by alternators
working parallel is the machine must
A. have equal rating C. be the same terminal voltage
B have the same phase rotation D. operate at the same frequency

48. A 75 kVA single phase, 240 volts, 60 Hz alternator has an effective armature
resistance of 0.02 ohm per phase and an armature leakage reactance of 0.08 ohm.
Calculate the induced emf when supplying rated load at unity pf.
A. 245.5 volts C. 249.2 volts
B. 250.4 volts D. 247.5 volts

49. A 1000 kVA, 3300 volt, 60Hz, 3-phase, 4 pole, wye connected, wound rotor
synchronous generator has an impedance of Zs=0+j1.0. The generator is operating at
full load 0.8 pf lagging to an infinite bus. Calculate the synchronizing power per phase
if a disturbance causes the power angle to swing by 1°(mech’l).
A. 133.6 kW C. 145.2 kW
B. 125.4 kW D. 168.9 kW

50. A 50 MVA, 30 kV 3-phase synchronous generator has synchronous impedance of


0+j9 Q/phase. The generator is delivering rated power at 80% pf lagging at the rated
terminal voltage to an infinitebus. With the same excitation, what is the steady state
maximum power the machine can deliver before losing synchronism?
A. 126MW C. 136 MW '
B. 108MW D. 256MW

51. Due to which of the following reasons, for aircraft alternators high frequency is used?
A. To reduced the bulk C. To compensate for high speed '
B. to compensate for high altitude D. To free the systems from external disturbance

52 At leading power factor, the armature flux in an alternator


A. distorts the rotor flux C. aids the rotor flux
B. opposes the rotor flux D. does not affect the rotor flux

53 In an alternator, when the load power factor is unity the armature flux
A. will be demagnetizing C. will be cross-magnetising
B. will reduced to zero D. will have square wave form

54. An exciter for generator is a


A. shunt motor C. series motor
B. shunt generator D. none of the above

55. The maximum current that can be supplied by an alternator depends on


A. exciter current C. strength of the magnetic field
B. number of poles D. speed of the exciter

PRACTICE Makes Perfect!!!


(TR AN SFO R M ER S)
This Free Quality Manual is
Found only at POWERLINE

POWERLINE REVIEW CENTER


MEZZANINE FLOOR, DONA A M P A R O BUILDING
CO RNER ESPANA & G. M. TOLENTINO STREETS
SAM PALOC, M ANILA
TEL. NOS. 735-73-02 & 733-21-18

CEERS-:
TEL. NOS. (03)261-2244 & (032)261-8452
POWERLINE ENGINEERING REVIEW CENTER, INC.

AC MACHINES (TRANSFORMER) REVIEW LECTURE

TRANSFORMERS
Is an electrical machine that transfers electrical energy from one circuit to another
without changing the frequency, it works on the principle of electromagnetic induction, also its
windings are linked by a common magnetic circuit

Classification o f Transformers

1. According to the core construction:


a. Core type
b. Shell type
c. Berry type
2. According to the purpose for which used:
a. Transmission transformer
b. Distribution transformer
c. Generator transformer
d. Testing transformer
e. Instrument transformer
1. current ( C T )
2. potential ( PT )
3. According to the method o f cooling:
a. Self-cooled (AA)
b. Forced air cooled (FA)
c. Oil-self-cooled (OA)
d. Forced- oil cooled with air or water (OA/FA)

IDEAL TRANSFORMER
A transformer is considered ideal if there is no voltage drop across its windings, if resistance and
leakage flux are neglected, in which losses are neglected, if losses are neglected, power input is the same
as power output.

PRACTICAL TRANSFORMER WITHOUT LOAD & VECTOR DIAGRAM

Notations :

NP, Ns = number of turns of primary and secondary winding respectively.


Vp, Vs =■■primary impressed and secondary terminal voltages respectively.
Ep, Es = primary and secondary induced voltage respectively.
rp, rs = primary and secondary winding resistance respectively.
xP, xs - primary and secondary leakage reactance respectively .
cpm ' mutual flux, useful flux which links the primary and secondary.
2

cp f = leakage flux, wasted flux


IN - no-load primary current = I ^ + I h+

Core loss
Ih+e - hysteresis and eddy current component =? — ■■
». ------
EP
Iex = exciting or magnetizing current

The total induced emf in each winding of the transformer is proportional to the number of
turns of the winding i.e.
N P E P
a ~ -----= ----- = ratio■ oj jr transformation
Es

TRANSFORMER GENERATED VOLTAGE EQUATION


EP =4A4 / (pm N p in rms Volts
Es = 4.44 / <pm N s in mis Volts

Where :
f = frequency in HZ
<pm = mutual flux in Wb
N = number turns for every windings
4.44 - a constant equal'to 4(1.11) where 1.11 is the form factor of sinusoidal wave
, r, RMS value1/ v 2 m ax. value
Note : K f = ---------------- = 1------ J.--------------- = 1/11
AVE value (2//r)m ax. value

PRACTICAL LOADED TRANSFORMER & VECTOR DIAGRAM

Ip - load component of primary current its magnitude is such that


mmf primary = mmf secondary
/ P NR—IsNs
Np Is
Ns ~ ^ ~a
since / N « I fp , thus I p « l'p

Ns ~ Ip
3

TRANSFORMER EQUIVALENT CIRCUITS:

a. Approximate equivalent circuit referred to the primary

Where:
ReP

Xe, = total equivalent leakage reactance referred to .


primary = XP + a2 Xs

Rr, Xm - core loss equivalent resistance (accounts for eddy current loss and hysteresis loss) &
equivalent magnetizing reactance respectively.

f'p = a Vs +1 p (Rep + j X ep) neglecting I N


b. Approximate equivalent circuit referred to the secondary
R es \e s

Secondary = rs + —
a
Xes = total equivalent leakage reactance referred to

Secondary = x s + - j
a2
zes = R c s + j X es = 4 RJ + X J

TRANSFORMER T OSSES AND EFFICIENCY

Input Output + Losses Input


TRANSFORMER LOSSES

L - are fixed losses result of hysteresis and eddy current losses in the magnetic
circuit ( core ) of the transformer
yi.6 ^
Hysteresis Loss, P*- - Kh f £m = Kh
r
Ph ocf ;Ph = Khf ►if (pni & pm is constant

Eddy Current Loss, ? e = K*f 2 Pm2 - K^V2;


Pe cc f 2; Pc * Kc f 2 => if cpm & P,„ is constant

/Vote; Ph & Pe can be determined by open circuit test.

2. \ ~ vary with the load and result of heating of transformer windings.

?<*= Ip21¥ + Is2rs :=> based from exact equivalent circuit

Pcu = lp2Rtfp- I$2R.s based from approximate equivalent circuit.

Note: P£u can be determine by short circuit test

a. Gp ?n circuit test
Xi Hi
rj ~ ® ~* !~fr lf
VVvV i
TO

Supply of normal SP Open


Voltage & frequency Y .l L-T,
i HM side
{
}
o~ u t
Wattmeter reading % core loss
Ha

— i L
r - © - 1h W
r i somatirrms
e m itte d
Source of reduced ---------- 1
voltage at the input
until rated current flows
© H,r,
(A tS ^ h o r te d
L y *slde
I i
f

Hz Xz
i
Wattmeter reading (W )» full - load copper loss

Let Voltmeter reading (V) * Ym


Ammeter reading (A) ~ Isc '
W
CONDITIONS OF MAXIMUM EFFICIENCY:

r O UT VI cos 6
*100%: - jc100%
VI cos# + 1 2R„ + Pk

dij
to have max. efficiency : take — = 0

drj _ (V Icos# + / 2/?e + PK)(V c o s0 )-V Ic o s'0 (y cos^ + 2/R e)


=0
dI (vicosd + I 2Re + PKJ

dividing by Vcos 0 & simplify:

?R . - P k
Variable loss = Constant loss
Also ;

ALL- DAY EFFICIENCY:


It is the ratio of the energy delivered with in 24 hr period to the energy input m the same period of
time . For distribution transformers whose primary energized for 24 hrs a day, thus the core loss of this
transformer must be designed to be low as possible. It is always less than the commercial efficiency.
ENERGYoutput / day
x 100%
t?all~day ~ ENERGYoutput Tday + ENERGYL0SSI day

at max. all day efficiency :

Energy Copper Loss = Energy Core Loss

TRANSFORMER POLARITY
Connection:
Conditions: ® When voltmeter reading is greater than the input test voltage the polarity is
Vp 1
A D D ITIV E . V = VP' + - P

® When voltmeter reading is less than the input test voltage the polarity is
V p'
SU BTRACT I V E V = Vp

Note: By industry standards all lcp, 200 kVA, 8660 Volts or less are ADDITIVE .

AUTOTRANSFORMER:
The autotransformer is one having a single continuous winding, part of which is common to both
primary and secondary circuits. It can be used for either step-up or step-down application with a
relatively small change in voltage between the input and output voltages, and should not used when there
is a large change in voltage because the circuits are connected electrically.

Step-down Autotransformer ( a > 1 )


%--------- +,-------------\ °
b &
! c Is i
I ac
s Icb N cb
vf O Vs cb
I
s I
-JL
This autotransformer is equivalent, in its transformation, to an ordinary two-winding transformer
if winding ab is regarded as primary winding , with winding be as the secondary winding as

Such: N ab _ Eab I.
N, be he

Hence : Using the approximate equivalent circuit:


Referred to secondary

I Rep x ep I
t i
r i
i
fa - ! ) V s
Vp i
I i
I i
i
t __
Where:

Re p = rp + ( a - 1) rs Rn = r< + -
(a-1 )2

Xep = xP + ( a - l) 2 xs
(a - 1)
Step - Up Auto transformer ( a < 1)
a ,----------- y -
N be
i
i
Vs
be I be ^_bc_ _
I
I ab Eab lp-h
6

Conditions: ® When voltmeter reading is greater than the input test voltage the polarity is

A D D ITIV E. V - Vp ’ + V"
a
® When voltmeter reading is less than the input test voltage the polarity is

SU BTRACTIVE^ y = Vp' Vp '

Note: By industry standards all lep, 200 kVA, 8660 Volts or less are ADDITIVE .

AUTOTRANSFORMER:
The autotransformer is one having a single continuous winding, part of which is common to both
primary and secondary circuits. It can be used for either step-up or step-down application with a
relatively small change in voltage between the input and output voltages, and should not used when there
is a large change in voltage because the circuits are connected electrically.

Step-down Autotransformer ( a > 1 )


%--------- * ------------- - fl
e
S IT
e Is r\

a = Ll v
j l
Icb
1 Vs Mcb cb
I i
i
2c _ jl
This autotransformer is equivalent, in its transformation, to an ordinary two-winding transformer
if winding ab is regarded as primary winding , with winding be as the secondary winding as

E gb j_cb_ _ [ ,
Such:• _
-1 * = a - l
be he

Hence : Using the approximate equivalent circuit

r
i
i*
/ V - 'W P -------------
Rep *ep
TTt
I
i
i
Vp (a -l)V s
I
i
i
i
__
Where:

R cP = r p + ( a - 1) 2 rs Rr, =/■,+-

XeP = xP + ( a - l) 2 xs Xgs X' +


(a-l)2
Step - Up Auto transformer ( a < 1)
o ,------------ V-
r
& T i
\
Here: a Hi be

B N. ac Ipr
Vs
lb c Njc, . ^_bc_ h
N ab Eab
7

As an equivalent ordinary two winding is


"U: . «

P ow er indu ctively tran sform ed (thru ob)P ow er conductive] F Transf o rm ed (th ru be)

?Tr “ (Vp - Vs ) I* = Pi„, ) P- = P“> )

PARALLEL OPERATION OF TKANSFOKMER^ I

Important conditions that must be fulfilled if two or more transformers are to be operate
successfully in parallfcV to deliver a eomrnon load:

1 Voltage ratings of both. primaries and secondaries must he identical, or the transformation ratio are
the same, (otherwise, current will circulate tluough the: liaiisformer windings which will contribute
to the losses of the transformer ).

2. The transformer must be properly connected with regards to polarity, (otherwise , a large current will
circulate through the transformer winding which could damage the windings ).

a. The equivalent impedances should be inversely proportional to the respective kVA ratings.
( otherwise, the transformer will not share the load in proportional to their ratings).

' Z' a - ; ~ i. = £ l
S Z e2
b. The ratio of the equivalent resistance to the equivalent reactance of the all transformer should
be the same.( Otherwise, each transformer will not lakes its correct fraction of the load)
Re, Rc 2
“ Acs

Also the condition for two transformers connected in parallel to operate at the same p.f.

F or unequal transform ation ratio (a2 & a >)the secondary currents will b e :

Is = (a2 Js ^ °l ^ l / z w - (n-i - OlW-h


a lZesr +a2Zes7 a] Zcs{ +a2Zes2

For the circulating current (set ILoad ~ 0 )


{ a 2 - a t )V.yt ._
1C{ —.......... .... ........... 01 1L.J—---- ------—
---------
a j Zes r -ir a2Zes 2 a\Zes, -t- a 2 Zes

MOST COMMON 3<p TRA N SFO RM ER CONNECTIONS:

1- ¥ - V with connected neutrals


has the advantage the windings are subjected to the line voltage divide by v3 . This
connection without neutrals arc rarely used since high unequal voltage may result from an
unbalanced load.
2, A —A connection
suitable for moderate voltage, large current operation.

3. A —Y connection
Primarily used to step-up the voltage. It is also used in step - down applications where a
conventional distribution voltage is reduced to supply two voltages for lighting and power
loads

4. Y ~ A connection
use for stepping down voltages

5. T (scott) connection
used to convert 3(p to 2cp or vice - versa.

6. V - V (open delta) connection


a) used when the load is still small so that to use of A - A bank is unwarranted.
b) used if one transformer of the A - A bank fails .

7. A utotransform er —also called as a “ compensator ” used for reduced voltage starting, also
used for stepping down or up the voltages.

PARALLEL OPERATION OF 3<p TRANSFORMER :

OPERATIVE PARALLEL CONNECTIONS


LV SIDE HV SIDE LV SIDE HV SIDE
BAN K A - A A - A B A N K BANK A- A A - Y BANK
A B A B
Y -Y Y -Y A -A Y- A
A -Y A -Y Y -Y A- Y
Y -A Y -A Y -Y Y- A
Utility
A - A Y -Y
* a
Y - Y A -A

Bank A Bank B

Load
Note : In dealing with 3<p transformer, calculations are made for one phase only.

INSTRUMENT TRANSFORMERS :

Instrument transformers are o f two kinds :


1. Current transformer ( CT )
2. Potential transformer ( PT )
These are used to supply power to ammeters, voltmeters, wattmeters, relays and so on,
Which is called the “Burden o f Instrument Transformer\
POWERLINe rev iew c en ter Inc.
/ P O W E R I. IJN E \
The Ultimate E.E. Review Center
AC MACHINES (TRANSFORMER) REVIEW PROBLEMS
1. The voltage, v(t)= 100 sin 377t-20sin1885t is applied to a 200 turn transformer
winding. Determine the rms value of the voltage and core flux.
A. 72.1 V and 0.94 mWb C. 102 V and 1.33m Wb
B. 144.2 V and 1.88 mWb D. 124.9 V and 0.0053 mWb

2. The 2300 volt primary winding of a 60 cycle transformer has 4800 turns. Calculate
the number of turns in the 230 volt secondary winding.
A. 480 turns C. 4800 turns
B. 48 turns D. 240 turns

3. A certain 10 kVA 60 cycle transformer with primary voltage rating 2400 volts,
secondary 240 volts, has a core area of 12.25 sq. in. The primary is wound with
1100 turns and the secondary with 110 turns. Calculate the flux density in Tesla.
A. 6.686x1 O'4 C. 1.036
B. 6.686x1 O'3 D. 8.19x10"3

4. The primary with 200'turn on the H winding is to be wound to step down from 240
volt to 120 volts. Find the number of turns on the X winding.
A. 100 turns C. 400 turns
B. 200 turns D. 50 turns

5. A 100-kVA, 60-Hz, 2200A//22O-V transformer is designed to operate at a


maximum flux density of: 1 T and an induced voltage of 15 volts per turn.
Determine the cross-sectional area of the core?
A. 0.0128 m2 C. 0.0236 m2
B. 0.0432 m2 D. 0.0563 m2

6. A transformer supplies: a load with 30 amp at 240 volts. If the primary voltage is
2400 volts, find the primary current and volt-amp.?
A< 3A & 7,200 volt-amp C. 300A & 7,200 volt-amp
B. 0.3A & 7,200 volt-amp D. 30A & 7,200 volt-amp

7. A 25 Hz, 120/30 volts, 500VA transformer is to be used on a 60Hz source. If the


flux remain unchanged, determine the maximum load and primary voltage.
A. 500 VA & 120 V C. 500 VA & 288 V
B. 1200 V A & 288 V D. 1200 VA & 120 V

8. An ideal transformer has a voltage rating of 2400/240 volts, a 50 A load is


connected to the low voltage winding. This load must have exactly 200 volts
across it. With 2400 volts applied to the high voltage winding, what resistance
must be added in series with the transformer, if located in the HV winding?
A.)80Q C. 50 Q
B. 30 Q D. 60 Q
9. A 10 kVA single phase transformer designed for 2000/400 volts has the following
constants: R i =5.5;R2=0.2; Xi=12; X2=0.45 ohms. Calculate the equivalent
impedance referred to primary.
A. 10.5+j 23.25 ohms C. 5.7+j 12.45 ohms
B. 0.42+j 0.93 ohm D. 14.5+j 12.45 ohms

10. A 2300/230 volt, 20 kVA transformer has the following short circuit test data:
Psc=250 watts; Vsc=50 volts and lsc= 8.7 amp. Calculate the percent regulation
at full load 0.7 power factor lagging.
A/2.15% C. 3.9%
B. 4.18% D. 5.7%

11. A 5 kVA transformer has the nominal voltage rating of 1100/110 volts. With the
low voltage winding short circuited, it is found by experiment that 33 volts is
required to circulate rated full load current and the corresponding power input is
85 watts. Find the percent regulation when the load takes rated current at a power
factor of 80% lagging.
A. 2.85% C. 3.9%
B. 2.18% D. 2.7%

12. A 2300/230 volt, 10 kVA transformer has the following constants: 1 percent
resistance & 10 percent reactance. Determine the voltage regulation of the
transformer at full load 0.8 power factor leading.
A. -4.8% C. -6.9%
B. - 3.8% D. -5.9%

13. A small single phase transformer has 10.2 watts no-load loss. The core has a
volume of 750 cu cm. The maximum flux density is 10,000 gauss and the
hysteresis constant of the core is 5 x 10'4. Using the Steinmetz law, determine the
eddy current loss.
A. 4.55 Watts C. 5.55 Watts "
,B. 3.55 Watts D. 2.55 Watts

14. A certain transformer has a core loss of 100 watts at 30 Hz and core loss of 300
watts at 60 Hz. What is the hysteresis and eddy current loss at 50 Hz?
A. 83.5W& 138.9W C. 100W&200W
B. 83.5W & 222.4W D. 75W&175W

15.The eddy current loss in a 2300 volts 60Hz transformer is 280 watts. What is the
eddy current loss at 2400 volts 50 Hz source?
A. 280 Watts C. 305 Watts
B. 300 Vyatts D. 257 Watts

16. The hysteresis loss in a 6,600 volts, 50 Hz transformer is 480 Watts. What will be
the hysteresis loss when the transformer is connected to a 6,900 Volts, 60 Hz
source ?
A. 382 Watts C. 602 Watts
B. 462 Watts D. 418 Watts
17. A 4400 volts 60 Hz transformer has a core loss of 840 watts, of which one third of
which is eddy current loss. Determine the core loss when the transformer is
connected to 4600 volt 50 Hz supply
A. 977 W C. 1066 W
B. 907 W D. 904 W

18. A 50 kVA 2200/220 volt 60 Hz transformer has a core loss determine by the open
circuit test, of 350 watts and the copper loss, at rated current of 630 watts,
determined by short circuit test. Find the efficiency at three fourths of rated load
80% power factor.
A. 97.7% C. 98.2%
B. 97.6% D. 98.8%

19. A 10 kVA distribution transformer has 240 volts secondary winding. The
equivalent internal resistance of this transformer, referred to that winding is 0.048
ohm. The core loss of this transformer is 75 watts. At what kVA load will this
transformer operate at maximum efficiency?
A, 9.48 C. 8.44
B. 12.6 D. 16.86

20. The resistance of a 500 kVA, 11000/2300 volts 60 Hz transformer are respectively
0.8 and 0.04 ohm for high and low voltage sides. The iron loss is 3,000 watts.
This transformer is loaded daily as follows: 2 hrs at full load, 3 hrs at three quarter
load, and 6 hrs at one-quarter full load. Calculate the all day efficiency.
A. 94.7 C. 97.1
B. 95.7 D. 98.7

21. A 20 kVA load is to be supplied at 500 volts. An ideal step up auto-transformer is


used to connect this load to a 400 volt source. Find the current in the common
winding.
A. 10A C. 50A
B.20A D. 40A

22. A 10 kVA, 440/110 volts 60 Hz, single phase transformer has full load copper loss
is 150 watts and the core loss is 100 watts. What is the kva rating if it is used as
550/440 volts autotransformer?
A. 50 C. 12.5
B .25 D. 20

23. A 60 Hz two winding transformer is rated 3 kVA, 220/110 volts. This transformer is
reconnected as a step up auto transformer to deliver 330 volts to a resistive load
when the input is from 220 volt source. Find the value of resistance for which
rated current will flow in each winding.
A. 27.3 ohms C. 82.8 ohms
B. 40.9 ohms D. 12.1 ohms
24. A transformer when it is operated as a conventional two winding transformer is
rated 10 MVA, 12/1.2 kV, 60 Hz. Under these conditions, its series resistance and
reactance are given as 2% and 7% pu respectively. This transformer is to be used
as 13.2/12 kV step-down autotransformer in a power distribution system. What is
the transformer’s series impedance in pu in the autotransformer connection?
A. 0.0017 +j0.0058 C. 0.0020 +j0.0070
B. 0.0022 +j0.0078 D. 0.0018 +j0.0064

25. A 1500 kVA , 3% impedance transformer is to be operated in parallel with a 1250


kVA, 3% impedance transformer, the total load is 2500 kVA, What kVA load
carried by each?
A. 750 & 1750 C. 1200 & 1300
B. 1363 & 1137 D. 909 & 1591

26. Two (2)-75 kVA,1<|) transformers are connected open-delta, calculate the
maximum kVA of a 3-phase balanced load without overloading any of the
transformers.
A .225 C . 100
B. 150 D. 130

27. Three single phase, 10 kVA transformer connected delta-delta supply a balanced
3-phase wye-connected load of 30 kVA. One transformer is removed for repair.
Find the kVA overload of each remaining transformers.
A. 5 C. 7.32
B. 12.68 D. 17.32

28. A 2000 kW, 2400 volt, 75 percent power factor load is to be supplied from a
34500 V, 3 phase line through a single bank of transformers. Give the primary
and secondary voltages in volts of the transformer for wye-delta connection.
A. 20000/1380 C 34500/1380
B.34500/2400 , D. 20000/2400

29. A 500 hp, 2400 V, 3-phase motor of 90 percent power factor and 95 percent
efficiency is to be operated from a 13.2 kV utility service through a wye-delta
connected transformer. What is the minimum KVA rating of each transformers?
A. 125 C.500.
B. 250 D .167

30. In a Scott-connection, calculate the line current in Amperes on the 36 side if the
load on th e ^ side both draws 5000 A at 100 Volt and the 3<t> line voltage is
3,300 Volt, Neglect transformer loss.
A . 195 A C . 152 A
B . 175 A D . 140 A
11. A transformer rated 2000 kVA, 20,000/240 volts has 5.75% impedance. What is the
impedance referred to primary.
A. 0.0635 C. 0.0656 —X L
B. 11.5 • D. 34.2 '• '
t >
12. A 125 kVA distribution transformer with 4% impendence is connected in parallel with another
transformer rated 75 kVA and with 3% impedance: Both have same voltage ratio. (Neglect
resistance of each transformer) The total load is 140 kVA at 85% power factor. How much
load does each carry?
A. 80.5 kVA/59.5 kVA C. 80kV A /6 0 kV A
77.78 kV A /62.22 kVA D. 87.50 kVA/52.5 kVA

13. A 13,800/480 Volts, 10MVA three phase transformer has 5% impedance. What is the
impedance in ohm referred to the primary?
A. 0.952 ohms C. 5.125 ohms
B. 0.03 ohms D. 9.01 ohms

14. A 3(|> transformer is rated 30 MVA, 115A/13.8Y kV, and 2 % impedance, the resistance is
negligible compared with the reactance. Determine the percent (%) voltage drop at the
secondary at 80 % load and at 75 % power factor.
A. 4.1 % • C. 2.26%
B. 1.05% D. 3.15%

15. A power transformer rated 50,000 kVA, 34.5 kV/13.8 kV is connected wye-grounded at the
primary and delta on the secondary. Determine the full load phase currents on the secondary
side.
A 2092 amperes C, 1449 amperes
B. 1725 amperes (D?) 1208 amperes

16. A 1<j>transformer is rated 4,160 Volts primary and 250 Volts secondary. It has 1,500 turns in
the primary and a voltage regulation of 8%. What should be the number of turns in the
secondary windings ?
A. 88 turns !’v - ™ - o j QC) 97 turns
B. 90 turns ‘ ■ 4 D. 86 turns

17. A 3-phase transformer has a primary voltage rating of 34.5 kV, wye-connectedwith two-2.5 %
taps above and two-2.5 % taps below the rated voltage. The secondary winding is rated 460
Volts and is delta-connected. If it has 100 turns, what will be the effective number of turns in
the primary winding if the voltage tap is set at minus 2.5% ?
A. 7,133 turns T t „ a ; . f v C 7,312 turns
B. 4,222 turns D. 4,333 turns
E a
18. A 1$ 440/110 Volt transformer taKes a no-load current of 5 Amp at 0.25 p.f. lagging. If the
secondary supplies a current of 100 Amp at 0.8 p.f, lagging . Estimate the primary current.
A. 32 Amp C. 25 Amp
B. 22 Amp D. 29 Amp

19. A transformer has primary and secondary turns of 60 and 240 respectively.If the 60 turn
winding has a 5 % tap, calculate the primary voltage applied to 105 percentof thewinding
that will permit the rated output voltage of 480 Volts ?
A 504Volts» C. 456 Volts
B. 126 Volts D. 114 Volts

20. A 14), 100 kVA, 2400/240 Volt, 60 Hz transformer has a primary resistance and s e ^
winding resistance of 0.828 ohms and 0.009 ohm respectively. Calculate
resistance referred to primary. - binding
A. 1.6 ohms C. 1.728 ohms
B. 0.837 ohms D. 1.46 ohms

21. A 230/460 V transformer has a primary resistance of 0.2 ohm and reactance of 0-5 ohm and
the corresponding values for the secondary are 0.75 ohm and 1.8 ohm respectively. What is
the secondary terminal voltage when supplying 10 A at 460V, 0.80 p.f. lagging ?
A. 455.2 V C. 450.6 V
t£> 425.3 V E , , V , / , ..

22. A 3,300/230 V, 50 kVA, single phase transformer is found to have impedance of 4% and a
copper loss of 1.8% at full load. What is the percent reactance?
A. 3.74% / -— " "" C. 3.83%
B. 3 57% Zt ^ D. 3.79%

23. A single phase 60 Hz transformer has its core volume of 1.5x1 O’3 cu m. The maximum flux
density is 1.0 tesla and the hysteresis constant of the core is 750C^r find the hysteresis loss.
£ 755 Watts p C. 355 Watts
675 Watts ^ D. 255 Watts

24. In a transformer, the iron loss is found to be 42 W at 30 Hz and 90 W at 60 Hz measured at


the same flux density. What is the hysteresis and eddy current loss at 60 Hz?
A. 60 W & 30 W C. 3 0 W & 6 8 W
78 W & 12 W D. 1 2 W & 7 8 W

25. A 4,400 Volt, 60 Hz transformer has a core loss of 840 Watts, of which one-third is eddy-
current loss. Determine the core loss of this transformer when it is connected to a 4,600 Volts,
50 Hz.
A. 779 Watts C. 879 Watts
B. 679 Watts Q D ) 977 Watts

26. A 25 kVA, single phase transformer the core loss and full load copper loss are 320 and 380
watts respectively What is the efficiency at half load and 0.866 power factor?
A. 97.8% C 94.7% « , u . h
B. 95.3% (D ) 96.3% •* ‘ ^ * ^6
fV ^
27. What is the ratio of capacity of open delta bank to the capacity of close delta bank ?
A. 72% ( C)57.7%
B; 63.2% S i 66.7%

28. In a Scott-connection, calculate the values of the line currents in Amperes on the 3300 volts
3<j> side if the load on the 2<j>side are both 300 kW at 100 Volt and 70.7% lagging p.f.
A. 157.2 C. 145.2
B 148.5 D. 85.7

29 A 1<j>, 5 kVA transformer has 35 Watts, iron loss and 40 Watts copper loss at full-load. It
operates at full-load, 0.80 p.f. iagging for 6 hours, half-load ,0.75 p.f. lagging for 12 hours and
quarter-load at unity pf for 6 hours, its all-day efficiency in percent is nearest t o ......
A. 98.9% C ) 97.8%
B. 95.2% IT 96.5%

30 A 120/180 Volt auto-transformer draws power at 120 Volt and supplies power to a 2 kW load
at 180 Volts with a p.f. of 0.80 lagging. An additional load of 1 kW is supplied at unity p.f. from
the 60 Voit Winding. What is the current drawn from the 120 Volt supply line ?
A 26 C. 28
B 30 D. 32
31. Three—1<)> transformers, rated at 50 kVA, 2,400/2,400 Volts (1:1ratio) each, are to be
connected as autotransformer to deliver 4800 V from a 2400 V line. What is the maximum kVA
that can be delivered by these auto-transformers if they are connected in A ?
A >00kV A C. 275 kVA
B. 250 kVA D. 150 kVA

32. Three (3)-1<j), 75 kVA transformers in delta-delta bank supplying a balanced load 160 kVA
(3<(>) at 0.80 p.f. lagging. If one (1) transformer is removed for repair. The V-V bank was
overloaded, then to prevent overloading of the remaining units, capacitor bank is connected
across the 3s|> load. What is the minimum capacitive kVAR required ?
A. 118.14 C. 73.86
B. 86.73 D. 114.18

33. A 500 kvA, 13200/2400 volts 60 Hz single phase transformer has a 4% reactance and 1%
resistance. The core loss under rated condition is 1800 watts. Calculate the percent
efficiency of this transformer at full load and 85% power factor. 2 , « ^ . , ■„/
A .98.4 % I t - H. C. 96.8 ^ ^
B. 96.2 /d D. 97.6 ’
IH'»V
34. A power transformer rated 50,000 kVA, 34.5/13.8 kV is connected wye-wye, "What are the line
currents in Amp at full-load ?
A. 525.2 & 1725 CJ836.7 & 2092 .
B. 483.1 & 1207.7 '"TT 1449.3 & 3623.2

35. A 750 Hp, 2400 volts, 3-phase motor 90% power factor and 95% efficiency is to be operated
from 13.2 kV utility service through a wye-delta connected transformer bank. What is the total
the kVA bank rating of the transformer?
2 )7 5 0 C. 595
B. 575 D .615

36. A 750 Hp, 2400V, 3-phase motor of 90 percent factor and 95 percent efficiency is to be
operated from a 13.2 kV utility service through a wye-delta connected transformers. Compute
the secondary or load current in ampere of the transformer.
A. 180.6 C ll5 7 .3
B. 125.6 0 . 175.4

37. The current at the common winding of a 20kV/7.97kV auto-transformer is 200A. What is the
current at the secondary?
200A C. 142.22A
J/332.45AL D. 132.45A

38. Name the transformer that provides an almost continuously variable stepless output from zero
rated line voltage.
A. Booster \C )Variac
B. AVR D. UPS

39. A 1<j> -250 kVA, 7967/240 Volt, 60 Hz transformer has 1.5% resistance. What is the full-load
copper loss? .
A. 626.25 watts ic ? 3750 watts
B. 1252.5 watts D. 2505 watts

40. Tertiary winding is provided in transformers thatteve


A. delta/delta winding t<c ) wye/wye winding
B. delta/wye winding D. wye-grounded/delta winding
41. The chemical used in a breather for transformer should have the quality of
A. ionizing air C.jabsorbing moisture
B. cleansing the transformer oil D7 cooling the transformer oil

42 The chemical used in breather is


A. asbestos fiber C silica sand
B. sodium chloride D. silica gel

43. The following are protection device built-in for safe operation of a power transformer,
EXCEPT:
A. breather C. conservator
B. bushing insulator D. pothead

44. Which of the following is NOT TRUE about “Buchholz Relay” ?


A.) it can detect faults above oil level in transformers
B it is installed between main tank and conservator
C. it is a gas actuated relay
D. it can detect faults below oil level in transformers

45. A common method of cooling a power transformer is


A. natural air cooling C. air blast cooling
B. Oil cooling ( r D. any of the above

46. Calculate the percent voltage regulation for a transformer with apercent resistance of 2 % and
a reactance of 4 % of rating 550 kVA. when delivering 450 kVA at 0.6p.f. lagging.
A. 5.5% C. 4.4%
B. 4.2% ( IT ) 3.6%

47. A quantity is expressed in per unit if it is divided by a chosen base quantity (having the same
physical dimension). Suppose that for a 10-kVA, 2400-V/240-V transformer we choose
P hase “ 10 kW Vj = 2400 V Va>tor = 2 4 0 V
This transformer has the following test data:
Open-circuit test (low-voltage side): 240 V, 0.8 A, 80 W
Short-circuit test (high-voltage side): 80 V, 5.1 A, 220 W
What is the series resistance in per unit?
A. 0.0148 pu C. 0.0184 pu
B. 0.0114 pu D. 0.0168 pu

48. A 5 kVA 60 Hz single phase transformer has a full load efficiency of 96% and an iron loss of
40 watts under rated condition. If the transformer is connected as an autotransformer
operating from a 240 volts mains and delivering 5 kW at unity power factor to a 120 volts
circuit, calculate the percent efficiency.
A 95.6 C. 98.7
B. 95.8 D. 98.4

49. Three single phase 37.5 kVA transformers are connected in delta. Load is 85 kVA at 85%
Jagging power factor. What is the overload kVA of each unit if one unit is brought for repair?
& ^ ... ^ ^ ^ C. 11.6
B. 27.5 D. 10

50. Three transformers are delta connected, one connection has been cut, what is the percent
reduced?
A. 33% C. 67%
51. The potential transformer of a line have a ratio of 132,000/66. These are connected wye-delta.
The voltmeter connected at the secondary reads 64 Volts. What is the voltage of the line ?
A. 221.7 kV C. 135.2 kV
B. 112.5 kV D. 132 kV

52. A 2200/220 volts transformer has two 110-volt coils connected in series Across the first of
these two coils is connected a resistive load drawing 15 A. Across the second coil is
connected a pure capacitor drawing 10 amperes. Across the two 110 volt coil in series is an
80% lagging power factor circuit drawing 20 A. Calculate the current to the 2200 volts coil.
A. 4 50 C. 8.45
B. 3.45 D. 2.45

53. A 500 kVA, 13200/2400 volts 60 Hz single phase transformer has a 4% reactance and 1%
resistance. The leakage reactance and resistance of the low voltage winding are 0.25 and
0.055 ohm, resp. Calculate the leakage reactance and resistance in ohms of the HV winding.
A. 6.38 & 1.82 D fa , n . ■ , J. C. 6.53 & 1.87 u . i
B. 4.86 & 1.56 ' f ' D. 6.23 & 1.85

54. Two (2) 1<|> transformers having equal turns, ratio and ratings are operated in parallel to
supply a load of 280 kW at 80 % p.f. lagging- Transformer A has 2 % resistance and 8 %
reactance, while transformer B has 1% resistance and 6 % reactance. What is the power
delivered by each of the transformers to the load ?
A. 123 and 157 kW C. 130 and 150 kW
B. 142 and 138 kW D. 127 and 153 kW

55. What is the maximum flux in Weber for a transformer with 200 turns at the primary when
supplied by 60 hertz, 200 volts source?
A. 0.0037 C. 0.375
B. 0 0375 D. 3.75

56. A transformer rated 1-kVA, 240/120, 60 Hz. Because of an emergency, the transformer has to
be used on a 50 Hz system If the flux density in the transformer core is to be kept the same
at 60 Hz and 240 volts, what is the volt- ampere rating at 50 Hz?
A. 0.833 kva C. 0.766 kva
B. 1.211 kva D. 0.899 kva

57. A transformer has a turn ratio of 5. If a 150 ohms resistor is connected across the primary,
what is the resistance referred to secondary?
A 2 ohms C. 6 ohms
B. 4 ohms D. 5 ohms

58. An ideal transformer has a voltage rating of 2400/240 volts, a motor is connected to the
secondary draws 50 A at 200 volts, what resistance in ohms must be connected to secondary
to maintain this voltage at secondary?
A. 6 C. 5
B. 80 D. 0.8

59. A 100 kVA two winding transformer, 2400:240 volts 3% impedance is converted as auto­
transformer 2040:2400 volts. What is the new percent impedance?
A. 0.27% C. 3%.
B. 2.7% D. 0.3%

60. What is the normal secondary current rating of current transformer in amperes?
A. 20 C. 5
B. 10 D.15
PRACTICE Makes Perfect!!!
AC MOTORS
This Free Quality Manual is
Found only at POWERLINE

POWERLINE REVIEW CENTER


MEZZANINE FLOOR, DO N A A M P A R O BUILDING
C O R N E R E S P A N A & G. M. TOLENTINO STREETS
S A M P A L O C , MANILA
TEL. N O S . 7 3 5 - 7 3 - 0 2 & 7 3 3 - 2 1 - 1 8

TEL. N O S . ( 0 3 ) 2 6 1 - 2 2 4 4 & ( 0 3 2 ) 2 6 1 - 8 4 5 2
POWERLINE REVIEW CENTER, INC.
AC MOTOR REVIEW LECTURE
1. IN D U C T IO N M O T O R S

An induction motor is one in which the magnetic field in the rotor is induced b;
currents flowing in the stator winding. The rotor has no connection whatsoever to the line. 1
differs from the synchronous motor, the rotor o f the induction motor does not rotate as fast a
the rotating field in the annature. The armature difference in speed is called the slip. The sir
is usually expressed as a percentage o f synchronous speed.

% Slip = — ?.~N r „tl 00%


Ns

Slip is alsc the measure o f the.rotor winding losses J .c.


„/OI. RCL
% S h p ------x l0 0 %
RPI

Slip is also connected with rotor efficiency, the higher the slip* the lower the clhciency, i,e.

% ?/= — *100%

T Y P E S 01 1 H I U 1 -l‘ll 1M ) L ( 1 IO N .M O T O R S :

1. S q u irre l -cage tvpe - the rotor winding is practically self-contained and its not connect!
either mechanically o f electrically with the outside power supply or control circuit,
consists o f number, o f straight bar uniformly distributed around the periphery ot the rot
and skewed and short circuited, at the both ends by end ring to which'they are integral
joined

Sneed Characteristics : nearly constant speed


Torque Characteristics : starting torque depends on the resistance oi the rot
winding
Applications : constant speed service as in fans, blowers, compressors, pumps, el

2- W o un d -rotor tvpe - This is a three-phase motor that has another three-phase winding
instead o f a squirrel -cage rotor, the terminals o f which are connected to three slip-rings.
Brushers rides these slip rings and deliver the current-to an external three-phase rheostat
variable resistor to vary the characteristics of the motor. At start, all the resistances are in
the circuit, as the motor picks-tip speed. The resistance are gradually decrease until fim i
slip rings are short circuited

Speed Characteristics : Variable speed


Torque Characteristics : by inserting value ol resistance in the rotor circuit
variety o f torque values (high or low) are obtainable.
* Applications : Where high starting torque with low starting current or where
limited speed control is required , as in fans centrifugal and plunger pumps,
compressors conveyors, hoist and cranes, etc.

IN D U C T IO N M O T O R E Q U I Y A I F N T C I R C U I T D IA G R A M

Per phase basis ; Rt Xt K i- t X * - i


-vwv—
I© h -l

Go* \S c RlA o d ^^7 £ ~ jR trt

’Where
Vo = Applied voltage per phase 12 Rotor current per phase
Ek = Rotor induced cm f per phase 1[ Stator current per phase
E 2 = Stand still rotor induced em f per phase lo = no load current per phase
R = Rotor reactance per phase S = Shp
X : = standstill reactance per phase f - supply or stator frequency
X ; = stator reactance per phase fi( “ rotor frequency
R ; = rotor resistance per phase Go no load conductance per phase
R. = stator resistance per phase f>s, no load susccptance per phase

INDUCTION MOTOR FUNDAMENTAL FORMULAS

1. Synchronous Speed, Ns
120/
2. Slip Speed n s - n r

3. Percent Slip S = — — ^-R-x\00%


Ns

4. Rotor Frequency, fR = «(/)

5. Rotor Reactance X R = S ( X 2)

6. F^otor Standstill Induced E M F E 2=a ( V J if a (1 .0

E, = if a ) 1.0
a

7. Rotor Induced E M F per Phase 5 (E ; )

■S
8. Rotor Load Resistance per Phase R LOad ~ R-
S
POWER FORMULAS

1. Rotor Pow er Input RPI = watts per phase

2. Rotor Copper Loss R C L - / ,' [/?, ] watts per phase


'1-5'
3. Rotor Pow er Developed RPD = I 2 R2 watts per phase

-U V \e&
A M

1C. ®PL \
P O W E R S T A G E S IN A N IN D U C T IO N M O T O R :

RPD ^> «PO


SPl ROTOR POWER
ROTOR POWER OUTPUT
ROTOR ~FOWE&\ DEVELOPED
STATCR POWER INPUT
INPUT

v
^ ll V
SSL
S C L+S 1L (M och'l, Loss)
( S tatorC u a Iron J g f"’ Cu iOSS>

SPI = RPI + SC.L + SIL


RPI - RCL + RPD
RPD - RPO + M L

A D V A N T A G E S O F S Y N C H R O N O U S M O T O R O V E R 1\ 1)1 ( H O N M O T O R :

They can be made to operate at leading power factor and they improve the power factor of
an industrial plant from one that is normally lagging to that is close unity.
They are less costly in certain horsepower and speed range.
They can be constructed with wider air-gap induction motor, which makes them better
mechanically.
They usually operate at higher efficiency, especially at low i,peed unity power factor range.

A D V A N T A G E S O F IN D U C T IO N M O T O R O V E R S Y N C H R O N O U S iM O T O R :

They are and rugged hence easier to maintain and operate


Their speed can be easily adjusted or varied oveiva wide range.
They are cheaper to first cost.
.M ETHODS E.MPLOYI-D TO START 3-PIIASE M O T O R S :

. Across the line starting - this customarily used when the motor must operate at a high
■tailing torque. It is to be used in connection with a time delay over-current relay which w ill
>ecoma operative before the winding can be damage by the surge current.

I. Reduced voltage starting - this is used in order to .-educed the staring in rush current which
nay have detrimental effects on the line voltage and may disturb other apparatus.

A. Line resistance starting - it uses suitable high current, low ohmic - value resistance
is Removed in steps or all at once.

B. Line reactance starting - it use suitable iron-core reactance in replaced o f the


resistance m (a) to a a >mpiish the result

C. Auto-rrans former starting - is uses tapped auto-transformers in open delta


connection to reduced the motor voltage.

D. Wye-deltu start inn - it used when the motor is designed for delta operation at its
rated voltage. The motor phase winding are connected by contractor for a wye
circuit starting. As a result, each phase w ill take the normal line current
divided by 3.

3. P art winding method - This method is particularly adapted to Dual-Voltage motors (those
designed to operate at two Voltages). The motor winding is divided into two identical wye-
cdnnected sections. One o f which is used when is used when the moior is started and after its
accel crates to about rated speed, the two section are joined in parallel for normal operation.

L A B O R A T O R Y T E S T F O R IN D U C T IO N M O T O R :

A. No -load Test:
W hen supplied with rated voltage and frequency, voltage, current and power input
arc measured. The no-load power input represent core loss, friction & windage , small
stator copper loss and rotor copper loss (which is almost negligible).

Pin = P nl - core loss = friction & windage + small copper loss

B. Blocked - Rotor Test:


W hen reduced voltage is applied, hold the rotor firmly so that cannot rotate,
voltage current and input power are measured,' Ammeter reading represents the block rotor
input current (short circuit current). The power input represent “ stator and rotor copper loss
and small core loss” (almost negligible).

Pin = P0R = Stator Copper Loss + Rotor Copper Loss + Small CoreLoss
DOUBLE SQ U IRREL CAGE INDUCTION M OTOR:

The main disadvantages o f squirrel-cage IM is its poor because o f low rotor resistance
The starting torque could be increased by having cage o f high resistance, but the motor \vm
have poor efficiency under running conditions! because there w ill be more copper losses;
This can be accomplished by introducing two rotor cages. The outer cage has a high-
resistance har low inductance and being close to the rotor, while surface, while the inner-
cage is low-resistance bars and has high inductance and being nearly surrounded by ion.

C A S C A D E D / C O N C A T E N A T E D 'O R IA N D E M O P E R A T IO N :

in this method, two induction motors arc used and are ordinarily mounted on the same
haft so that both run the same speed (or else they may be geared together). The stator winding
of motor A ( a wound rotor IM) is connected to the main source, and its rotor winding is the
joined to the stator winding of motor B ( a squirrel IM or wound rotor IM)

II - SY N C H R O N O U S M O T O R S

An ac motor that can run only at definite constant speed called " synchronous speed
regardless o f the load applied to it.

Some Characteristics Features of Synchronous M otor are:

1. It runs either at synchronous speed or not ail, i, e. while running it maintains a constant
speed. The only w ay to change is speed is to vary the supply frequency

2. It is inherently not self-starting. It has to be run up to synchronous ( or near


synchronous) speed by some means before it can synchronized to the supply, so in
practiced they are built with "damper or ainonissenr winding" this damper bars on
the rotor come into play during transient or starting. It also prevent hunting swinging
o f the rotor with varying loads.

3. It is also capable o f being operated over a wide range o f power factor, lagging or
leading. Hence, it can be used for power factor correction purposes , in addition to
supply torque to drive mechanical loads.
TWO BASK PAR I S OESYN CH RO N O U S MOTORS:

A. Field (ro to r > tru ch ire -'carrying.dc-excited winding?

B. A rm atu re (stator) structure often has 3- phase winding which is connected to an A C source

T O R Q U E A S S O C IA T E D W I T H S Y N C H R O N O U S M O T O R :

1. Starting T o rq u e - Torque developed when voltage is applied in stator.

2. Pu ll - in torque - amount of torque or load at which the motor w ill pull into step, when it
is started as an induction motor ( 2 to 5 % ) below synchronous speed until it is synchronous
to the system.

3. Running T o rq u e - torque developed by the motor while running

4. Pull-out Torque - maximum torque at which the motor w ill develop nulling out-of- step
or out o f synchronism, with an increase in torque produces backward shift “ a ” , too much
shift o f torque angle o f the rotor ( at about Zz distance between adjacent pole) w ill cause
motor to stop.

A P P L I C A T IO N O F S Y N C H R O N O U S M O T O R S :

A ) Pow er factor Correction :


When synchronous motors are employed as a p.f. correction device, it is a called
" synchronous condensers” , this can be done by running the synchronous, motor without
mechanical load with field over-excited.

B ) Constant speed load drives:


When constant speed is required such as drives for centrifugal pump, dc generator, belt
.driven reciprocating compressor, fans, blowers etc.

C ) Voltage Regulators:
At the end o f a transmission line, the voltage is controlled.

M E T H O D S / P R O C E D U R E S F O R S T A R T IN G S Y N C H R O N O U S M O T O R :
1. First, main field winding is short-circuited.
2. ’Reduced voltages, with the help o f auto-transformers, were applied across stator terminals,
the motor starts up,
3. When it reaches a steady (as judged by its sound), a weak excitation is applied by removing
the short-circuij on the main field winding. I f excitation is sufficient, then the machine will
be pulled into synchronism.
4 Full supply voltage is applied across the armature (stator) terminals by cutting out the auto­
transformers.
r The motor may be operated at any desired power factor by changing the dc excitation.
SINGLE-PHASE AC M O T O R :

(a) S I N G L E P H A S E IN D U C T IO N M O T O R - it is similar to 3<j>IM , but it is N O ®


self-starting. Jj

(1) Split-phase motor it has stator windings, which is the main and
- auxiliary windings.
a. Standard split-nlnisc motor in series with a the auxiliary
winding is a centrifugal switch that w ill disconnect this wmdinjB
'from the source once the motor has reached -75% to 80% o f u |
synchronous speed.

U S E S : grinder, blower, etc.

b. C apacitor start motor - similar to the standard split phase


motor, only it has a capacitor connected in series with the
auxiliary winding to make the starting torque higher.

U S E S : refrigerator, air conditioning unit, pump, etc.

c. C apacitor start and run motor it has two capacitors, one for
starting another for running.
U S E S : same as capacitor start motor

2. Shaded pole motor - it uses shading coils instead o f auxiliary winding.


Uses: small fans.

(b; ( '( I.MM1 I V I O R I V IM - its rotor is the dc armature

1. U niversal M o tor - it is small series wound motor which can be operated using
ac or dc source at approximately same output power.
U S E S ; sewing-machine, portable tools, etc.

2. Repulsion its brushes are short circuited making the armature as a large
magnet.
U S E S : vacuum cleaner, floor polisher, etc.

3. Repulsion Start-induction rug motor - similar to the repulsion motor, only


the commutator is automatically short circuited once the motor has reached 80%
o f its synchronous speed.
U S E S : oil-bumer pump, blower, etc.

4. Repulsion induction motor - similar to the repulsion motor only it has squirrel
cage winding placed on the rotor. This-motor start as induction motor and
repulsion motor at the same time.
U S E S : refrigerator, compressor, gasoline pump, etc.

(C) U N E X C I T E D l<p S Y N C H R O N O U S M O T O R it runs at a constant speed and it


is self-starting.
These are two types:
1, Reluctance M o to r - it has a squirrcl-eage rotor o f unsymmetricai magnetic
construction which is achieved by removing some teeth o f the symmetrical rotor
puncnmg.
U S E S : timers, recording instrument, photographs, etc.

OC
2. Hysteresis motor - it operates on the presence o f a continuously- revolvin
magnetic flux, usually shaded-pole principle is employed, the only difference it
has smooth chrome-steel cylindrical rotor having high retentivity so that
hysteresis loss is high.
U S E S : driving-tape deck, turn-tables, clocks, etc.

M E T H O D S E M P L O Y E D T O S T A R T S IN <. I I PH A S K M O T O R S :

1. Splii-phase method (capacitor start is the most popular)


2. Shaded-pole method
3. Repulsion method
4. Repulsion-induction method

Methods of Reversing the Direction of Rotation:

S tandard Split Phase M otor } You reverse the connection-of the


C apacitor Sta rt M o tor j main or the auxiliary winding but
Capacitor Run M o to r j- not both.
Ci.pacitor Sta rt Run M otor
\

Repulsion induction M otor you move I he brushes m the rotation ol the original rotation:
until the direction o f rotation is reversed.

Universal M o tor - you interchanged the brushes.


POWERLINE REVIEW CENTER Inc.
H e Ultimate E.E. Review Center
/ powERLIWE X
/VO MOTORREVIEW PROBLEMS

1. A 3-phase, 60-Hz induction motor has 8 poles and operates with a slip of 0.05
for a certain load. Compute the speed of the rotor field with respect to the
stator field.
A. 900 rpm 855 rpm
B. 45 rpm D' 0 rpm

2. A 3-phase, 6-pole induction motor runs on no-load at 1160 rpm. Determine the
slip and frequency of rotor currents on no-load.
A. 0.034 & 2Hz C. 0.09 & 5.4Hz
B. 0.04 & 2.5Hz D. 0.05 & 6Hz

3. An induction motor runs at 3510 rpm. Calculate the percent slip.


A. 5% C. 3%
B. 4% D. 2.5%

4. A 4-pole induction motor, running with 5% slip, is supplied by a 6 pole


synchronous generator driven at 1200 rpm. What is the motor speed?
A. 1200 rpm C. 1710 rpm
B. 950 rpm D. 1128 rpm

5. An induction motor are often braked rapidly by a technique known as


“plugging”, which is the reversal of the phase sequence of the voltage
supplying the motor. With 4 poles is operating at 1750 rpm from an available
infinite bus at 60 Hz. Two of the supply leads are suddenly interchanged. What
is the new slip and rotor current frequency?
A, 1.97 & 118.33 Hz C. 0.027 & 1.62 HZ
B. 0.97 & 58.2 Hz D. 0.078 & 4.68 Hz

6. An 3-phase induction motor has rotor resistance and standstill reactance of 0.1
ohm and 3.5 ohm per phase respectively. The rotor emf at standstill is 60 volts
per phase , for a slip of 5 %, what is the rotor power input?
A^1328W C. 1156W
B. 1262W D. 1068W

7. Induction motor has resistance per phase of 0.1 ohm, phase current of 10A
and running with 5% slip. Find the power developed per phase.
A. 200 W C . 240 W
J 190 W D. 150 W

8. A 20 hp, 3-phase, 400-V, 60-Hz, 4-pole induction motor delivers full-load at 5: :


slip. The mechanical rotational losses are 400W. What is the rotor copper loss7
Nt 806.3 W C. 746 W
B. 860.3 W D. 803.6 W
9. A 3-phase, 6-pole induction motor is rated 400 Hz, 150 V, 10 hp, 3% slip at
rated power output. The windage and friction loss is 200 W at rated speed
With the motor operating at rated voltage, frequency slip, and power output,
determine the power crossing the air gap.
A . 7897 W C .7486 W
B .7863 W D .8036 W

10. A 15 Hp, 4 pole 60 Hz, wye connected, 208 volts induction motor draw 12 kW
from the line have losses as follows: Stator copper loss 900 W, Rotor copper
loss 300W, Iron loss 800W and Friction & Windage Loss 100W. Determine the
speed of the motor in rpm.
A. 1710 C.1700
B . 1720 D . 1747

11. A 3-phase, 6-pole induction motor is rated 400 Hz, 150 V, 10 hp, 3% slip at
rated power output. The windage and friction loss is 200 W at rated speed.
With the motor operating at rated voltage, frequency slip, and power output,
determine the output torque.
A. 9.2 N-m C. 8.2 N-m
B. 10.4 N-m D. 7,2 N-m

12. The rotor of a 3-phase, 60-Hz, 4-pole induction motor takes 120 kW at 3 Hz.
The motor has stator copper loss of 3 kW, a mechanical loss of 2 kW and a
stator core loss of 1.7 kW. Determine the motor efficiency.
A. 89.7% C. 90.2 %
B. 89.5% D. 85.7 %

13. A 20 hp, 3-phase, 400-V, 60-Hz, 4-pole induction motor delivers full-load at 5%
slip. The mechanical rotational losses are 400 W. Calculate the
electromagnetic torque.
A. 83.3 N-m C.- 85.5 N-m
B. 84.6 N-m D. 87.6 N-m

14. A 50 hp, 3-phase, 4-pole, 60hz, 230 volts induction motor is operating at full
load slip of 3%, what is the full load torque in Ib-ft?
A. 125 C. 170
B . 150 D. 195

15. A 4 pole, 60 Hz., 200 volts single phase induction motor having a full load
current of 350 amperes. The torque developed is 700 N-m. What is the
approximate speed of motor?
A.)955 rpm C. 1196 rpm
B.' 1704 rpm D. 1800 rpm

16. An eight pole, 60 Hz SCIM is deliberately loaded to a point where pull-out or


stalling will occur. The rotor resistance pr phase is 0.3 and motor stalls at 650
rpm. Determine the standstill reactance or locked rotor reactance.
A' 1.08 ohms C. 2.18 ohms
B. 5.08 ohms D. 4.86 ohms
9. A 3-phase, 6-pole induction motor is rated 400 Hz, 150 V, 10 hp, 3% slip at
rated power output. The windage and friction loss is 200 W at rated speed.
With the motor operating at rated voltage, frequency slip, and power output,
determine the power crossing the air gap.
A. 7897 W C .7486 W
B .7863 W D .8036 W

10. A 15 Hp, 4 pole 60 Hz, wye connected, 208 volts induction motor draw 12 kW
from the line have losses as follows: Stator copper loss 900 W, Rotor copper
loss 300W, Iron loss 800W and Friction & Windage Loss 100W. Determine the
speed of the motor in rpm.
A. 1710 C.1700
B . 1720 D . 1747

11. A 3-phase, 6-pole induction motor is rated 400 Hz, 150 V, 10 hp, 3% slip at
rated power output. The windage and friction loss is 200 W at rated speed.
With the motor operating at rated voltage, frequency slip, and power output,
determine the output torque.
A. 9.2 N-m C. 8.2 N-m
B. 10.4 N-m D. 7,2 N-m

12. The rotor of a 3-phase, 60-Hz, 4-pole induction motor takes 120 kW at 3 Hz.
The motor has stator copper loss of 3 kW, a mechanical loss of 2 kW and a
stator core loss of 1.7 kW. Determine the motor efficiency.
A. 89.7% C. 90.2%
B. 89.5% D. 85.7 %

13. A 20 hp, 3-phase, 400-V, 60-Hz, 4-pole induction motor delivers full-load at 5%
slip. The mechanical rotational losses are 400 W. Calculate the
electromagnetic torque.
A. 83.3 N-m C; 85.5 N-m
B. 84.6 N-m D. 87.6 N-m

14. A 50 hp, 3-phase, 4-pole, 60hz, 230 volts induction motor is operating at full
load slip of 3%, what is the full load torque in Ib-ft?
A, 125 C. 170
B.150 D. 195

15. A 4 pole, 60 Hz., 200 volts single phase induction motor having a full load
current of 350 amperes. The torque developed is 700 N-m. What is the
approximate speed of motor?
A. 955 rpm C. 1196 rpm
B. 1704 rpm D. 1800 rpm

16. An eight pole, 60 Hz SCIM is deliberately loaded to a point where pull-out or


stalling will occur. The rotor resistance pr phase is 0.3 and motor stalls at 650
rpm. Determine the standstill reactance or locked rotor reactance.
A 1.08 ohms C. 2.18 ohms
B. 5.08 ohms D. 4.86 ohms
17. A 10 hp, 550 Volts, 60 Hz, 87% efficiency and 75% power factor 3-phase
induction motor has starting current of 5 times the full load current. If the motor
is used in a 440 V at 60 Hz system. What is the starting current ?
A. 60 Amp. . C: 48 Amp.
B. 41.5 Amp. D 52.4 Amp.

18.An induction motor has starting torque of 100 N-m at full-voltage starting. If
the starting voltage is reduced to 80% of rated voltage, what is the starting
torque?
A. 150 N-m / C. $ 4 N-m
B. 50 N-m 80 N-m /

19. For a-given voltage supplied to the primary of a wye-delta started and the
secondary changed from wye to delta. Find the ratio of startingphasecurrents
(/,. / / A ).

A) 1/ a/3 C. 1/3 V'


B. V3 D. 3

20.A delta connected 6-leads 20 hp, 3-phase 60 Hz, 230 volts 1750 rpm, 54
amperes squirrel cage induction motor at standstill draws 5 times full load
current with normal applied voltage and develops 1:5 times full load torque.
What is the starting current in each phase when wye connected?
A. 160 Amp x - G .-270-Amp
B. 110 A m p * D> 90 Amp

21.A 15 hp, 230 Volt, 3-phase, 42 Amp, 1750 rpm squirrel cage induction has
starting torque of 2.5 times that of full-load and starting current of 6 times the
full-load current when started at full voltage. The motor is equipped with an
autotransformer or starting compensator. If the auto-transformer is set at 65 %
tap, What is the starting current?
A. 252 Amp (L)106.5Am p v'
B. 126 Amp Vs D. 163 Amp

22. A 20 HP, 230 volts, 1750 rpm, 3-phase, 60 Hz induction motor draws 5.5 times
the full load current and develops 1.6 times the full load torque when starting
under the rated voltage. What is the starting voltage to produce a rated
torque? ,
A. 144Volts r. C . 1 8 2 Volts *•
B. 230 Volts f . Vj v.c V D. 165 volts

23. A 50 HP, 440 volts, 3-phase induction motor delivers during the starting period
of 150% of the normal torque and draws 550% of rated current with the rated
voltage. The full load pf and full efficiency are 80% and 90% respectively. If the
starting torque of the load is only 50% of the rated torque of the motor and an
auto transformer is used as starting unit, what should be the starting current at
this voltage?
A ^1 2 5 amp. C. 374 amp.
B. 130 amp. £ D. 254 amp. *
24. An induction motor is to be started at a reduced voltage such that the starting
current will not exceed four times the full-load current, while developing the
starting torque of 25% of the full-load torque. The full-load slip is 3%.
Determine the factor by which the motor terminal voltage must be reduced at
starting.
A) 0.722 C. 0.655
B. 0.825 D. 0.688

25. The synchronous speedof an induction motor is 900 rpm. Under abiocked-
rotor condition, the input power to themotor is 45 kW at 193.6 A. The stator
resistance per phase is 0.2 Q and the transformation ratio is a = 2. Calculate
the ohmic value of the rotor resistance per phase.
A. 0.04 ohm C. 0.02 ohm
B. 0.01 ohm D. 0.05 ohm
V J " '■

26. A 6 pole, 60 Hz, three-phase induction motor operates with slip 2 % to requires
11 Amp and 3500 Watts when driving its rated load. At no-load this requires
4.3 Amp and 290 Watts, when the rotor is blocked 440 Watts and 52 Volts are
required to circulate a current of 14 Amp. Calculate the rated efficiency?
A. 83.15% C. 85.13% •
B. 81.53% D. 88.53 %

27. A 6 pole synchronous motor has a speed of 1145 rpm at certain instant, what is
the supply frequency?
A.'57.25 Hz C. 59.45 Hz
B. 58.25 Hz D. ' 60 Hz

28. A 380 V, 3-phase star connected synchronous motor has an effective armature
resistance of 0.25 ohm and synchronous reactance of 4 ohms per phase. At
certain load, with over excited field it draws 20 ampere current at 90% power
factor. What is the excitation emf?
A. 592 V ( c . 451 V '
B .477 V D. 350V

29. A 2300 volts, three phase, wound rotor synchronous motor has Xs= 2 ohms
per phase and Ra= 0 1 per phase. The motor operates at 0.866 leading power
factor lagging while taking a line current of 350 amperes. Find the power
angle?
A. -20.7° > C. -22.9° *
B. 22.1° D. 19.5°

30. A 100 hp 600 volts, 1200 rpm, 92% rated load efficiency, 3-phase wye
connected synchronous motor has an armature resistance of 0.052 ohm per
phase and a leakage reactance of 0.42 ohm per phase. At rated load and 0.8
p.f. leading, determine the mechanical power developed within the armature .
JU79.6 kW C. 86.8 kW
B. 108.6 kW v D. 69.8 kW
AC Motors Supplementary Problems
1. A 3-phase 6-pole induction motor has a nameplate speed of 1160 rpm, What is its full load
slip?
A. 2.58% - C. 3.3% '
B. 3.59% "0 :1 .4 %

2. A 3-phase induction motor is wound for 4 poles and is supplied from a 50 Hz system . What is
the rotor frequency when rotor runs at 1410 rpm?
A. 3 Hz C. 2.5 Hz
B 2.9 Hz D. 3.5 Hz

3. A 3-phase induction motor is rated 50 Hp, 220 volts and 1745 rpm and 78% power factor. What
is the percent slip?
A. 2.5% C,'3.1%
B. 3.5% D. 4%

4 A 3-phase 440 volts, 50 Hz induction motor has 4% slip. What is the frequency of the rotor
induced emf?
A. 20 Hz C. 50 Hz
B 2 Hz D. 0.2 Hz

5. What is the speed of an induction motor of 6 poles if the percent slip is 2.5% ?
A. 1462 rpm C. 1170 rpm
B. 877 rpm 1755 rpm

6. The nameplate speed of a 60 Hz induction motor is 1164 rpm. If the speed at no load is 1188
rpm , what is the percent speed regulation?
A. 2.06 C. 2.86
B. 2.77 D. 4.24

7. A 4 pole, 60 Hz, 3-phase induction motor has a full load speed of 1750 rpm. What is the speed
at half load in rpm?
A. 1770 ; 0^3500
B. 1775 W875

8. A 20 HP, 3 phase 400 Volt, 60 H? , 4 pole induction motor delivers full -load at 5% slip. The
mechanical rotational losses are 400 Watts. Calculate the electromagnetic torque.
A. 58.5 N-m C. 53.3 N-m
B. 85 5 N-m D. 95.5 N-m

9. A 3-phase 150 hp, 60 Hz, 460 volts, 6 pole induction motor has a full load torque of 679 Ib-ft.
What is the full load slip?
A. 1.5% . . Kr T c -3%
B. 2.8% „• i-i 1 D' 4o/o

10. A 4-pole, 60 Hz , 3- phase induction motor has a blocked rotor reactance of 0.4 ohm per
phase and the rotor resistance is 0.1 ohm per phase. At what speed will the motor develop
maximum torque?
A 1200 rpm 1350 rpm
B. 1500 rpm D. 1620 rpm

11 If the maximum torque of an induction motor is 200 kg-m at slip of 12% and the torque at 6%
slip would be____ kg-m
A. )00 C . 150
BT 160 D. 140

12. A squirrel cage motor is started at 50% its rated voltage. What is the starting torque relative to
its rated voltage starting torque.
A. 50% C. 25%
B. 100% "T). 75%
13. A three phase 230 volts, 77% power factor, 75 hp induction motor draws a starting current 5
times of full load current. What is the starting current?
A. 1125 amperes C ... 250 amperes
B, 360 amperes D. ,912 amperes

14. A 10 hp, 550 V, 60 Hz, 3-phase induction motor has starting current of 425% of full load
current. If the motor is used in 440 V lines at 60 Hz system. What is starting current expressed
in percent of full load value?
A. 425% p. 400%
B. 360% D 340%

15. Magnetic contactor required for wye-delta starting of motors


A. 2 v ft 3
B. 1 D.4

16. For wound rotor motor, where is the reduced voltage starting device applied
A. rotor . C) Stator
B. Panel board v[5. field

17. Out of the following methods of starting a 3-phase induction motor, which one requires six
stator terminals?
A. direct-on-line Q. rotor rheostat
B. auto-transformer D.', star-delta

18. A 25 Hp, 230 Volts, three-phase motor with 85 % power factor has a starting current of 5.5
times rated current. To reduce the starting current, a wye-delta starter is installed. What is the
new starting current ?
M , 195 Amp C 135 Amp
B. 302 Amp D. 155 Amp

19. A 7.5 HP, 440 Volt, 3-phase , 10 Amp, 1730 rpm induction has starting current of 6 times the
full-load current when started at full voltage. The motor is equipped with an auto transformer or
starting compensator. If the auto-transformer is set at 65 % tap, what is the starting line
current?
Aj 25.4 Amp C. 21.6 Amp
B. 60 Amp D. 36 Amp

20. Find the starting torque as percentage of full load torque when an auto-transformer starter with
80% tap is used to start a squirrel cage motor. The short circuit current on normal voltage is 5.5
times the full load current and the full load slip is 3%
A. 72 % C. 50 %
B. 5 8 % D. 65%

21. A three phase induction motor has the following data on its nameplate: 10 hp, 440 Volts, 14.1
Amps, 60 cps, 1750 rpm. If the motor is operated from 370 Volt, 3-phase, 50 Hz source, What
is the approximate rotor speed when delivering its nameplate rated torque?
A. 1,750 rpm C. 1,660 rpm
B. 1.560 rpm &)1,460 rpm

22. A 380 volts,'20 hp three phase 50 Hz six pole wye-connected stator winding squirrel cage
rotor induction motor yielded the following test measurements. The resistance between
terminals of the stator winding is 0.4 ohm. The no-load test input of the stator winding at rated
voltage is 14 amperes, 680 watts. Locked rotor test input at 50 Hz at 20% rated voltage is 55
amperes, 3500 watts. What is the resistance per phase of stator windings in ohms?
A. 0.4 C. 0.3
B. 0.6 D. 0.2
23. At what speed will a synchronous motor operate, if it has 10 poles and is connected to 60 Hz
source ?
(2)720 rpm C. 1800 rpm
B. 120 rpm D. 600 rpm

24. A 10 Hp, 250 V, 3-phase star connected synchronous motor has an effective armature
resistance of 0.25 ohm and synchronous impedance of 4 ohms per phase. To what voltage
must motor be excited to give full load output at unity power factor? Assume armature
efficiency of 88%.
A. 292 V C. 253 V
B. 277 V D. 250V

25. A single phase 10Hp, 220-v motor rated 75%pf, 97.56 % efficient has a load of 8 hp at 210V.
Find the line current?
A. 36.67 A C. 46.3 A
B. 35 A D. 38.84 A

26. A 100 hp, 1750 rpm, 2300 volts, three phase motor draws 150 A and produces a starting
torque Of 120% f the full load value, when started at 2300 volts. If a compensator is to be used,
and the 80% voltage tap is selected, find the starting line current and starting torque.
A. 85 amp & 360 Ib-ft C. 120 amp & 230 Ib-ft
B. 96 amp & 230 Ib-ft D. 150 amp & 360 Ib-ft

27. Find the percentage tap in an auto-transformer starter to start a squirrel cage motor against Vi
of full load torque. The short circuit current on normal voltage is 6 times the full load current
and the full load slip is 3%
A 72 % C. 50 %
B 68 % D 65 %

28. Jbe advantage of a synchronous motor over a wound rotor motor is that
Q y the power factor may be varied
B. its speed does not depend on the frequency of the supply
C. it has no brushes to contend with
D. its speed may be more readily controlled

29. The overheating of an induction motor may be due to


A. overloading A J o w supply voltage
B. loss of ventilation (JD/all of the above

30. Which statement about split phase motor is FALSE ?


it has two windings called main and starting winding
B.) the two stator windings are connected in series across the supply
C. the main winding has low resistance but high reactance
D. the starting winding has high resistance but low reactance

31. If starting winding of a single phase induction motor is left in the circuit, it will
A. run faster C. run slower
B. draw excessive current& overheat D.lspark at light load

32. J f a single phase motor runs hot the probable cause cannot be
(_A jplown fuses C. high voltage
B"Mow voltage D. shorted stator coils

33. If a single phase induction motor runs slower than normal, the most likely defect is
worn bearing C. open-circuit in the winding
B. short circuit in the winding D. none of the above

34. The shaft of induction motor is made of


A. high speed steel C. stainless steel
B. cast iron Dj carbon steel
35. Slip rings are made up of
A. copper Ci phosphor bronze
B carbon D. aluminum

36. An induction motor driven above synchronous speet^-is called


A. induction regulator ( o induction generator
B. frequency changer D. phase converter

37. Which of the following motor has no inherent starting torque ?


A. split phase motor C. dc series motor
B.) synchronous motor D. induction motor

38. Ijne starting capacitor of a single phase motor is generally


(Aj electrolytic capacitor C. ceramic capacitor
K paper capacitor D. none of these

39. A 50 hp three phase motor, rated 220 volts, 75% power factor has 6 poles. The Slip at full load
is 3%. What is the frequency of the rotor current at full load?
A 1.8 Hz C. 60 Hz
B. 5.4 Hz D. 58.2 Hz

40. The no-load speed of a 3<p 4-pole induction motor connected to 50 Hz supply is expected to be
A 1400 rpm C. 1480 rpm
B. 1450 rpm D. 1498 rpm

41. A 3<j> induction motor develops 15 hp when running at slip of 5%.. If the motor has secondary
resistance of 0.075 ohm per phase. What is the rotor current?
A. 46 5 - C. 38.2
B. 51.2 D. 153 5

42. A 3<j>induction motor running with 2.5% slip takes 100 kW from the mains. If the stator copper
loss amounts to 1.8 kW. Determine the mechanical power developed.
A. 90.745 kW C. 98.20 kW
B 95 745 kW D. 97.5 kW .

43. A 30 hp 6 pole, 3 phase, 440 volts, 60 Hz squirrel cage motor operates at full load with an
efficiency of 92%, a power factor of 90% and a slip of 2%. Calculate the torque at full load,
A. 134 Ib-ft. C. 1702 Ib-ft.
. B. 155 Ib-ft. D. 165 Ib-ft.

44. What is the ratio of maximum to full load torque in a motor having slip of 5% at full load and
25% at maximum torque ?
A. 15 C. 2.6
B. 3 D. 5.2

45. An autotransformer used for starting induction and synchronous motor to reduce the current
drawn from the supply lines
A. motor starter C; starting compensator
B. reduper transformer & . four point starter

46 A 3-phase delta connected induction motor, 10 hp, 240 volt, 0.85 power factor has a load of 6
hp. One winding is suddenly cut,
A. the motor will run but reduce speed the motor will run
B. the motor will stop D. the motor will run but overheat

47. A 50Hp, 440-V, three phase 60 cycle induction motor has a power factor of 87 2 and efficiency
of 90.6%. What is the power input Hp?
A. 50 C. 47
B 45 D. 55.2
48. The efficiency of a 550 volts 3-phase motor is 90% when the line current is 100 ampere per
wire at a power factor of 0.92. Calculate the output of the motor.
A. 68 C. 79
B. 72 D. 96

49. A 3-phase motor rated 50 hp, 220 volts, 75% power factor and 95% efficiency. What is the
current at 210 volts.
A. 144 Amp. C. 238 Amp.
B, 137 Amp. D. 79 Amp.

50. A single phase induction motor is rated 220V, 25 hp and 75% pf it has 3% losses at full load.
What is the full load current?
A. 151.5 A C. 116.5 A
B. 161.2 A D. 113 A

51. What is the average slip in percent of a synchronous motor at running speed?
A. 2.5% C. 2%
B, 3 % D. 0

52. For a 3-phase, 4 pole, 60 Hz synchronous motor, the frequency, pole number and the load
torque are halved. The motor speed will b e ...... rpm
A. 1750 C. 900
B. 1800 D. 3300

53. A 300 hp, 2300 volts 60 cps synchronous motor has a synchronous impedance at starting by
action of the amortisseur grids, so that 6.00 per unit current will be drawn if rated voltage is
applied. It is desired to reduce the starting current drawn from the line to 2.5 per unit. What
voltage would be applied to motor at starting?
A. 1285 volts C. 1485 volts
B. 1385 volts D. 1585 volts

54. A 15 Hp, 4 pole 60 Hz, wyfeconnected, 208 volts induction motor draw 12 kW from the line
have losses as follows: Stator copper loss 900 W, Rotor copper loss 300W, Iron loss 800W
and Friction & Windage Loss 100W. Determine the speed of the motor in rpm.
A. 1710 C.1747
B . 1720 D. 1700

55. Select the motor thatwill give relatively high starting torque......
A. shaded pole motor f l capacitor run motor
B. split phase motor D. capacitor start motor

56. A split phase induction motor is defective, but if the motor is energized and the rotor is rotated
by the hand, the motor runs. What is wrong with the motor.
A. One winding is open C. capacitor is defective
B. loose connection winding is burn out.

57. What is the part of a synchronous motor that allows it to develop starting torque?
A. Cranking system C. Amortisseur winding
B. Pulley D. Pilot starter

58. In a shaded pole single phase motor, revolving field is produced by the use of__________ .
A. inductor , C. capacitor
B. resistor D. ^hading coils

59. A three phase induction motor rotates at of 3510 rpm at normal voltage and frequency of 60
Hz. What is the motor slip in percent'?
A. 3.5 % C. 2.5%
B. 4% D. 3%
a
60. What type of motor is used for food mixer?
A. Split phase Universal
B Shaded Compound shunt

61.- If a motor has a slip of 2% at rated voltage , determine the approximate value of the slip when
developing the same torque at 10% above the rated voltage.
A. 1.8% C. 1.55%
B. 1.75% D. 1.65%

62. A 440V 3-phase, 50 Hp, 1750 rpm squirrel cage induction motor is connected to 440 V bus
through a feeder which the voltage drop of 10% of bus voltage when the motor is drawing rated
current. What is the he maximum continuous power the motor can safely deliver to its load?
A. 40 Hp C. 42 Hp
B. 45 Hp ■ D. 48 Hp

63. A 2300-V, 3-phase, wye-connected round-rotor synchronous motor has a synchronous


reactance of 3 Q per phase and an armature resistance of 0.25 .Q per phase. The motor
operates on load such that the power angle is -15°, and the excitation is so adjusted that the
internally induced voltage is equal in magnitude to the terminal voltage. Determine the
armature current.
A. 115.6A C. 105.6A
B. 151.6A D. 156.1A

64. A 400-V. 3-phase, round-rotor synchronous motor has an efficiency of 92% while delivering 18
hp (at the shaft). The per-phase synchronous impedance of 0.5 +j 1.5 Q If the motor operates
at 0.9 lagging power factor, determine the power angle.
A. -7.4° C. -24.3°
B. 15.4° D. 10.8°

65. A synchronous motor has moment of inertia 20 kg« m2 and damping coefficient 15
N • m • s/rad. If the machine has a synchronizing torque constant of 800 N • m/rad, determine
the natural frequency of oscillation,
A. 1 Hz C. 2 Hz
B. 4 Hz D 6 Hz

66. If the no load speed of a squirrel cage induction motor connected to a three phase 50 cycle
supply is 740 rpm. The motor has
A. 2 poles 6 poles
B. 4 poles D: 8 poles

67. A certain motor is driven by a 1750 rpm dc shunt motor. If the power supply is to be changed to
three phase, 50 cycles, ac., the most suitable replacement motor would be
A. synchronous motor C. ac commutator motor
B. capacitor motor D, squirrel cage inductionmotor

68. The value of slip at maximum torque condition ( under running condition ) in terms R2and X2 is
A. S = X’-IR -, ( f r p = R ?lX 2
B. S = 2 R J X - V s = E-J 2X2

69. An over-excited synchronous motor take


A lagging^current C. leading current
B. lagging and leading current "Dr none of these

70. When the excitation of an unloaded salient pole synchronous motor suddenly gets
disconnected, then
°/<Othe motor stops C. it runs at same speed
C. it runs at lower speed D. it runs at very high speed

P R A C T IC E Makes P erfect!!!
CIRCUIT
TRANSIENTS
This Free Qualit\ .Manual is
Found Only at P O W E R L IN E

E-MAIL:powerl inereviewcenter@yahoo. com

M E Z Z A N IN E F L O O R , D O N A A M P A R O B U IL D IN G
C O R N E R E S P A N A & G. M . T O L E N T I N O S T R E E T S
S A M P A L O C , M A N IL A
TEL. NOS. 7 3 5 -7 3 -0 2 & 7 3 3 -2 1 -1 8

CEERS - POWEEINE REVIEW CENTER


T E L . N O S . ( 0 3 2 )2 6 1 -2 2 4 4 & (0 3 2 )2 6 1 -8 4 5 2
—^

CIRCUIT TRANSIENTS
(B ILA TER A L CIRCUIT ELEM EN TS ONLY)

Transient disturbances are produced whenever:


1. An apparatus or circuit is suddenly connected to or disconnected from the supply
2. A circuit is shorted and,
3. There is a sudden change in the applied voltage from one finite value to another

Note. Transient currents are not driven by any part o f the applied voltage but are entirely associated
with the charges in the stored energy in inductor and capacitor .

Classification of Transient Disturbances


1 . Initiation Transients - these are produced when a circuit, which is originally dead, is energized
2 Subsidence Transients - these are produced when an energized circuit is rapidly de-energized and
reaches an eventual steady state of zero current or voltage, as in case o f short
circuiting an RL or R-C circuit.
3. Transition Transients - these are due to sudden but energetic changes from one steady state to another
4 Complex Transients - these are produced in a circuit which is simultaneously subjected to two
transients due to two independent disturbances or when the disturbing force
producing the transients is itself variable
5 Relaxation Transients - the transition occurs cyclically towards states, which when reached become
unstable themselves.

RESPONSE O F CIRCUIT PARAM ETERS

Resistor
= R/ volts
i _ dq
1 ~ — amp
dt

Inductor
di
eL = L — volts
dt
- C l ----- *-
ubut 1' = —
dq
« ----- dt
L(henr^) ,2
„ d q
et. - l — —. volts
d t2

Capacitor
Qo
ec = — ±
c c

, H ± if aiding E or opposing

c (fttrod}
e c = — |idt
C b

Transients in R-L series Circuit

When the switch is closed


V 'i
di
E = Ri + L
i dt
di
L — = E - Ri
dt
. i
L
di dt
E -R i L
Integrating

f di____fdt
J E - Ri “ J L

Rt
In ( E- Ri) = - — + K
L

Rt
E - Ri = e
L K
e
Rt
E - Ri = Ke

For constant of integration K, consider initial condition

@ t = 0, i = 0
E = Ke° then K=E
-Rt
E - iR = Ee

f Rt ^
E L
/ =— 1 - e Amp
R
J
Voitage across R at any instant

f „Rt A
eR= ir = e 1- e ^ Volts
V J
Voltage across L at any instant

Rt
e„ L = h, —
^ = L — Re “I
E . -----
dt R L
_ bl
61,, E e ^ Volts

Power across R at any instant

( K t\ 2
p R= eR. / = 1 - e v watts
R
V J
When t = 00 PR= E2/R

Power across L at any instant

( 2Rt
R t ____
Pl = eL. 1 e~L -e L
R
V y
When t = oo P l=0

Energy Stored in the mantic field

dWL = P L . dt

d wL = e L . i dt

T . di
=L1_ . dt
dt
\dw = \ L i di

WL « - joules
2

Time Constant - the time for the current i to reach 63 .2% o f its final value or the time for the negative
exponent o f e to become unity

Time Constant, y - — sec


R
Transient in R-C Series Circuits (DC)

When the switch is closed


q
E =R /+
C
but / = ’H
dt
L

J x
E=R —
dt
+ I

dq dt
E -q/C R
1
Multiply both sides by - - and integrating
C

1
1
| J — = j j _ — dt
J E -q/C J Rc

Ln ( E -q/c) = - — + K
Rc

E - q/C = K e £

Initial Condition: @ t = 0 , q = q„ = 0

K=E

Rc
Then E - q/C = E e

coul

dq
Transients Current:
dt
t
i = E - RC amp
—e
R

If C has initial charge

t
E amp
e» RC
^R
Voltage A cro ss R at any instant

eR = / R = E e rc Volts

Voltage A cro ss C at any instant

(
t
q R RC
< ? c = - = E 1- e Volts
C

Energy Stored in the Capacitor

Wc = jV c dt = J e c • i • dt

wc= fa 5a = i =i f qd, =
Jc dt C J dt C J 2C

q2
Wc = — joules
2 C

but q = ec C

Wc = — ec 2 C joules
2

Time Constant in RC Series Circuit - is the time for charge q to reach 63 .2% o f its final value.

Time Constant, y = RC sec


6

DO UBLE ENERGY TR A N SIEN TS


( Transients in RLC Series Circuit)

When the switch is closed


j- i j*

E = Ri + L — + — \idt
dt C J
Taking the first derivative

0 = R * + L d— + —
■2 + C
dt dt"

d 2i R R
& di _i_ _0
d t2 L dt LC

Let D= —
dt

D2i + - ' D i + ' = 0


L LC

R 1
D2 + — D + — " i = 0
L LC

R
By Quadratic Formula: D=
f Rl - 4 (1)
^LC
u ,

D = _ A ±
2L f -
V2L v LCy
R
Let a = - —
21

R 1
2L LC

Roots: Di = a + P
D; = a - P

Case I Overdamped Case

f 1 ^
VLCy
P is real number
D, = a + P
7

D; = a - P

[ D - ( a + P ) ] .[ D - ( a - 3 ) ] » '“ 0

the solution to differential equation is

/ = C,,e <“- » ' + Q e ^ ' -^Sec

/ = e “‘ [C, e e, + C2e pt] amp

Case II Critically Damped Case

2
if m
V2Lj U cJ

“ P. 3=0
Di=a, D2= a

[D - a] [D-a]/ =0

the solution to differential equation is

/ = C, e0 , + t C 2 eat

/ = ea t [ Ci + 1 C2]
I amp

R
(
2L vLC y
3 is imaginary

Di = a + j3 D: = a -jP

t D- ( a +jP)] [ D - (a - jP] / = 0

/ = K,e “*•*'+ K: e (a' )P)t


= e m [ Ki ( cos P t + j sin pt) + K2 (cos Pt - j sin pt ]
=e ( K, + K 2) cos p t + ( jK, - jK ,) sin Pt]

i = eat [ Ci cos Pt + C2 sin pt]

Note: to obtain Ci & C2 constant, consider the conditions


AC TRANSIENTS I

Resultant Transient: i J transient componnent ^steady state component

RL Circuits

e = Em sin(©t + (j))
@ t=0 if e=0, then cp=0°
@ t=0 if e=Em, then cp=90°
J: t=(?
W hen the switch is closed,
di
R; + L — =? Em sin(cot+q>)
dt
The solution to differential equation is

o)L
ZL= R + j o L 0z=arctan
R

(p - position o f sinusoidal voltage at the instant o f switch

RC Circuits
W hen the switch is closed,

Ri + — J i dt= Em sin(ot+(p)
C

The solution to differential equation is

t Em
i = Ke + ----- sin ( rat + (p + 0z)
POWERLINe r e v ie w c e n te r Inc.
Hie Ultimate EE. Review Center
CIRCUIT TRANSIENTS REVIEW PROBLEMS
1. A coil of inductance 10.5 H is connected to a source voltage of V=(3t+25.4)1/2.
What is the current in the inductor after 5.25 sec?
A. 5.5 A Cp2.88 A
B. 6.15 A D. 3.22 A

2. Find the voltage across a 0.01 micro-farad capacitor at t=0.25micro-second if


v(0)=0.6 volt and the current through it is 0.2sin2;r 106t amperes.
A. 3.18 volts C. 5.12 volts
B j 3.78 volts D. 4.56 volts

3. The voltage across 20 micro-farad capacitor varies with time as given by ve


=(10.75 -1.5e ~1000t) volts. What is the current through capacitor after 1 ms?
A. 0.023 A C JO.011 A
B. 0.013 A D. 0.033 A

4. A 1000 pF capacitor, a 1.0 mH inductor, and a current source of /'=2 cos 106t mA
are in series. The capacitor voltage is zero at t=0 . Find the energy stored in the
inductor at t=0.
'A, 2x1 O'9 J C. 2x1 O'8 J
B. 2x10"® J D. 2x1 O'3 J

5. A shunt winding of a machine has a resistance of 80 ohms and inductance of 4 H.


It is switched suddenly on to 220 V dc supply. The time taken for current to rise to
half its final steady value is nearest to
A. 0.035 sec C. 0.05 sec
B. 0.025 sec D. 0.25 sec

6. The decay current of a coil is recorded. It is found that iL= 10 A at t =2ms and i[_
= 3.58 A at t = 6 ms. What is the time constant of the coil
A. 3 ms C. 2 ms
B. 6 ms D. 4 ms

7. The armature of a relay working on 200 V circuit operates when the current
reaches 0.24 A. It is required that the relay shall closes, 0.004 sec after the relay
circuits is closed, this time is corresponding to the time constant of the circuit.
What is the inductance of the relay circuit?
A. 2,1 H C. 3.3 H
B. 5.27 H D.1H

8. A 50 fi F capacitor is discharged through a 100 KQ resistor. If the capacitor was


initially charged to 400 V, determine the energy stored after 600 ms.
A. 3.14 joules C. 4.0 joules
B. 2.4 joules D. 4.2 joules
S '

9. A certain precision 1 micro-farad capacitor has a very high resistance material


used between its conducting surfaces. The capacitor is charged to 1 volt at t=0
and disconnected from the source. It is found that the voltage drops to 0,9 volt in
100 hour. Find the insulation resistance of the capacitor.
A 3420 GQ C. 4220 GQ
B. 1240 GQ D.6524 G Q

10. A 5 micro farad capacitor is discharged suddenly through a coil having an


inductance of 2 H and resistance of 200 ohms. The capacitor is initially charged t
a voltage of 10 volts. Find the additional resistance required just to prevent
oscillation.
A. 1625 ohms C. 1065 ohms
B. 1265 ohms D. 1025 ohms

11. A series RLC with R=2 ohm and L=0.5 H. Find the value of capacitance so that
the circuit is critically damped.
A. 0.25 F C. 1F
B. 0.5 F D. 2F

12. A circuit consisting of 10 ohms resistor, 10 mH inductor 10 ^F capacitor in series


is connected to a 10 V dc source. The transient component of the current after
the switch closes has a frequency of oscillation of
A. 497 Hz C. 159Hz
B. 2420 Hz D. 1000 Hz

13. A series RLC circuit consisting of 20 ohms resistor, 10 mH inductor and


uncharged 100 juF capacitor in series is connected to a 100 V dc source. Find
the resulting transient current when the switch is closed at t=0.
A. 1 o V 1000t C. 104t e 1000t
B i o V 1000tsin1000t D. 1 o V 10001 sinh1000t

14. A 2 H inductor, 16 ohms resistor and 0.02 F capacitor is suddenly connected to a


300 volts Find the transient current when the switch is closed at t=Q
A. 25 e-4* cos3t C. 50 e*41 cos3t
B. 25 eAt sin3t ~ D. 50 e"41 sin 3t

15. A series RC Circuit with R = 100 ohms and C = 25 micro-farad has a sinusoidal
voltage source e= 381 sin (377t +cp) volts applied at t=0 when passing to its
maximum value. What is the resultant transient current equation.
A, 2e'400t+2.6sin(377t+136.7°) C. 2e'400t+2.6sin(377t+46.7°)
B. 1.9e‘400t+2.6sin(377t+136.7°) D. 2e~400t+1.9sin(377t+136.7°)

%
CIRCUIT TRANSIENTS SUPPLEMENTARY PROBLEMS

1. A coil of inductance 25 H with initial current of 1 ampere is suddenly connected to a source


voltage of V=8.9+(3t)1/2 volts. What is the current in the inductor after 5sec?
A. 4.329 A j -O'. 3.329 A
B. 4.525 A i r . A, D. 2.329 A

2. A Capacitor has an initial charge 8 3 coulomb If the current flows through it is given by the
equation i=2t+3 amperes, find the charge when t=1 sec?
A. 16.3 C. 4
B. 12.3 D. 6
I - ( /
3. A coil of inductance 15H is connected to a source voltage of V=6t1/2+28.5. If the initial current is 15
ampere, What is the current in the inductor after 2.5 sec?
A. 15.5 A C)y20 8 A
B. 16.5 A D. 18.2 A

4. The current in a 125 farad capacitor is i=t(t+6.83)1'2 ampere. Find the voltage after 1 second if it is
initially charged at 25 volts.
A. 26 volts / . { cl < C. 11 volts
B 25 volts ’ 77' D. 19 volts
Cl S
5. The rate of rise of current through an inductive coil in maximum
A. afterone time constant *C> at the start of current flow
B. near the finalmaximum steady value 6. at 63.2% of its maximum steady value

6. A coil of 15 H inductance and 10 ohms resistance is suddenly connected to a 20 volts DC source


by closing switch. The energy stored in the magnetic field 2 seconds after the switch is closed is;
A!16.3joules C. 3 0 joules
B. 20 joules D. 10 joules

• 7. A DC voltage of 80 volts is applied to a circuit containing a resistance of 80 ohms in series with an


inductance of 20 Henries. Calculate the growth of current at the instant completing circuit.
A'i 4 Amp/sec C. 2 Amp/sec
B: 1/2 Amp/sec D .1/« Amp/sec

8. A 10 ohms resistance R and a 1.0 H inductance L are in series . A direct current voltage of E 50
volts is applied across a series circuit at time t = 0 initial current is i(0)=0 Solve for the differential
equation and determine the resulting current i(t) at t=0.1 second.
A. 3.48 C. 2 87
B. 3.16 D 3 82

9. A constant voltage is applied to RL circuit at t = 0. The voltage across the inductance is 20 V at t =


3.46 ms and 5 V at 25 ms. If the inductance is 2 H, what is the value of R?
A. 178 2 Q C. 182.7 Q
B. 127.8 D '■-d? D. 128.7 Q

10. A circuit whose resistance is 20 ohms inductance of 10 H has a steady state voltage of 100 volts
suddenly applied to it. For the instant of 0.50 second after the voltage is applied, determine the
total power input to the circuit.
A. 200 watt? C. 116 watts
6;31 6 watts D. 500 watts

11. A coil has 2500 turns, and a current of 10 A flowing in it produces an effective flux of 0.06 Weber
If the resistance is 12 ohms, what is the time constant of the coil
A. 1.0 sec C. 0.75 sec
12 A coil having L = 150 H, R = 200 ohms is connected in series with 100 ohms resistor. A 240 -V
dc source is connected to the circuit at t = 0. What is the voltage across the coil at t= 0.5 sec.
A. 185.9 V C. 189.4 V
B. 88.2 V D. 120 V

13. A 240 V dc generator supplies current to a parallel circuit consisting of a resistor 600 Cl and a coil
with resistance 300Q and inductance 200 H. The system is at steady state. Determine the current
In the coil 5 ms after the breaker is tripped.
A 0 78 A C. 0.8 A
B. 0,68 A D. zero

14. A coil of 10 H inductance and 5 ohms resistance are connected in parallel with a 20 ohm resistor
across 100 V DC supply which is suddenly disconnected. The voltage
across the switch contacts at the instant of separation is .
A, zero C. 100V
B. 400V 9 D 500V

15. A 250 volts, 10 Kw, DC generator is separately excited. It has an effective armature circuit
resistance of 0.5 ohm andinductance of 0.10H when it is supplying its rated current.Suddenly the
terminals beyond itsprotective circuitbreaker are short circuitedwith a short circuitresistance of
0.2 ohm. The breaker operates 0.02 sec after the fault occurs. Neglecting the saturation of the
magnetic circuit, the maximum current which tfje generator is subjected is nearest to
A. 80 Amp C. 85 Amp
B. 90 Amp D. 95 Amp

16. A generator has a field winding with inductance of L=10 H and a resistance Rf= 0.1 ohm. To break
the initial field current of 1000 amperes, the field inserts a field discharge resistance of Rd across
the field terminals before it contacts open. As a result, the field current decays to zero according tc
Ldi/dt-t- iR=0; where R=Rf+Rd the differential equation, preventing the sudden decrease of / to
zero, a resulting high inductive voltage due to L. Solve for the differential equation and determine
the vaiue of Rd that will limit the voltage across it to 1000 volts.
A. 0.9 C. 0.85
B. 0.8 D. 0.95

17. A coil of 10 H inductance and 5 ohms resistance are connected in parallel with 20 ohm resistor
across 100 V DC supply which is suddenly disconnected The rate at which energy stored is losinc
0.3 sec after switching is
A. 2232 j/s C. 2332 j/s
B 2223 j/s D 3322 j/s

18 A 20 ohm resistance R and a 0.001 farad capacitance C are in series. A direct current voltage E
100 volts is applied across series circuit at t=0 and the initial current is l(0)= 5 amperes. The
Applicable differential equation is Rdi/dt+i/C=0. Solve for the differential equation and determine
the resulting current i(t) at t= 0.01 second.
A 3 34 ,C, 2.78
B. 3.67 D. 3.03

19. A capacitor of 8 micro-farad is to be charged by a voltage of 400 volts through a resistor of


100,000 ohms. How long it will take for the voltage across capacitor from its initial zero value to
300 volts?
A. 1.3 sec C. 1.1 sec
B. 1.5 sec D. 0.8 sec

20. A series RC circuit consist of 40 ohms resistor and an uncharged 300 micro-farad capacitor. What
is the time constant of the circuit?
A. t = 0.0132 sec C .t = 0.012 sec
B. t = 1.2 sec D t = 1.333 sec
21. A capacitor of 2 micro-farad with an initial charge q = 100 micro-coulombs is connected across
the terminals of a 100 ohm resistance at t = 0. Calculate the time ( micro-sec) in which the
transient voltage across resistor drops from 40 to 10 volts
A.)277.40 C. 477.20
B, 274.20 D 472.70

22. An 80 (j, F capacitor in series with a voltmeter of 10,000 ohms resistance is connected suddenly to
a 100 V dc supply How long it will take the voltmeter read 40 volts
A. 0.04 sec (2^0.733 sec
B. 0.02 sec D. 0.523 sec

23. A 100n F Capacitor charged to 24 V is connected with a 200jj. F uncharged capacitor, a 1 kilo-ohm
resistor and a switch . What is the current 0.1 sec after the switch is closed
A. 6.3 mA C. 5.36 mA
B. 24 mA D. 0.16 mA

24 A capacitor of 2 micro-farad with an initial charge is connected across the terminals of a 10 ohm
resistor and the switch is closed at t = 0. Find qo ( micro-coulomb) if the transients power in the
resistor is known to be Pr = 360e ' 100,0001 -,
A. 1200 C . 120
B. 102 TT2100

25. The transients current in a loss-free L-C circuit when excited from an ac source is a/an
sine wave.
A. over damped Oundamped
B underdamped D. critically damped

26. Transients current in an RLC circuit oscillatory decaying when


A. R = 0 C. R > 2 -4 L IC
R < 2 VzTC D. R = 2 4 l f c
<3
27. A series RLC circuit consisting of 20 ohms resistor, 20 mH inductor 100 jiF capacitor in series is
connected to a 100 V dc source. Find the additional resistance just to prevent oscillation of the
resulting transient current. . ...
A. 82.8 ohms C. 8.28 ohms pL- V 3 -0 '1 O )
, B. 28.2 ohms D 282 ohms

28. A 60 hz sinusoidal voltage of maximum value 400 V is applied to a series circuit of resistance 10
ohms and inductance 0.1 H. Assuming that the voltage is zero at the instant of application,
calculate the resultant transient current 0.002 second after witching on
A. 2,7 A C. 7.3 A
d T -3 .2 A D. -4.2 A

29. A circuit consisting of 20 ohms resistor, 20 mH inductor and a 100 micro-farad capacitor in series
is connected to 200 V DC supply. The capacitor is initially uncharged. Find the maximum
instantaneous current.
A. 6.45 Amp C. 7.45 Amp
B. 8.45 Amp D. 9.45 Amp

30. A 10 ohms resistance R and a 1.0 H inductance L are in series . An ac voltage of e(t)=
100sin(377tvolts is applied across a series circuit. The applicable differential equation is
Ri+L(di/dt)=e(t). Solve for the particular solution (without complementary solution) to the
differential equation and determine the amplitude resulting sinusoidal current.
A. 0.2645 Ampere C. 0.292 Ampere
B. 0.321 Ampere D. 0.241 Ampere
CONTROLandSYSTEMS
tUECTROHAGNETICS
This Free Quality Manual is
Found only at POWERLINE

E- MAI L: powerlinereviewcenter@yahoo.com

POWERLINE REVIEW CENTER


M EZ Z A N IN E FLO O R , D O N A A M P A R O BUILDING
' CO R N ER ESPAN A G. M. TO LEN TIN O S T R E E T S
S A M P A L O C , M A N ILA
TEL. NOS. 7 3 5 -7 3 -0 2 Sc 7 3 3 -2 1 -1 8

TEL. NOS. (0 3 )2 6 1 -2 2 4 4 Sc ( 0 3 2 )2 6 1 -8 4 5 2
PART 1 : CONTROL SYSTEMS
Control System - is an arrangement of physical components connected or related in such a manner as to
Commend, direct or regulate itself or another system.
Purpose of control system
The purpose of control system usually identifies or defines the “output” and “input”. If the
output and input are given, it is possible to identify or define the nature of system's components.

Three Basic Types of Control Systems:


1. Man-made control systems
2. Natural, including biological control systems.
3. Control systems whose.components are both man-made and natural.

Two General Categories of Control Systems:


1. An Open-Ioop control systems is one in which the control action is independent of the output.
2. A Closed-loop control system is one in which the control action is somehow dependent on the
output. They are commonly called “Feedback” control systems.

The Control Systems Engineering Problem:


1. Analysis - is the investigation of the properties of an existing system.
2. Design - problem is the choice and arrangement of control system components to perform a
specific task

The Representation of the Problem: The Model


Three basic representations (models) of physical components and systems are extensively
employed in the study of control systems.
1. Differential equation^and other mathematical relations.
2 Block diagrams
3. Signal flow graphs

The Solution of Linear Constant Coefficient Ordinary Differential Eouation:


Consider the class of Differential Equation of the form:
A' d*y A d lx
> a. - 4 - = > —
P d tl to d tl
Where: “ t ” is time, the coefficients a t & bt are constant,
X = x{t) <the input> is a known time function and
y = y {t) <the output> is the unknown solution of the equation.
n —is called the order of the differential equation.
The solution of a differential equation of this class can be divided into two parts, a “ free
responsed’ and a “force responsed. The sum of these two responses constitute the “total respon sed
or the solution y (t ) of the equation.
Electrical Analogies: There are two electrical analogies of mechanical systems;
1 The Voltage-Force or Mass-lnductance analogy
2 The Current-Force or Mas$~Capacitance analogy
The following table shows both the voltage-force & current-force analogies for mechanical systems:
Voltage-force analogy Current-force analogy
Force, F Voltage, u Force, F Current, /
Velocity, dx/dt Current, / Velocity, dx/dt Voltage, o
Damping, b Resistance, R Damping, b Conductance, 6
Mass, m Inductance, L Mass, m Capacitance, C
Spring constant, k Elastance, 1/C Spring constant, k Reciprocal of L (ML)
In general, in laying out equivalent electric circuits for mechanical systems, the following rule is observed
If the forces act in series in the mechanical system, the electrical elements representing these forces
are put in parallel. Forces in parallel are represented by elements in series in electric circuits.
Stability & Transfer Functions:
Stability of a system is determined by its response to inputs or disturbances. A stable system is one
that will remain at rest unless excited by an external source and will return to rest if all excitations are
removed. A system is “stable” if every bounded input produces a bounded output.

Routh Stabiiity Criterion:


Is a method for determining system stability that can be applied to an nth. Order
characteristic equation of the form: dn^n + ■+* — ^ ai $ + = 0
The criterion is applied through the use of Routh table defined as follows;
an an-2 an - 4*'*
Sn
S n~i <*71-1 ^ n —3 an -
C* 71 — 2 bi 62 b% • • •
Cl C2 c3 -

Where an , CLn-i, ***, a 0 are the coefficients of the characteristic equation and
, ^ n -l^ n -2 T l^ n - 4 ~ &n&n-5
b1 = ----------------------------- , &2 = ------------------------------, etc.
a 7i-t a n~~l

biUn-2 “ an- l^2 h ^ n -S “ ^ - 1 ^ 3 ^


cl = ----------- ---------, c2 = ------------ ---------.etc.

The table is continued horizontally and vertically until zeros are obtained. Any row can be multiplied by a
constant before the next row is computed without disturbing the properties of the table. All roots of this
characteristic equation have negative real parts if and only if the elements of the first column of the
Routh table have the same sign. Otherwise, the number of roots with positive real parts is equal to the
number of changes of sign.
Transfer Function H (s ) of a system is defined as that factor in the equation for F (s ) multiplying
the transform of the input . For the given system describe above, the transfer function is.
Y (s) = H (s) *X (s) 4- {terms due to all initial values X* , y$ }
Note: independent of the input signal, it solely determined by the parameters of the circuit or network.

Block Diagrams & Signal Flow Graphs:


Block diagrams - these are shorthand, graphical representations of either the schematic diagram of
a physical system, or the set of mathematical equation characterizing its parts.

Signal flow graphs - is a pictorial representation of the simultaneous equations describing a


system. It graphically displays the transmission of signals through the system, as does the block diagram. But it
is easier to draw and therefore easier to manipulate than the block diagram.

The Mason's Gain Formula:


A procedure that allows to find the transfer function, by inspection, of either a block diagram or a
signal flow graph.

W R(s) A£t
Where: M k = gain- of the Mi. term forward path {k —1,2,..., ft)from input node R.{s) to the output
node C (s)
A = 1 - (sum of all loop gain) + (sum of products of loop gains over all sets o f ‘two non­
touching loops) - (sum of products of loops gains over all sets of ‘three’ non-touching
loops) + ...
A* = value of A if all loops that touch the kxh. forward path from R to C are expected.
The function A(^)is known as the “determinant” of the signal flow graph.
PART 2 : ELECTROMAGNETICS

Coulombs Law / Electrostatics:


It states that the force between two very small objects separated in a vacuum or free space by a
distance which is large compared to their size is proportional to the charge on each and inversely proportional
to the square of the distance between them.

Mathematically. F - - — — ■- a R Newtons
4 kS 'R 2
Where: Qx, Q2 = respective charges in Coulomb, R = separation distance in meters
£ = permittivity of the medium = S0 • Sr ( for air 8r = 1.0 )

£0 —permittivity constant = 8.854 xlO 12 F/m « ----- *10 9 F/m


36 k
a R = unit vector in the direction of R

Electric Field Intensity:


It is a vector force on a unit positive test charge. Electric field intensity must be measured by the unit-
Newton per Coulomb or Volts per meter.
mm
Mathematically: E1?= — -Q—- a R Volts Newton
----o r---
4 n s-R m Coul
Standard Charge Conf/auartion: '
1. Point charge - a field of a single point charge Q is given by:
„ Q Volts Newton , . f
^ ■■ a r ------- o r --------— <spherical coordmates>
4Trs-r m Coul
2. Infinite line charge - if charge distribution with uniform density of p f (Coul/m) along an
infinite straight line which will be chosen as the Z-axis, then the field intensity is given by:
pP Volts Newton ,
.j^ = ----- :— a r -------o r ------- — Cylindrical coordinates>
2ns *r m Coul
3. Infinite sheet charge - if charge is distributed with a uniform density of p s (Coul/m2)
over the plane, then the field intensity is given by:
Ps Volts Newton
k = — a R ------- o r ----------- <cartesian coordinates>
2s m Coul
Where: a n = unit vector perpendicular (normal) to the plane
Gauss's Law:
The electric flux passing through any closed surface is equal to the total charge enclosed. For the
r -»
electric flux density D we have: y/ —Qcndoscd = f D« dS
Where: if/ —electric flux in Coulomb (a scalar)

D = electric flux density in Coulomb/m2 (a vector)

dS = differential surface in m2 (a vector) & j> = integral of closed surface


Note: D = £ 0£ rE in Coulomb/m2
Gauss Divergence Theorem:
The surface integral of normal component D over the surface is equal to the volume integral of
divergence D over the volume integral.
! » • dS ~ \ p vdV = f (V • D )dV in Coulomb
J vl1 V
But p v = div D = V • D from Maxwell’s equation
d d d
Where V = ”del” or “nabla” operator = — a v 4-— H-----a y
dx x dy y dz z
Electric Potential Between Two Points:
The potential of point UA” with respect to point “B57 is defined as the work done in moving a unit
positive charge <2u from “B” to “A”
Work f . Joule
^-us = “ — - - J E • <11 in volts or
Qu Coul
Relation ofV toE is E = —V V 9 where V V = gradient of V.
Current and Current Density:
The electric charges in motion constitute a current, Current density is a vector J.

I = j \ j • dS in Amp
Poisson’s Equation & Laplace's Equation:
From V • D = p , also D = S * E and E = —V V

Consider, V • 6'(—V F ) —p V • V V = —— or V 2V = ——
thus,
8 £
Where: V • VV —divergence of divergence of V = Laplacian of V
Ampere's Law & Magnetic Field:
Biot-Savart Law - a differential field strength , results from differential current element I ♦dt
. The field varies inversely with the distance squared, is independent of the surrounding medium, and
has a direction given by the cross-product of I *d l and a^ .
■I<#xa» Amp
H = in --------
4n r2 meter
Current Density. J & Magnetic Field. H:

V x H = J = (curl H ) = crE
Where: <7 “ conductivity of the material in mho/meter
a, ay a*
d d d dHz 8H y A (d n y an 'dHy dH,
Curl H = av + a>’ +
dx d> dy dz dx dx dy
Hy Hz
F I j x Density & Stokes Theorem: (Ampere's Circuital Law)

B= j A * d( = £ b • dS ~ / i0j>J • dS
Where: A = magnetic potential vector.
Electromagnetic Waves:
Wave Equations: V x H = (cr + jo)e)E also V x E = ~jcojuH
V • E = 0 & V «H = 0
Propagatfon Constant: y - a + jj3

Where: a = attenuation factor in neper/m = co


X

CARTESIAN COORDINATE SYSTEM

P(P> <P, Z)

CYLINDRICAL COORDINATE SYSTEM


Z

<P)

■y

SPHERICAL COORDINATE SYSTEM


P O W E R U n e r e v i e w c e n t e r Inc.
The Ultimate E.E. Review Center

CONTROL SYSTEMS & ELECTROMAGNETICS


REVIEW PROBLEMS
1. Any externally introduced input signal affecting the controlled output is known a s _____ .
A. signal C. feedback
B. stimulus D. gain control

2. A power-amplifying feedback control system where in the controlled variable is


mechanical position or a time derivative of positiqn such as velocity or acceleration is...
A. timer C; servomechanism
B. regulator D. automation

3. Is that part of the total response which does not approach zero as time approaches
infinity?
TA.; steady state V1 ! ' C. signal
B. transient D. force

4. Is that part of the total response which approaches zero as time approaches infinity?
A. steady state C. signal
B. transient t ^ „ D. force

5. Analysis of control systems by Laplace Transform technique is NOT applicable f o r ____.


A. discrete-time system C. linear system
B. time-invariant system D. unstable continuous-time systems

6. Method of determining continuous system stability, a criterion that can be applied to an


nth. Order characteristic equation.
j V ) Routh criteria C. continued fraction criteria
B. Hurwitzcriteria D. locus of point

7. The laplace transform of a unit impulse function, 8 (t) is ______ .


A. 1/s C. 1/s2
. B. 1.0 D. 0

8. Determine the initial value of the function f ( t ) whose Laplace is F(a) = - -------
s 4 + 353 + 52 + 5
A. 1 B. 2 , C. 0 D. 3

9. Determine if the following characteristic equation represents a stable system:


' 53 + 4 s 2 +85 + 1 2 - 0
A. No, it is unstable C. indeterminate
B. Yes, it is stable D. undefined

10. Consider the system with differential equation y'+2y - u'+u, find the transfer function.
a 5+1 r , ' S+ 2 „ s ^ s
A. * B. ------- C. ------- D.
•S+ 2 5+1 5+ 2 5+ 1
11. Find the transfer function C (s )/ R{s) for the system shown.

G\G2 c. G |~ G2
1- G 2H, \ - G 2H, X

B. Gl + G 2 D.
1- G ,H \

12. Determine the damping ratio C and undamped natural frequency <y„in rad/sec for the

d 2y , Ady_
second order system: 2 — ~ + 4 — + 8>> = 8x
dt
A 0,5, 2 B. 0.5, 4 C. 0.25, 1 D. 2, 4

13. Determine the time constant r in sec. and damped natural frequency o)d in rad/sec for
d 2y , dy
the second order system: + 5— + 9y = 9x
d t1 dt
A. 0.4, 0.66 B. 0.5, 1.66 C. 0.4, 1.66 D. 0.25, 0.55

14. A servo system for a pen plotter is given by the following block diagram. Find the value
of K required to get the fastest response without any overshoot is given by:

\ w 1

m
- o IP- K ---------------^

s (s + 2) c% )

A. 10 B. 1 C. 5 D. 100

15. What is the flux crosses the closed surface which contains a charge distribution in the
sin <j)
form of a plane disk of radius 4m with a density p s Coul/m ?
2r

A. 2/71 Coul C. n Coul


B. tc/2 Coul D. 2n Coul
16. Find the charge, given charge density p v = x y 2e 2~ mCoul/m 3 of a cube with sides 2
jtiplaced in the rectangular coordinate axis with one of its endpoint at the origin.
A. 2.62 mCoul C. 4.26 mCoul
B. 6.42 mCoul D. 8.42 mCoul

17. The electric field between two concentric cylindrical conductors at r = 0.01 m and
r = 0.05 m is given by E = (105/r) a r (Volts/m), fringing neglected. Find the energy
stored in a 0.5 m length. Assume free space.
A. 0.224 Joule C. 224 Joules
B. 2.25 Joules D. 0.0224 Joule

18. A long straight steel metal rod has a radius of 5 cm and a line charge density of
30 nCoul/m. Find the electric field 100 cm from the axis of rod. The distance measured
is perpendicular to the axial rod.
A. 539.5 V/m C. 107.9 V/m
B. 567.9 V/m D. 540.9 V/m

19. A certain electromagnetic wave traveling in sea water was observed to have an
amplitude of 98.02 V/m at a depth of 10 m, an amplitude of 81.87 V/m at a depth of
100m. What is the attenuation of the sea water in neper/m?
A. 0.003 C. 0.001
B. 0.002 D. 0.004

20. A wave radiated by a source in air is incident upon a soil surface where upon a part of
the wave is transmitted in the soil medium. If the wavelength is 60 cm to 30 cm. What is
the soil permittivity? Assume soil to be a very lossless medium.
A. 6 C. 4
B. 10 D. 5
6
21. Calculate the wave velocity for a conducting medium in which conductivity a = 58 x10
mho/meter, pr = 1, at a frequency of f= 100 MHz.
A. 4152.3 m/sec C. 5243.1 m/sec
B. 4521.3 m/sec D. 5432.1 m/sec

22. A 50-ohm load is being fed from a 72-ohm transmission line. W hat is the reflection
coefficient resulting from this mismatch?
A. 0.018 0 . 0.180
B. 0.694 D. 1.440

23. If an SWR (Standing Wave Ratio) of 4:1 exists on a transmission line, what is
respective the percentage reflected and absorbed power by the load?
A. 36, 64 C. 33.33, 66.67
B. 64, 36 D. 60, 40

24. An antenna 0.02 sq. meter area operates 300 MHz at 0.3 mV per km. What is its
magnetic field intensity, H?
A. 0.6 nA/m _ C. 1.0nA/m
B. 2 nA/m D. 0.8 nA/m

25. What is the electric field intensity of an omni-directional antenna at 1 km distance if the
transmitter power is 1 kW? (77 = 1 20 ^)
A. 150.8 C.. 173.2
B. 135.2 D 120.5
Rev.Probs. Sept. 2073
CONTROL SYSTEMS & ELECTROMAGNETICS
SUPPLEMENTARY PROBLEMS
1. Is an algebraic or transcendental equality which involves more than one value of the dependent
variable corresponding to more than one value of at least one of the independent variable(s).
A) differential equation C. linear equation
B difference equation D. quadratic equation

2. Is a short-hand, pictorial representation of the cause-and-effect relationship between the input


and output of a physical system.
A. flow chart C block diagram
B. bar chart D. signal flow graph

3. Due to which of the following reason excessive bandwidth in control system should be avoided?
A. noise is proportionalto band width C. it leads to a low relative stability
B. it leads to slow speed response D. has no considerable effect

4. Is another method for determining whether all the roots of the characteristic equation of a
continuous system have negative real parts.
A. Routh criteria C. continued fraction criteria
B. Hurwitz criteria D. locus of point

5. Find the initial value of the function / ( / ) whose Laplace is: F(s) = —------- t------------
■ 7 5 + 2s +9.y + 6
A. 1 C.) 0
B. 2 DT 1/a

6. Determine if the characteristic equation given represents a stable or an unstable system:


s3 + 8 s 2 +145 + 24 = 0
A. stable system C. undefined
Bf unstable system D. indeterminate

7. What restrictions must be placed upon the parameter “K” in order to insure that the system is
stable from the characteristic equation given:
s4 + 6s3 + l l s 2 +6s + K = 0
A. K>10, K<60 C. K>10, K<11
B K< 10, K>Os D. K<60, K>11

8. What is the transfer function of a system whose input and output are related by the following
differential equation: y"+3y'+2y = u'+u
5+1 „ 5+4 5 -1 _ 5+3
A. —----------- B. —----------------------------C. ^ -------- D. —-------------
5 +35 + 2 5 +35 + 4 s~ — 3 5 - 2 5 " + 4s+ 5

9. Given y ”+3y'+2y = u , with initial condition y(o ) = 0, jv'(o) = 1, find the free response
A. e~‘ - e - 2' B. - e " + e '2'C. e~'+e~2' D. - e " - e ~ 2‘

10. From the differential equation of prob. #9, find the force response if u(t) = 1
I
A. 1—2e-' + e ~ 2‘ C. i(l-2(?w + ^ 2')
2

B. I ( l - e- ' + ^ ' ) D. H \ - 2 e - ‘ +2e-2‘)


11. Determine the damped natural frequency 0)d, damping coefficient a respectively for the

following second order: - r f !0


> + 4 ^ + s^ = 8 „
dt2 dt
A. 2,1.5 C. 3,0.5
B. V2 , 1.25 D. V 3 . 1
12. Determine the damping ratio C, , undamped natural frequency a>n respectively for the following

d 2y , ,d y
second order: + 5 — + 9y = 9u
dt2 dt
A. 2/3, 2 rad/sec C. 5/6, 3 rad/sec
B. 4/5, 2.5 rad/sec D. 1/3, 3.5 rad/sec
C.
13 Find the transfer function C (.s)//?(s) for the system shown.

G,G2 G, - G2
A. C.
1+ G 2H , + G ,//, 1 - G , / / , + G,

g l + ^2 G,G2
\ - G xH , - G 2//, I - G, 7/1 - C 2//,
14. Find the output J ^ o f a system described by the differential equation: y"+3y’+2y = 1 + /
with initial conditions ,y(0) = o, y ( o ) = i
l& r' - l e ~ 2' + 2 t - \ \ C. 1/4[8ew + 3 e"2' -2 /-lj
[8ew + 7e~2' +2/ + lJ D 1/4[8ew - 5 e - 2‘ - 2 t - \ \

15. From the differential equation of prob. #11, find the transient response.
A. l/4 [8 e "'-2 r-lj C. 1 / 4 ^ ^ + 7e"2( J
B. 1/4[2/ - 1] D. 1/4 [8e~' - 7e~21j
16. Given the following close-loop system: Find the value of K for the which the system becomes
marginally stable and the frequency of oscillation respectively when K is set to make the system
marginally stable. Hint: apply Routh-Hurwitz criterion

1
K ------------- ^ f — ^
s (s + 2) (s 4* 10)
R(sy A -.

A. 10, 10 Hz C. 100, 1.2 Hz


B. 240, 0.71 Hz D. 60, 2 Hz

1 1 >3 / — ~~ ✓ IVJLkLXu
17. Find the force on a +100 |iCoulfcharge at (0, 0, 3)m, if four like charges of +20 (iCoul are located
on the x and y axes ± 4mr
A. 1.73 az Newton C. 3.71 az Newton
B. 0.72 az Newton D. 2.73 az Newton

18. Two point charges, Qi= 5 0 / / C and Q2= 1 0 / / C are located at (-1, 1, -3) m and (3, 0, 1) m
respectively. Find the force on Qv
A. 0.144 ax - 0.108 az Newton C. -0.144 ax - 0.108 az Newton
B. 0.144 ay + 0.108 az Newton D. 0.144 ay - 0.108 az Newton

19. Find the charge defined in the volume defined 0 < x < 1 m, 0 < < 1m, & 0 < z < 1 m, i f :
py ~ 30x 2y p Coul/m3
A. 4 pC C. SpC
B. 6 pC D. 8 pC

20. Find the charge in the volume defined by 1 < r < 2m in spherical coordinates i f :
5 cos2 <j> „ ,, 3
. p v = ------t Coul/m
r
A. 10 n Coul C. 5 n Coul
B. 5I n Coul D. 10/^CqjuI

21. For a line charge p t - 0.5 nCoul/m on the z-axis. Find VAB , where A (2m, nl2, 0) and
B (4m, n, 5m)
A. 6.24 volts C. 2.46 volts
B. 4.26 volts D. 4.62 volts

22. An electromagnetic wave in free space has a wavelength of 0.20 m. When this same wave
enters a perfect dielectric, the wavelength changes to 0.09 m. Assuming that (jr = 1, determine
the relative permittivity of the dielectric.
A 4.94 C. 5.94
B. 4.49 D. 6.54

23. Calculate the attenuation factor for a conducting medium in which 8 = 58 x106 mho/m, = 1.0,
sr= 1.0 at a frequency of 100 MHz in neper /m.
A. 151,319 neper/m C. 315,915 neper/m
^*"115,391 neper/m D. 319,515 neper/m

24. A transmission line having characteristic impedance of 75 ohms is delivering power to a 300-
ohm load. What is its Voltage Standing Wave Ratio j(§WR)?
A. 1:4 C. ;4:1
B. 1:2 D '2:1

25. If an SWR (Standing Wave Ratio) of 4:1 exists on a transmission line, its reflection coefficient is

A. 0.6 C. 0.8
B. 1.67 D. 0.25

P ractice m a k e s p e rfe c t !

SuppLProb Sept 2013


DC CIRCUITS
This Free Quality Manual is
Found only at POWERLINE

POWERLINE REVIEW CENTER


M E Z Z A N I N E F L O O R , D O N A A M P A R O B U IL D IN G
C O R N E R E S P A N A & G. M. T O L E N T I N O S T R E E T S
S A M P A L O C , M A N IL A
TEL. NOS. 7 3 5 -7 3 -0 2 & 7 3 3 -2 1 -1 8

CEERS-
TEL. NOS. (0 3 )2 6 1 -2 2 4 4 & (0 3 2 )2 6 1 -8 4 5 2
POWERLINE ENGINEERING REVIEW CENTER, INC.
DIRECT CURRENT CIRCUITS :
P A R T - I
RESISTANCE /NETW ORK THEOREM S

RESISTANCE ( R )•. It may be defined as that property o f a substance or material which opposes the
flow o f electron or current.

The Resistance o f a conductor is given by:

D= p — ^ = /»- ^
1 =P—
R
A V r A2
Where : I = length o f the conductor in m or ft.
A = cross sectional area in m 2 , sq. mils or circular mils (CM).

A = —d 2 sq. mils ; A = d 2 CM ; if cl = diameter o f the conductor in mils.


4

NOTE : 1 inch = 1000 mils , 1 sq. inch = 10 6 sq. mils , 1 sq. inch = — x 10 6 CM
n
p = specific resistance or resistivity o f the material o f the conductor in Q-m or Q-CM/ft.
= 1.77 x 10 8 Q-m =10.66 Q -C M / ft => for hard-drawn Copper (HDC) @ 20 °C
= 1.72 x 10 ~8 Q-m =10.50 Q -C M / ft => for annealed Copper @ 2C °C
= 2.83 x 10 "“ Q-m = 17.10 Q -C M / ft for Aluminum @ 20 °C
V = volume o f conductor in m 3 or ft3.

The Variation o f Resistance with Temperature is given by :


R2 = /? ,[l + a, {t2 - /,)]
Where : R2 = resistance @ t2 , final value o f resistance in Q
R} = resistance @ f , , initial value o f resistance in Q
ti t t2 ~ initial and final temperature in °C respectively,
o', =temperature coefficient @ f, in °C 1

a\ = 7— -— ; T - —— = inferred absolute zero resistance temperature in °C.


|r| + <i a0
T = -234.5 °C for annealed copper, T = -2 4 2 °C for HDC, T - - 2 3 6 °C for aluminum.

O H M 'S L A W : it is stated that the ratio o f potential difference (V ) between any two points on a conductor to
the current (I) flowing between them, is constant provided the temperature o f the conductor
V V
does not change. i.e. — = constant or — = R

RESISTANCE in SERIES : RESISTANCE in PAR ALLEL :

*t - r >+r w i =i +i +i +-+t
y.r = v , + y ,+ V j+ - - + y„ v r = Vi = K ,= V, =•■• v„
I, = / , - h ' = In h = h 1 h H j '••• 1 f«

- L = — H— ^ i — h ... h— — Gr — G/ + G> + G; + +. G„
gt g, g2 g3 g„
where : G = conductance in mho Q
W O R K , PO W E R and E N E R G Y :

J o u le ’s Law o f Electric Heating is the amount work required to maintain a current I amperes through a
resistance R ohms fo r t seconds is given b y :

W = I2 Rt = Vlt =

This work is converted into heat and is dissipated away, the amount o f heat produced is given by :

Q = ------- in kcal or BTU


J
Where : J = mechanical equivalent o f heat =4.186 KJ / kcal = 778.16 ft-ib/ BTU
NOTE : 1 Joule = 10 7 ergs = 0.24 gm-calorie , 1 erg = 1 dyne -cm

T H E R M A L E F F IC IE N C Y :

it is the ratio o f the heat actually utilized to the total heat produced electrically, Hence, the relation
between heat produced electrically and the heat absorbed usefully becomes :

where : 7 = thermal efficiency, Cp = specific heat constant in c a l./g m -° C


m = mass in gm., AT = change in Temperature in °C

NET W O R K DEFINITIONS :

CIRCUIT- is a conducting path through which either an electric current flows or is intended to flow.
PARAMETERS - it is various elements o f an electric circuits such as Resistance', Inductance and
capacitance. These may be Lumped or Distributed.
LINEAR CIRCUIT - a circuit one whose parameters are constant
i.e. do not change with voltage or current.
NON-LINEAR CIRCUIT- a circuit one whose parameters change with voltage and current.
UNILATERAL CIRCUIT- it is that circuit whose properties changes with the direction o f its
operations. A vacuum tube rectifier is a unilateral circuit.
BILATERAL CIRCUIT - a circuit having identical properties in either direction. Transmission line
is a bilateral circuit.
ACTIVE NETWORK - a network containing one or more sources o f e.m.f.
PASSIVE NETWORK - a network containing no source o f e.m.f.
IDEAL CONSTANT- VOLTAGE SOURCE - is that voltage source whose output voltage remains
absolutely constant whatever the change in load current. Such a voltage source
must possess zero resistance so that internal voltage drop in the source is zero.
IDEAL CONSTANT- CURRENT SOURCE - is that voltage source whose internal resistance is
infinity. In practice, it is approached by a source which possesses very high
resistance as compared to that o f the external load resistance.

F A K A D A Y ’ S L A W O F E LE C T R O L Y SIS:

where : m = mass o f ions liberated in gins. z = electro-chemical equivalent wt.in gm / coul.


a = atomic svt. o f the substance. v = valency o f the substance.

F = Faraday's number = ----- Chemical^— equivalent— _ Coul/ gm - » equivalent

Electrochemical equivalent
P A R T - II
MAGNETIC FIELD & INDUCTANCE:
MAGNETIC FIELD : exists in a region o f space if a moving charge there experiences a force (other than
Friction ) due to its motion.

The Magnitude o f F orce is given by :


F = qvB sin 9 in Newton
where : q = the charge magnitude in coul. ( C ), v = the magnitude o f the velocity in m/sec.
0 - the angle between the field lines and the velocity.
B = magnetic flux density or magnetic induction in Tesla ( Wb/m 2 )
NOTE : 1 Tesla = 10 4 Gauss = 10 4 lines/cm 2 = 1 N/A-m

F orce on a Current in a M aenetic Field is :


In metric : F - BlLsmO in Newton
where : B = magnetic induction, Tesla / = current in Amp
L = length o f conductor, m 8 - angle between the direction o f / and B.
lill.sinO
In COS /' i n dynes
10
where : B ~ in Gauss, / = in Amp , L = in cm
, „ c BILsinO .
In English : F = --------------- m lbs.
6 11,300,000
where : B = in lines/in2, / = in Amp, L= in inches

Torque on a Flat Coil in a Uniform M aenetic Field is :


t = NIAB sin 6 in N-m
where : A/= number o f loops or turns, A = area o f the coil, m2
0 ~ angle between B and perpendicular to the plane o f the coil
NOTE : To determine the direction o f rotation o f the coil use Right - Harnl Rule.

M aenetic Field on a L on s straight wire :

B= where : r is the distance to a point from the axis o f the wire.


2tt7'

Maenetic Field on Center o f a Circular coil with N toons'.

B= where : a is the radius o f the circular coil.


2a

Maenetic Field in the Interior point o f a L on e Solenoid :


N
B = / / Q» / where : n is the number of turns per meter - —

Magnetic Field in the interior point o f Toroid with N h o p s :

5 - where : r is the radius o f the circle or, which a point lies.


2m

Force between Two Parallel Conductors :

f - s 2 x 10~7 2-~ where : i is the length o f the conductor, and


2nd d
d is the distance between conductors
Coulomb '.s' Law o f Magnetic Forces :
MMtM 2
F = ----------------- -— where : M, & M2 are pole strength in Wb., and
4^p //r r 2
r is the distance between the poles, m.

Faraday's Law o f Electromagnetic Induction (ea t’us. o f Induced em f):


A<j> Ai Aq
H= N - = R = B fv x 10~s in volts
At At At

INDUCTANCE (L) : is the property o f A C circuits which opposes any change in the amount o f current,
Its unit is Henry.
Expression o f S elf Inductance :
, N<p HrVoAN2 .
I - _ r _ = ... r »------- ln Henry
I I
where : A =is the area o f cross section o f the core in m\ <p= is the magnetic flux in Wb,
t =is the length o f the core in m (ir = is the relative permeability o f the core

Expression o f Mutual Inductance:

where : k = coefficient o f coupling N\, N2 = respective number o f turns


Lu!,2 = respective self- inductances o f the two coils.

Inductances in Series : Inductances in P arallel:


When M assists Lt & Li When M assists L, & L 2
L .L t-M 2
L — Li 4- L-> ■+■2.M
L, + L2 - 2 M
When M opposes L| & L2 When M opposes L, & L2
L,Z-2 - M 1
L = L, + L-, - 2M

O hm ’s Law o f Magnetic Circuits :


= = in Weber
9? <n t/ A p
/H r!* O'*
Where : 9? = is the Reluctance o f the magnetic circuit, it is reciprocal o f Permeance
Energy Stored in a Magnetic Field:

W , = — L I 2 in Joules
L 2

Energy Stored in a Magnetic Field per unit Volume:


B^
jp _ ---------- jn Joules/ m 3
Ij'oMr

Steinmett’s Emperical Law for Hysteresis loss:


Wh =T]B,ltffi> in watts
Where : r| = Steinmetz’ s coefficient in J/m 3 . / = frequency in cycles/sec (Hz)
Bm- maximum flux density in W b/m 2 v = volume o f the core in m3
P A R T - III
ELECTROSTATICS, CAPACITANCE <&
INSTRUMENTATIONS :

ELECTROSTATICS : is the study o f the behavior o f static-electricity (i.e. electric charges which are at rest).

Coulomb ’.v Law o f Electrostatics :

F = ----- ------= 9x10* ^ 1 % in Newton


4w,|£r (I2 e ril
Where : F = the force attraction or repulsion between two charges.
Q u Q i = are the respective charges in coulomb (C) NOTE : 1 Statcoul. = 3.3356 x 10 _l5
Coul.
d = distance between the two charges in meters (m)
e Q= permitivity o f free space or vacuum = 8.854 x 10 ~12 F/m
e , = relative permitivity o f the medium = 1.0 ( f o r air)

Electric Field Strength at any point within an Electric Field :

E = — = 9 x l0 9 in N/C or Volts/m
- Q e rd 2

Electric Potential at a point lying in an Electric F ie ld :

V = -H'° — = 9x10 9 -Q— in J/C or Volts


Q crd

Electric Flux Density o f an Electric Field :

D =—= E in C/m 2
A
Where : \p= electric flux in Coul = Q ( according to Gauss’ s Law )
A = area at which electric flux is passing normally in m2

CAPACITANCE (C) : is the property o f AC circuits which opposes any change in the amount o f voltage. Also
defined as the charge required to create a unit potential difference between its two plates.

Capacitance o f a Capacitor is :
Q in Coul./Volt or Farad
C=—
v

Capacitance o f an Isolated Sphere :


C = 4n£0£ rr in Farad
Where : r = is the radius o f isolated sphere in a medium o f relative permitivity e r , in m

Capacitance o f Concentric Sphere ;

(i) with outer sphere earthed : C = 47re0e r -------- in Farad


b -a
(ii) with inner sphere earthed : C = 4rte0e r ------- in Farad
b -a
W here: a and b = are the radii o f two concentric sphere in m.
£ r —the relative permitivity o f the dielectric between two spheres.
Capacitance o f Parallel Plate Capacitor :
A
(a) Uniform dielectric medium : C = £0£r — in Farad
d
Where : A = area o f each plate in m 3 , d - thickness o f dielectric medium in m.
s , = relative permitivity o f the medium.

£
(b) Composite dielectric medium : C = --------------------------------------- =r in Farad
4-^2 , ^3
--------j---------- 1---------- h. o o o -j----—
£rl e rl e r) Srn
Where : d\ , d2 , d^, . . . d„ ~ are thickness o f dielectric medium with relative permitivity
o f Sri, £,2 , , •••Sm respectively.

6A
(c) Dielectric medium partly air : C = -------f—-------— in Farad
t
d - t - —-
- \ £r _
Where : d = air gap in m , t —thickness o f dielectric medium in m

Capacitance o f Multiple Plate Capacitor :

C ~ {n —l ) -— in Farad
d
Where : n = number o f parallel plates, A = area o f each plate inm 2
d = separation between two plates in m, £ , = relative permitivity o f medium

Capacitance o f Cylindrical Capacitor :


C= in Farad
In —
n
Where : ra , = outside and inside radii resp. in m, I = length o f the cable in m

Capacitance in Series ; Capacitance in P arallel:

= ----4-------- 1------4-ooo + ----- C. —Ct + C t 4- C-i + 0 0 0 + C


c, c, c2 c3 c„ 2
Kf = K , + V 2 + V 3 + o o o + K„ V, = K, = K j = o o o K „

Q, =02 =03 = °°°Q» Q, =Q\+Q.2 +03 +°°° + Q„

Energy Stored in a Capacitor :

w c = - C V 2 = i QV = -^1 i„ Joules
C 2 2 2C

Energy Stored per volume o f a dielectric medium :


D2
Wr = ---------- in Joule/m 3
2£{)£r '

F oree o f Attraction between two plates o f parallel plate capacitor :


D2 e 0e r E 2 . 2
F = ---------= ------------ in N/m
2e 0c r 2
E L E C T R IC I N S T R U M E N T S
2 C l a s s e s o f E le c t r ic I n s t r u m e n t s
1. Absolute - can indicate the presence o f an electric quantity. No calibration or comparison is
necessary, (e.g. tangent galvanometer)
2 Secondary - an instrument in which the value o f electrical quantity to be measured can be
determined from the instruments, only when they have been pre-calibrated by
comparison with absolute instruments

T y p e s o f S e c o n d a ry In stru m e n ts
1 Indicating instruments - displays the instantaneous values o f electrical quantity on a calibrated
scale, (e.g. voltmeter, ammeter, wattmeter etc.)
2 Recording instruments - these give a continuous record o f variations o f an electrical quantity
with respect to time. (e.g. load or demand graph, recording wattmeter e tc )
3. Integrating instruments - these measure and register by a set o f dials and pointers, (e.g. watt-hour
meter, Amp-hour etc.)

T o rq u e s o n M o v in g S y stem s
1. Deflecting (operating) torque - the force developed in an instrument which moves the moving
system o f an instrument in accordance with the magnitude o f the quantity to
be measured.
2 . Controlling (restoring) torque - the force which controls the deflection o f the pointer o f an
instruments and which brings the pointer back to zero. Controlling torque
may be provided by spring control, gravity control.
3. Damping torque - torque which stops the oscillations o f the pointer o f an instrument. Damping
can be produced by air friction, fluid friction, eddy currents

M o v in g C o il In stru m e n t
It works on the principle o f a o f a dc motor, it has a uniform scale, and a sensitive instrument,
sometimes called “ D ’ Arsonval” instrument It is only used for dc measurement.

M o v in g Iro n In stru m e n t
It has no moving coil but which has a moving iron strip or disk it works in the principle o f attraction
and repulsion. It can be used both for ac and dc measurements.

D y n a m o m e te r In stru m e n t
These instrument are based on the principle that mechanical force exists between the current carrying
conductors. It is essentially consists o f a fixed and moving coil, these coils may be excited separately
or they may be connected in series. It can be used both for ac and dc measurements.

In d u c tio n ty p e In stru m e n t
The principle o f operation is similar to that o f induction motor. A rotating magnetic field is set up by
the suitably located coils o f the instrument. An aluminum disc is suspended near to the coils in which
eddy currents are induced by the rotating flux which tends to rotate the disc. It is only used for ac
circuits.

T h e r m o -c o u p le In s tr u m e n t
These are based on Seebeck effect (thermoelectric effect) The thermo em f is proportional to the
difference in temperature between hot and cold junction. These can be used for ac/dc measurement.

H o t w ir e In s tr u m e n t
It is an ac/dc current measuring instrument which is based on the heating effect o f electric current. It
consists o f platinum-iridium (it can withstand oxidation at high temperatures) wire which expands
when heated which provide deflection o f the pointer.

E le c t r o s t a t i c I n s t r u m e n t
The basic principle o f such instrument is that a force o f attraction exists between two or more charged
bodies. So these are basically a voltage measuring devices.
POWERLINE r e v i e w c e n t e r Inc.
TH« Ultimate E.E. Review Center
DC CIRCUITS REVIEW PROBLEM S/QUESTIONS

. The presence of electric current is only made known by the effect it produces. Three
important effects are:
^H^eating, electric shock & generation C. generation, chemical & electric shock
y B . Seating, magnetic & electric shock D. heating, magnetic & chemical

. Out of the following, select the best conductor of electricity?


A. Polyethylene C. China Clay
B. Ceramic Graphite

. Which of the following conductors has the highest resistance? -a ^ ^


A. Manganin w . H3 yc/a' 9Jtn C. Constantan &
/%>)Nichrome /O0,5 * io~8 -fc-r* D. Steel 7o = <r. * (0

What is the size in square millimeter


er is the cable 250 MCM size? . , \v
.656 ----- 1 —- M cM tf CM C.
C. 112.565
112.565
( j j * 1126.
18-
675 ~ D - 132 348 ' ,

A copper bar is 1/4 inch by 4 inches by 20 feet, if the resistivity of the copper bar is 10.37
ohms-cmf. What is the resistance of the copper bar? p ~ J)
A. 17.3x1 O^ohm C. 0.0052 ohm Jr 4 '
B. 125 ohms (^57^8.15 x10~5ohm ^

. A one-meter rod of 2 cm diameter is drawn until its resistance is 100 times the initial
distance, its length afterward is?
10 m C. 100 m
12.5 m D. 5 m

A one-meter of annealed copper 2.5 cm in diameter is drawn until its resistance is 100
les the initial resistance, its diameter afterward is?
0.79 cm C. 0.25 cm
0.47 cm D. 0.15 cm

. Suppose that you wish to fabricate a uniform wire out of 1.0 gram of copper. The
resistance of wire is to be 0.5 ohm and if all of the copper are to be used, what will be
the diameter of wire? Specific resistance is 1.7 x1 O'8ohm-meter and mass density is
8.92 x103kg/m3. w
C. 260 R =y. V ‘ /o
(3 ) 280 pnv-" ^ D. 290 pm r

A dc motor 120 Volts has 100 Amp rated current. The allowable voltage drop of 3% of
input voltage. Find the maximum distance from the supply if the wire has a diameter of
316 mils and resistivity of wire is 10.4 ohm-cmf~_
A. 508 ft © 1 7 8 ft \/ & |
B. 316 ft y v . 203 ft ^ c~^"
10. A light bulb having a tungsten filament draws 0.5 Amp at 110 Volts. The cold resistance
of the filament is 20 ohms at 20°C with temperature coefficient of resistance for
tungsten at this temperature of 0.005 per °C. Determine the operating temperature of
the bulb. rj , ., ,T -r- -■
A. 1220°C r'^>~ C?520° C
B. 520°C ■"■■■ iV-- ^J2 0 2 0 °C

11. A 100-Volt carbon filament lamp takes 1 Amp when glowing with a filament temperature
of 1615°C. Calculate the momentary current when the lamp is first switched on in air
temperature of 15°C. The temperature coefficient of resistance for carbon may be taken
as -0.000265 per °C at 15°C.
A. 1.72Amp *r =• (^ )0 .5 8 A m p A
B. 0.65 Amp cT 1.94 Amp

12. Find the work in kWh to lift 1 ton at a height of 55 ft for 10 sec. The motor gear
efficiency is 51%.
Q . 0.083 C. 0.83
B. 83 v “ D. 830

13. A 24 Volts battery delivers 15 Amp to a motor load in 30 minutes per day. What is the
energy delivered per month in kWh?
A. 6.2 C. 7.31
® 5 . 4 (j j ^ \ K . t D 018

14. A certain process uses warm water at rate of 2 liters per minute. The water enters at
electric heater at 25 deg C to 50 deg C. Assuming no heat losses, what is the wattage
of the heater?
A. 5125 ■- — C. 2420
B. 6250 3488

15 J t is a cell designed to produce electric current and can be recharged.


Secondary cell C. Electrolytic cell
B. Chemical cell D. Battery

16. What is the watt-hour efficiency of storage battery?


AH output over WH input in recharge
>WH output based on WH input in recharge
C. WH output based on AH input in recharge
D. AH output based on AH output in recharge

17. A 12-Volt battery has 50 Amp-hr capacity. The internal resistance is 0.1 ohm. A 5-ohm
resistor is connected for 5 hours. How many Amp-hours are still left?
0 . 38.23 C. 41.24
B. 3 D. 11.76

18. A 20-ohm resistor is connected in parallel with avariable resistor R The parallel
combination is then connected in series with a 4-ohm resistor and connectedacross a
240 V source. Determine the minimum value of R if the power of R is equal to the power
taken by the 4-ohm resistor.
A. 7.95 ohms ( c ) 7.64 ohms
B. 8.40 ohms IX 6.87 ohms
19. Twelve resistances 1 ohm each are arranged along the edge of a cube with junctions at
comers. What resistance will be measured between terminals of symmetrically apposite
corners of the cube?
A. 6/5 ohm C. 5/9 ohm
5/6 ohm D. 6/111 ohm

20. What is the power absorbed by 900 ohm resistor in the network shown below (Fig,1)7
A. 1.44 Watts C. 5.43 Watts
B. 3.52 Watts D. 4.84 Watts

0.1 Amp

21. Two batteries are connected in parallel supply a load taking 60 Watts. First battery has
10.5Volt emf and 0.5 ohm internal resistance while second has 9 Volt emf and 0.4 ohm
internal resistance. Find the resistance of the load.
A. 1.333 ohms f/ C . 1.067 ohms
B, 1.444 ohms ~tf. 1.557 ohms

22. From the given network shown in Fig. 2, determine the current supplied by the 12 Volt
battery and the voltage Vo.
A. 1.5 Amp, 12.2 Volts .C. -1.5 Amp, 8.8 Volts
B. -2.2 Amp, -2.2 Volts D. 1.9 Amp, -8.8 Volts

‘Cft. \ i
:3. Find the current in the 3 ohm resistor of the circuit of Fig. 3 by Thevenin’s theorem.
A. 9 Amp C. 12 Amp
B:> 5.143 Amp D. 6.143 Amp

3Q

4. Determine the total current supplied by a battery to the resistive network shown Fig. 4.
A. 0.5 Amp C. 0.2 Amp
B. 0.3 Amp D. 0.7 Amp

5. A lead storage battery is rated at 12 Volts. If the internal resistance is 0.01 ohm, What is
the maximum power that can be delivered to the load?
A. 1,200 Watts v Q j 3,600 Watts V £ P
B. 7,200 Watts D. 1,800 Watts

3. A steel sheet 1 m long and 30 cm wide is to be plated with 0.02 mm copper. The
density of copper is 8.9 gr/cm3 and its electrochemical equivalent weight is 0,3292x10~J
gm/Coul. How long should the steel sheet remain in an electroplating bath that contains
copper (Cu1-2) ions in which the current is 100 Amp?
A. 541 mins. C. 27 mins.
B. 145 mins. D. 54 mins.

1. In Flemming’s Right Hand Rule, which finger point the direction of current?
A. Thumb C.‘ Middle
B. Index D. Ring
i. A 120 cm long conductor is carrying a current of 1.2 Amp and is situated at right angles
to the field of flux density of 0.65 Tesla. Calculate the force on the conductor.
A. 3.264 N C. 1.348 N
B, 0.936 N D. 1.587 N

>. A core of annealed steel sheet is wound with 1500 turns of wire through which a current
of 40 mA is flowing. If the length of the coil is 20 cm, calculate the magnetic strength is
Amp-turns per meter.
A. 300 C. 400
B. 350 D. 450

). A flat circular coil having 50 loops of wire on it has a diameter of 48 cm. What current
must flow in its wires to produce a flux density of 0.005 Tesla at its center?
A. 45.5 Amp C. 32.8 Amp
B, 38.2 Amp D. 40.5 Amp

1. A substance or material having relative permeability of less than unity is classified as:
A. ferro-magnetic C. para-magnetic . .. ;
B. dia-magnetic D. none of these
J <’*» i
2. Permeability in a magnetic circuit corresponds to ______in an electric circuit.
A. resistance C. jconductivity
B. resistivity D. conductance

3. A solenoid has a magnetic reluctance of 2.2 x10~3. It has 300 turns and a core area of 5
sq. cm. What is the flux density when the current flowing is 1 Amp?
A. 26,300 Gauss C. 34,200 Gauss
B. 12,200 Gauss D. 21,200 Gauss

4. A magnetic circuit consists of silicon steel of 3000 permeability and an air gap. The
length of the steel core is 10 cm and the air gap is 2 cm both have the same cross-
section of 1.5 sq. cm. A current of V* Amp flows through the windings to produce 2351
Maxwell flux. How many turns are there in the coil?
A. 4,120 turns C. 2,500 turns
B, 500 turns D. 1,250 turns
'N * , ,
5. A magnetic circuit consists of silicon steel 3000 permeability of 10 cm length and cross
section of 1.5 sq.cm and an air gap of the same cross section and of 2 cm length. A !4
Amp flows through 500 turns. What is the field intensity at the air gap?
A. 250 C. 795
-B. 2,262 D. 1,567

16. Pole strength 160 and 192 are separated by a distance has a force of 19600 dynes.
What is the distance in cm? integer choices.
A) 1 ' C. 3
B. 2 D. 4

57. A current of 2 Amp through a coil sets up flux linkages- of 4 Weber-turns. What is the
inductance of the coil?
A. 8H C)2H
B. 0.5 H D. 1 H
a

38. The direction of induced e.m.f can be found by______.


A. Laplace’slaw C. Fleming’s right hand rule
Lenz’s law D. Kirchhoff s voltage law

39. A magnetic coil produces 100,000 maxwells with 2,000 turns and with a current of 2
Amp. The current is cut-off and the flux collapses in 0.01 sec. What is the average
voltage that will appear across the coil?
A. 20 kV C. 200 V
B. 2,000 kV D. 2 kV

40. A 50 mH inductance carries a current of 10 Amp which reverses in 5 milliseconds. What


is the average voltage induced in the inductor because of this current reversal?
A. 100 Volts C. 400 Volts
B. ) 200 Volts D. 50 Volts

41. A metal transport plane has a wing spread of 88 ft. What difference of potential exists
between the extremities of the wings, when the plane moves horizontally with the speed
of 150 miles per hour? The value of the vertical-component of the earth’s magnetic field
is 0.65 Gauss at the plane.
A; 0.12 Volt . C. 12 Volt
B. 0.24 Volt D. 1.2 Volts

42. Two coils of inductance L| = 1.16 mH, L2 = 2 mH are connected in series. Find the total
energy stored when the steady current is 2 Amp.
A. 1.75 mJ C. 17.5 mJ
B.: 6.32 mJ D. 63.2 mJ

43. A non-magnetic ring having a mean diameter of 30 cm the cross-section area is 4 sq.
cm and uniformlywound with twoconductors A and B over one another. A has 90turns
and B has 240turns. Calculate the firstprinciple of mutualinductance between the
core.
A. 10.62 C) 11.52
B. 10.55 D. 11.00

44/When two inductors are connected in series aiding, their total inductance is 40 mH,
when connected series opposing the inductance is 17.5 mH. What is the value of
mutual inductance?
A. 0.5625 C. 56.25
B) 5.625 D. 562.5

45. Two coils of inductance 6 & 4 Henries are connected in parallel such that its coefficient
of coupling is 41% with mutual inductance opposes the self-inductance. What is the
total inductance of the combination?
A. 0.94 H C. 1.43 H
B. 3.33 H D. 0.49 H

46. When one coil of magnetically coupled pair has a current of 5 Amp, the resulting flux
0 n = 0.4 mWb and flux <t>12 = 0.8 mWb respectively. If the turns are Nt = 500 and
N2= 1500, Find U ■
A. 60 mH C,i120mH
B. 80 mH D. 100 mH
47. Equal amount of electrons are placed in two spheres 3 cm apart in air. If the resultant
force is 4 kN, how many electrons does each spheres contain?
A. 1.44 x1014 C. 1.25 x1014
B. 1.33 x1014 D. 1.75 x1014

48. Four 3 pCoul spheres are in corners of a square 1 cm on sides. Find the force in
Newton?
A. 8.1 x10-10 C. 4.05x10~10
B) 15.5x1 O'10 D. 20.25x1 O'10

49. Two similar charges of 250 state are srtuated on small spheres 15 cm apart. What is
the potential at point midway between the spheres?
J5 16.7 statV CJ 66.7 statV
B. 25 StatV D. 33.3 statV

50. A small sphere of mass 0.5 gram hangs by a thread between two parallel vertical plates
53 mm apart. If the charge on the sphere is 25 micro-Coul, What potential difference
between the plates will cause the thread to assume an angle of 30° with the vertical?
A. 24 Volts C. 16 Volts
B, 6 Volts D. 12 Volts

51. An isolated sphere 10 cm in radius is charged in air to 500 Volts. How much charge is
required?
A.) 5.563 nC C. 6.553 nC
B. 2.782 nC D. 8.227 nC

52. A capacitor with air between its plates has a capacitance of 3 ^F. What is the
capacitance when wax of dielectric constant 2.8 is between the plates?
A. 1.07 p.F d: 8.40 nF
B. 23.52 jiF D. 0.38 |aF

53. Find the capacitance of a two plate capacitor of plate area 12 sq. inch dielectric
constant of 6 and dielectric thickness of 0.01 inch.
A. 1.3 nF C. 1.2 nF
B. 1.6 nF D. 1.5 nF

54. A plate capacitor is made up of 501 sheets of aluminum 25 cm x 30 cm with a dielectric


of paraffin paper 0.0015 cm thick. What is the capacitance in micro-farads of the
condenser (K for paraffin paper is 2.3)
A. 35 C. 75
B. 25 D. 50

55. An electrolytic capacitor can be used for?


A. DC only C. DC as well as AC
B. AC only ^ D. none of these

56. Which of the following materials has the highest value of dielectric constant?
A. glass -+-» * 3 ceramics tt O C
B. vacuum ^ D. oil ^ x v ^ %^
57. The sparking between two electrical contacts can be reduced by inserting a ______.
A. capacitor in parallelwith contacts C. resistance in theline
B. capacitor in series with eachcontact D. inductor in parallel with contacts

58. Find the current in a conductor through which 2.5 x1021 electrons pass in 10 seconds.
'3 . 40 Amp C. 20 Amp
B. 80 Amp D. 10 Amp

59. A current of 5 milliamp charges an insulator for 3 sec. How much charge is
accumulated?
A. 0.015 Coul C. 0.0015 Coul
B. 0.15 Coul D. 1.5 Coul

60. A certain capacitor is charged at 48 Volts after which its energy is 5.76 x10 to negative
2 Joule. What is the capacitance of the capacitor?
A. 2 micro-farad C. 60 micro-farad
B. 30 micro-farad D. 50 micro-farad

61. Two capacitors 30 and 10 micro-farad are connected in series, what is the total
capacitance?
A. 40 micro-farad C. 20 micro-farad
B. 7.5 micro-farad D. 300 micro-farad

62. A 12 micro-farad capacitor charged at 500 Volts and a 6 micro-farad capacitor charged
at 250 Volts are connected in parallel. What is the potential difference in the
combination?
A. 500 Volts £ . 750 Volts
B. 450 Volts D. 417 Volts

63. A galvanometer has a resistance of 300 ohms. What is the ohmic resistance to make it
read one is to ten?
A. 100 C. 33.33
B. 10 D. 30

64. A 100,000-ohm voltmeter is used in measuring the insulation resistance of a motor.


When connected in series with the insulation across a 250-Volt line, the instrument
reads 1 Volt. What is the insulation resistance of the motor?
A. 25.1 MQ C. 24.9 MQ
B. 18.7 MQ Cr;"29.4MQ

65. To roughly check his kWh meter an electrician hook-up a wattmeter, he switched on the
load, he counted 5 rev in 18 sec, the meter constant is 3.2. The wattmeter reads 368
Watts. What is the error of the meter?
A. 8% slower C. 8% faster % •,
B. 5% faster DJ5% slower

revprobs.5ept.2013
DC CIRCUITS SUPPLEMENTARY PROBLEMS
1. When an electron moves because of the pressure applied to it. What is the resulting
process?
A. melting C. generating
B. magnetic induction D. flow of current

2. What is the other term for American Wire Gauge?


A. JIS C. DIN
B, Brown & Sharpe D. VDE

3. In American wire gauge as the number gage increase the wire diameter?
A A.- increase C. does not change
B. decrease D. does change

4. In metric standard, as the number of gage increase, the size_____ .


A. do not change C. decrease
B. does change D. increase

5. Current has many applications in chemistry, which of the following is NOT one?
A. Electrolyticdiffusion C. Electroplating
B. Distillation D. Battery manufacture

6. What is the equivalent area in sq. inch for a conductor 336,400 circular mil?
A. 0.428 C. 0.678
B. 0.264 D. 0.768

7. A wire has a resistance of 0.75 ohm per ft at an area of 10“5 m2. Determine the
resistance of a 200 ft long conductor if the area is 3 x10"5m2.
A. 2.5 C. 50
B. 25 D. 12.5

8. The tungsten filament lamp has its filament wire of length 3.89 cm and 0.01 mm in
diameter. The resistivity is 5.55 x10“® ohm-meter at 20°C. What is the resistance at
20°C?
A. 15 ohms . C. 27.5 ohms
B. 20 ohms d . 60 ohms

9. A round aluminum bus bar having a cross-section of 0,185 sq. inch and a length of
2.956 ft. has a resistance of 207 micro-ohms at 20°C. What is the resistance in ohm per
circular mil foot at this temperature of the aluminum bus bar?
A, 18.56 C. 14.50
B. 16.49 D. 19.00

10. The power taken by a 500 ft resistive coil made of copper wire is 250 Watts at 110
Volts. Resistivity of copper is 10.4 ohm-cmil per ft. Calculate the cross sectional area of
the coil in circular mil?
A. 146 C; 107
B. 168 D. 175

11. Copper has a resistivity of 16 ohm-nm at zero degrees centigrade. What is its
temperature coefficient of resistance at zero degrees centigrade?
A. 3.93 x10“3 /°C C. 2.73 x10~3 l°C
B. 3.65 x10“3 /°C D.' 4.27 xIO"3 1°C
12. A copper wire has a resistance of 2.4 ohms at 20°C. Find the resistance at 75°C.
Resistance temperature coefficient at 20°C is 0.034/°C.
A. 3.78 ohms C. 6.89 ohms
B. 12.18 ohms D. 7.48 ohms

13. Calculate the voltage drop in voits of a 1000 ft AWG in gage wire connected to a 2 Amp
load (rho is 10.4 ohm-cmf, area is 10,380 cmil)
A. 2.4 C. 3
B. 2 D. 1.5

14. A dc motor 10 hp, 230 Voits has 0.85 efficiency. The allowable voltage drop of 10% of
input voltage. Find the area of wire in circular mil if the length is 150 ft and resistivity of
copper is 10.4 ohm-cmil per ft.
A. 3088 C. 5176
B. 6176 D. 2588

15. A dc motor draws 200 Amp is located 100 ft from the supply line. The wire has a
resistance of 0.01 ohm per 100 ft. What is the voltage at the motor terminals if the
supply voltage is 120 Volts?
A. 122 Volts C. 118 Volts
B. 124 Volts D> 116 Volts

16. A series loop contains the following circuit elements in order: an 8-Volt source, 2 kQ
resistor, a 3 kQ resistor, a 16-Volt source and a l kCl resistor. What is the voltage
across the 2 kQ resistor? Source voltage are additive.
A. 6 Volts C. 14 Volts
B. 8 Volts D. 4 Volts

17. Three resistance are connected in series has a total resistance of 120 ohms. If Rx = 20
ohms, and Ry is twice greater than Rz. Find Ry and Rz.
A. Ry = 50 ohms& Rz = 50 ohms C. Ry = 67 ohms & Rz =33 ohms
B. Ry = 60 ohms& Rz = 40 ohms D. Ry = 75 ohms & Rz =25 ohms

18. The resistance of a copper wire at 30°C is 50 ohms. If the temperature coefficient of
copper at 0°C is 0.00427°C~1. What is the resistance at 100°C?
A. 63.24 ohms C. 54.25 ohms
B. 72.26 ohms D. 58.15 ohms

19. The resistance of a wire is 126.48 ohms at 100°C and 100 ohms at 30°C. Determine the
temperature coefficient of copper at 0°C.
A. 0.00615 °C“1 C. 0.0256 °C'1
B. 0.00427 °C“1 D. 0.0356 °C"1

20. Three resistors are connected in series, the impressed voltage V = 240 Volts, the
resulting current i = 4 Amp, R, = 15 ohms, R2 = 1.5 R3) what is R3 in ohms?
A. 18 C. 27
B. 24 0 ^ 0 ■■ H U ’r . D. 16
Zkljo H (_ i“*> !<3 Pb )
21. How many erg per second are there in one Watt?
A. 10 exp 7 C. 10 exp 15
B. 10 exp 8 D. 10 exp 12
22. How much heat in kcal is produced by a current of 20 Amp flowing for one half hour in a
circuit having a resistance of 6 ohms?
A. 1032 C. 1016
B. 1088 D. 1066

23. A 100 Liters of water is heated from 20qC to 40°C. How many kWhr of electricity is
needed assuming no heat loss?
A. 5.6 C. 4.24
B. 2.3 D. 2.45

24. A 2.3 kWh is used to heat 100 liters of water. What is the change in temperature in
degrees farenheit?
A:' 36 C. 42
B. 80 D. 20

25. Power in Watts is required to melt 10 lbs ice in 5 minutes. (Latent heat of ice is 80
Calories per gram; 1 gram-Calorie is 4,184 Joules) What is the cost of operation if
power rate is Php 6.00/kWh?
A. 2.54 C. 3.50
B. 6.25 D. 3.05

26. Twenty four (24) dry cells are arranged in rows of six (6) in series and the four (4) rows
in parallel. The emf of each cell is 1,40 Volt, and internal resistance of each is 0.08
ohm. Determine the current to an external resistor of 3.2 Q.
N. 2.35 Amp C. 5.23 Amp
B. 5.32 Amp D. 2.53 Amp

27. A 25 ohm resistance and a 10 ohm resistance are connected in parallel. Then the two
are connected in series with a 5 ohms resistance. If the circuit is then connected across
a 50-Volt battery with internal resistance of 0.5 ohm What is the current flowing in the
25 ohm resistor?
A. 2.83 Amp C. 3.13 Amp
B. 1.13 Amp D. 3.96 Amp

28. Two resistances of 10 and 15 ohms, each connected in parallel. The two are then
connected in series with a 5-ohm resistance, if then connected across a 12 Volt battery.
What are the current and power?
A. 1.2 A, 17.28W C. 1.5 A, 20.25 W
B. 0.96 A, 11.52 W D. 1.09 A, 13.10 W

29. Two-24 Volt battery supply power to a 10 ohm load. One battery has 0.2 ohm interna!
resistance while the other has 0.4 ohm. What is the load voltage and power does this
load draws? '
A. 23.84 Volts and56.8 Watts C. 23.68 Volts and 56.1 Watts
B, 24 Volts and48.2 Watts D. 23.24 Volts and 60.8 Watts

30. A voltage source delivers 4 Amp when a load connected to it is 5 ohms and 2 Amp
when the load becomes 20 ohms. What maximum power which the source can supply?
A. 180 Watts C. 60 Watts
B. 90 Watts D. 360 Watts
31. Find the output voltage in the network below (Fig. 5).
A. 2 Volts C. 6 Volts
B. 4 Volts D. 8 Volts

Figure 5

32. Find the current flowing in the 4-ohm resistor in the network shown Fig.6.
A. -0.333 Amp C. 1.522 Amp
B. -2.33 Amp D. 1.567 Amp

80 2 Q

33. Calculate the Thevenin voltage & resistance at terminal “ab” of the circuit shown (Fig 7).
A. 33.75 Volts, 4 ohms C. 37.35 Volts, 8 ohms
B. 22.48 Volts, 10 ohms D. 28.24 Volts, 6 ohms
8Q c 6 £2 a

Figure 7

34. A coating of nickel 1mm thick is to be built on a steel cylindrical shaft 20 cm in diameter
and 30 cm in length in 2 hrs. Calculate the electrical energy used in the process if the
voltage is 10 Volts, electro-chemical equivalent of nickel is 0.304x10-3 gram per
Coulomb, specific gravity of nickel is 8.9.
A. 15.33 kWh C. 153.3 kWh
B. 13.53 kWh D. 135.3 kWh

35. A long straight conductor is carrying 100 Amp current. Determine the flux density at a
point 8 cm from the conductor. ^
A. 2.5x10"3 j£?0.25x1Q-3
B. 0.025 x10'3 D. 0.25

36. Flux of 14,000 lines has a flux density 7000 gauss, What is the area of core in sq. cm?
A. 1 C 2
B. 3 4

37. What is the magnetic field intensity in air at a point 2 cm from a long straight wire
carrying a current of 5 Amp?
A. 250 A/m C. 39.8 A/m
B. 25 A/m D. 48.5 A/m

38. A rectangular coil of 25 loops is suspended in a field of magnetic induction of 0.2 Tesla.
The plane of the coil is parallel to the direction of the field. The dimensions of the coil
are 15 cm perpendicular to the field lines and 12 cm parallel to them. What is the
current in the coils if there is a torque of 5.4 Newton-meter acting on it?
A. 30 Amp C. 40 Amp
Bj 60 Amp , D. 50 Amp

39. A solenoid has 250 turns. What is the mmf when-the current is 0.12 Amp?
A. 41.2 Gilberts C.. 37.7 Gilberts
B. 25.1 Gilb'erts D. 30.0 Gilberts

40. An iron ring electromagnet having a relative permeability of 1250 when it is excited by a
coil having 500 turns at 0.8 Amp. The mean length of iron parts is 48 cm, the cross
section of core is 10 cm2, and the air gap is 0.5 cm. What is its total reluctance?
A. 306x103 AT/Wb C> 4286x103AT/Wb
B. 3980x103AT/Wb D. 3674x103 AT/Wb
!

41. What is the heating process makes use of the currents induced by the eiectromagnetic
action in the charge to be heated, used for hardening of steel?
A. Induction heating C. Dielectric heating
B. Convection D. Infrared heating

42. The e.g.s. unrationalized unit of magnetomotive force is .


A. Oersted C. JGilberts
B. Maxwells D. Ampere-turn

43. An amperes is how many abamperes?


A. 0.4 C. 0.2
B. 0.3 D. 0.1

44. The relative permeability of a certain silicon steel is 4,500. A certain magnetic loop
consist of the a silicon steel of 10 cm square, 20 cm long and an air gap of % cm. What
is the reluctance of the magnetic circuit?
A. 6,44x10^ C. £.54x10~3
B. 6.44x10'3 D. 2.54 x10“2

45. A magnetomotive force is'supplied by a current of 1 Amp through 100 turns. The
magnetic circuit consists of a steel core of 1000 permeability, 10 cm long and 4 sq. cm.
area and an air gap of one cm long. What is the field intensity at the air gap?
A. 12.25 Oersted C. 100.53 Oersted
B. 497.66 Oersted D. 124.4 Oersted

46. An emf induced by the changing of current in the neighboring coil is called:
A. speed C. seif-induced
B. mutually induced D. series

47. A south pole of 35 nit pole is placed under a south pole of 20 unit pole is 1 inch apart.
What is the force between them?
A. 700 dynes repulsion C. 108.5 dynes attraction
B. 108.5 dynes repulsion D. 700 dynes attraction

48. The flux density emanating from a pole of a generator is 20,000 gauss. A conductor one
meter long cuts the flux perpendicularly at a speed of 40 m per second. What voltage is
developed ?
A. 80 Volts C. 230 Volts
B. 800 Volts D.8 Volts

49. How much energy is stored in the magnetic field of a 50-miiiihenry coil carrying a steady
current of 3 Amp?
A. 0.225 Joule C. 0.15 Joule
B. 0.45 Joule D. 0.75 Joule

50. Three inductors !_•, =20 mH, L2=20 mH and L3=40 mH connected in parallel. Determine
the equivalent inductance.
A. 8 mH C. 24 mH
B. 12 mH D. 36 mH
51. Two coils having inductance Li = 0.5H, L2 = 0.3H. At one instant, h = 12 A and i2 = 5 A.
Find the total energy stored when the coefficient of coupling is M = - 0.45H.
A. 12.75 J C, 127.5 J
B. 1275 J D. 1.275 J
This Free Quality Manual is
Found only at POWERLINE

E-MAIL: powerlinereview@yahoo,com

M E Z Z A N IN E FLO O R , D O N A A M P A R O B UILD IN G
C O R N E R E S P A N A & G. M. TOLEN T1N O S T R E E T S
SAIVIPALOC, M A N IL A
TE L. N O S 7 3 5 -7 3 -0 2 & 7 3 3 -2 1 -1 8

TE L. NOS. ( 0 3 ) 2 6 1 - 2 2 4 4 & ( 0 3 2 ) 2 6 1 - 8 4 5 2
POWERLINE ENGINEERING REVIEW CENTER, INC,

DIRECT CURRENT MACHINERY

DYNAMO - is a rotating machine for converting mechanical energy into electrical energy, or the reverse
process, electrical energy into mechanical energy.

Induced Voltage in Dy namo


ZP(p N
E = ------- L - * 1 0 ~ 8 Volts
60a
General Classification:
[I]. Motor
[II]. Generator

I - DC Motor - a continuous energy conversion device used to convert electrical energy to mechanical
energy.

General types of Self Excited Motors :

1. Series wound D.C. Motor- a motor where its winding called series field winding is connected
in series with the armature.
Speed Characteristics: Variable speed
Torque Characteristics: high starting torque
Uses: elevator, crane, converyor, hoist gear drive.
*Atote: to reverse direction of rotation of this motor, interchange the brushes.
Caution: Never operate the motor if unloaded because it will K'r<ice" or 'run away■

Schcrnafic Hiring Diagram fw/wvalcnl Circuit

VT - terminal voltage
Eb - back emf or counter emf
Vbc - brush contact voltage drop
Vs - series field winding voltage drop
h - line current or input current
Ia - armature current
- series field winding resistance
Rbc - brush contact resistance
Ra - armature resistance
R - equivalent resistance due to the power developed in the armature
Pi - power point
Pd ' - power developed in the armature
P0 - power output
- useful power
2. Shunt Wound D.C. Motor - a self excited motor where its field winding called shunt field is
connected across the armature .

Speed Characteristics: nearly constant or adjustable speed.


Torque Characteristics: medium starting torque
Uses: fan, pump, grinder, blower etc.
* Note: to reverse the direction of rotation of the motor interchange the brushes or
reverse the connection of the field windings but not both.
Caution: Never open.the field windings while motors is running because it will “race”
“run away”
Schematic Wiring Diagram Equivalent Circuit

Shunt
V t a -field
i
w dg.

i
3.Compound Wot*nd PC Motor - where its windings has series field and shunt field windings,
either connected long shunt or short shunt.
Speed Characteristics: variable or adjusted speeds
Torque Characteristics: high starting torque
Uses: milling machine, punching machine, elevator crane etc.
*Note: To reverse the direction of rotation of the motor interchange the brushes.

a. Long Shunt Compound Motor ( adjustable speed)


Schematic Wiring Diagram Equivalent Circuit
---------- > —
+ r - T„
feries. O
o R& % %
■fteld o sl
Wdj. Shuni °o '
O r bc
-field o
o
wdg. O
o
r v
*©-
b. Short Shunt Compound Motor (variable speed)
Schematic Wiring Diagram Equivalent Circuit

- in ­
sert e$ VS
field , i
Vt fie ld o C>vt
I W(}(), C
DC MOTOR FORMULAS:

1. Induced EMF

60a
PZ
Let K =
60a
E = K<£N

Where:
P- number of poles
<J>- flux per pole in weber
N- armature rotation in RPM
Z- total number of active conductor
Z= no. of slots x Conductor per slot
a- number of parallel paths
a = mP
m - winding multiplicity
2. Voltage Equations:

Series Motor:
Eb = Vt - IaRa - IaRs - Vbc
Pin
la = I, = Is = —
L Vt
Shunt M otor:
Eb = V t- IaRa - Vbc
la = IL - If

f Rf

3. Speed of DC Shunt Motors, N = -V t' laRa b-C


F K0>

4. Armature Power Developed, Pdev = Eb •la

PQNZ
- ^
Torque J
Developed, ^T = ------
Pdev = ------------
Eb-Ia o 60a
y a — - PZ
---- O • laT =K<D-Ia
2*N' 2^1' ln V 2m
60

6. Motor Efficiency, 77
Po Vt • I, - Copper Losses - Rotational Losses
^OVERALL ~ ~ r '
Pin Vt • I,
Po Po
7? 'o v e r a ll ~
Pin Po + Copper Losses + Rotational Losses

Pdev
Pdev + Copper Losses

________ Po _____
'm e c h a n ic a l ~
Po + Rotational Losses

Pdev
Pdev + Armature Copper Loss

7. Maximum Efficiency Condition

Variable Losses = Constant Losses


For Shunt Motor,
Ia2Ra = SPL + If2Rf
(Constant Losses
Ia’=
t Ra
I L’= Ia M f

Vt • IL'- 2 • Constant Losses


^JxfAX ^j i

8. Core Loss or Iron Loss, Wh+E


Wh+E = Hysteresis Loss + Eddy Current Loss
Hysteresis Loss, Wh = r]Bl bf • Vol
For constant flux density, Wh = A N
Eddy Current Loss - KhB 2f 2t2Vol
For constant flux density, Ws = B N 2
Where:
rj- Steinmetz Constant in Joule/(m’ cycle) or ergs/(cm3 cycle)
B - flux density in Tesla or Gauss
PN
f - frequency of magnetic reversal in cps f = y—

N - speed in rpm
Vol - volume of armature core in m3 or cm
t - thickness of armature laminations
Kh - hysteresis constant
II - DC Generator - an energy conversion device used to convert mechanical energy to electrical energy

General Types o f S e lf Excited D C Generators :

1. Series Wound DC Generator - like the series wound DC motor, its field winding is connected in series
with the armature. It is used for constant current application like in
series street lighting.
Schematic Wiring Diagram Equivalent Circuit

Ka Ks — ,

2, Shunt Wound D.C. Generator - like the shunt wound D,C. motor its field winding also called shunt
I field winding is connected across the armature. It is used for constant
voltage application like in electric welding.
1 Schematic Wiring Diagram
------------*

3. Separately Excited Shunt Generator - its winding also called shunt field winding is connected to an
outside source ( a battery or another D.C. generator) It is used for
laboratory experiment purpose where voltage are likely to be varied.
Schematic Wiring Diagram Equivalent Circuit

o
FieU o
o
tody
o
o

4,Compound W'ound D.C. Generator - like compound wound D.C. motor. It has series field windings
and shunt field windings also either connected long shunt or short shunt.

Types o f compound Gen . according to field windings connection :


Long Shunt Schematic Diagram

£ shunt Sprite fi n
i
field wdg v-t
wdg i
t
si--
Short Shunt Schematic Diagram
DC GENERATOR FORMULAS:

1. Generated EMF
P$NZ
Eg =
60a
PZ
Let K =
60a
Eg = K<E>N.

Where:
P- number of poles
<E>- flux per pole in Weber
N- armature rotation in RPM
Z- total number of active conductor
Z= no. of slots x Conductor per slot
a- number of parallel paths
a,= mP
m - winding multiplicity
2 . Armature Winding:
Conduc tor Current, / = —

Coil Pitch, Ys = —~ K
P

Back &.Front Pitch (Progressive Winding)


Back Pitch, Y„ = — +1
p
FrontcPitch, YF = — -1
P
Commutator Pitch, Yc

Lap Winding, Yc = ±m
Wave Winding, Yc = <:- ^ -
P! 2
C- number of commutator bars or number of coils

3. Voltage Equation:

Series Generator
Eg = Vt + IaRa + IaRs + Vbc
Shunt Generator
Eg = Vt + IaRa + Vbc
la = IL + If

Rf

Voltage/Speed Relation of DC Shunt Generators, N

Armature Power Generated, Pgen - Eg • la

Efficiency, 7
_ P° _ Vt IL
V'o v e r a l l
Pin Vt • IL + Copper Losses + SPL

Maximum Efficiency Condition

Variable Losses = Constant Losses


For Shunt Motor,
Ia2Ra = SPL + If2Rf
Constant Losses
Ia'=
Ra
IL'= Ia '-If

Vt *1L
Vm ax
Vt • Il '+2 •Constant Losses
Types o f Compound Generator According to direction o f flux :

a) Cumulative Compound Generator - the series field aiding the shunt field to supply power
and lighting loads.

a - 1 Under Compounded - the full load terminal voltage is less than the no load
voltage. It is used when the load is near from the generator.
a - 2 Flat Compound - the full load terminal voltage is the same as the no load voltage,
it is used when the load is at medium distance from the generator.
a —3 Over Compounded - the full load terminal voltage is greater than the no load
voltage, il is used when the load is far from the generator.

b) Differential Compound Generator - the direction of flux of the series field winding opposes
the direction of flux of the shunt field winding. It is used electric welding.

Prime Movers
1. Hydraulic turbines
2. Steam Turbine
3. Internal Combustion Engine

* Note : Compensating Winding -Connected in series with the armature


- used to neutralize reaction
Interpole Winding - In series with the armature
- used for spark less commutations.
Diverter resistance - parallel with the series field.

PARALLEL OPERATION o f D C GENERATORS :

Conditions to he met for idea! operation o f D C generators in parallel arc

(L) The no-load voltages of both generators should be adjusted to the same value ( otherwise„
current will circulate through the two machines that will cause additional losses ).

(2.) The external characteristic curve of the two generators should be identical ( otherwise, the
generator will not share the load properly).

(3,) The polarity of the voltages of two generators must be “opposing” in the series circuit of the
two machines ( otherwise, large current will circulate in the armature windings of the generate
will damage the windings ).

(4.) For the case of compound generators in parallel there must be an 14Equalizer ’ connection to
make the operation stable ( otherwise, there will be “motorization”, i.e. one of the generator
will acts as a m otor).
POWERLIne r e v ie w c e n t e r Inc.
The Ultimate E l Review Center
DC MACHINES REVIEW PROBLEMS
1. Calculate the voltage induced in the armature winding of a 4-pole, lap-wound, dc
machine having 728 conductors running at 1800rpm. Flux per pole is 0.03Wb.
A: 655.2 V C. 720.4 V
B. 468.2 V D. 236.4 V

2. A dc machine has a 4-pole, wave-wound armature with 46 slots and 16


conductors per slot. If the induced voltage in the armature is 480 V at 1200rpm,
determine the flux per pole.
A. 48 mWb C. 0.94 mW b
B. 1.2 mWb D. 16.3 mWb

3. A 115 volt shunt m otor has an armature whose resistance is 0.22 ohm. Assuming
voltage across the brush contact is 2 volts, what armature current will flow when
counter em f is 108 volts.
A. 22.7A C. 34.6 A
B. 31.8A D. 27.2 A

4. The nameplate rating of a shunt m otor is 150 hp, 600 volts, 205 amp, 1700 rpm.
The resistance of the shunt field is 240 ohms and the total arm ature resistance is
0.15 ohm. Calculate the counter emf.
A. 569.63 volts C. 569.25 volts
B. 586.52 volts D. 630.75 volts

5. The term inal voltage of a motor is 240 volts and its armature current is 60A. The
armature resistance is 0.08 ohm. W hat is the hp developed by the motor.
A. 18.92 C. 16.25
B. 17.26 D. 26.64

6. A certain load to be driven at 1750 rpm requires a torque of 60 Ib-ft. W hat hp will
be required to drive the load?
A. 20 C. 12
B. 15 D. 16

7. A shunt m otor is running at 1200 rpm for a load which requires an armature
current of 50 ampere from 230 volt source. At no load, the arm ature current is 5
amp. If the effect of armature reaction has reduced the air-gap flux by 2% from no
load to full load, determ ine the no load speed. The armature resistance is 0.15Q
A.. 1212 rpm C. 1200 rpm
B. 1012 rpm D. 1182 rpm

8. In a motor, the armature resistance is 0.1 ohm. W hen connected across 110 volts
mains, the arm ature takes 20 amp and it speed is 1200 rpm. Determ ine its speed
when the arm ature takes 50 A from the same mains, with field increased 10: :
A. 1023 rpm C. 1171 rpm
Bi 1061 rpm D. 1251 rpm
9. A 120-V dc shunt m otor having an armature circuit resistance of 0.2 ohm and field
circuit resistance of 60 ohms, draws a line current of 40A at full load. The brush
drop is 3-V and full load speed is 1800 rpm. W hat is the speed at 125% full load?
A 1765 rpm C. 3600 rpm
B. 1863 rpm D. 720 rpm

10. A Shunt generator while running light as a m otor at 1000 rpm takes 12 ampere
from 115-V mains. The field current is 7 A and the armature resistance is 0.03
ohm. Determine the stray power loss?
A. 574W C. 942W
B. 543W D. 862W

11. A shunt m otor runs at 1100 rpm, at 230 V, and draws a line current of 40 A. The
various losses are: core loss, 200 W; friction and windage loss, 180 W; electrical
loss due to brush contact, 37 W; stray-load loss, 37 W. The armature- and field-
circuit resistances are 0.25 Q and 230 £2, resp. Solve for m otor efficiency.
A. 89.56% C. 87.89%
B. 88.43% D. 92.48%

12. A 50 hp 500 volt shunt m otor draws a line current of 4.5 amp at no load. The
shunt field resistance is 250 ohms and the armature resistance is 0.3 ohm. The
brush drop is 2 volts. The full load line current is 84 amp. W hat is the efficiency?
A. 89.45% C. 86.25%
B. 87.26% D. 76.64%

13. Calculate the efficiency of a 230 Volt shunt m otor from the following data : Input
current is 19.8 Amp, balance reading is 14 lbs, tare weight is 2 lbs., brake arm is
2 ft long & Speed is 1100 rpm.
A.' 82.4 % C. 84.2 %
B. 8 1 .4 % D. 8 6 .5 %

14. A 230 Volts, 15 hpd.c. shunt m otor has friction and iron loss of 200 W . The
armature resistance is 0.3 ohm and shuntfield resistance is 115 ohms. W hat is
the maximum efficiency of the motor?
A. 86.9% C. 88.3%
B. 87.4% D. 85.1%

15. A 120 volts shunt m otor takes 75 A armature current at full load. Arm ature
resistance is 0.2 ohms and brush contact drop is 2 V. Find the starting resistance
to limit the starting torque to 150% of rated load torque at the instant of starting.
A. 0.85 Q C. 0.82Q
B. 0.82 Q D. 0.88 Q

16.The armature of a 230 volt shunt motor has a resistance of 0.18 ohm. A
resistance is inserted in series with the armature at the instant of starting so as
not to exceed 76 amp at starting, calculate the value to which this resistance can
be reduced when the armature accelerates until Ec is 168 volts. Brush drop is 2V
A. 0.61 Q C. 0.81 Q
B. 2.82 Q D. 0.28 Q
17. The armature circuit resistance of a 25 hp, 250 volts series m otor is 0.1 ohms.
The brush drop is 3 volt and the resistance of the series field is 0.05 ohm. When
the series m otor takes 85 amperes, the speed is 600 rpm. Calculate the speed
when the current is 40 am peres? >,
A. 1260 rpm C. 1282 rpm
B. 1171 rpm D. 1246 rpm

18. A series motor, having an armature resistance of 0.1 Q and a field resistance of
0.15 Q , takes 48 A at 230-V and 720 rpm. The total core and friction losses are
650 W. neglecting stray-load and brush-contact losses, calculate torque
developed.
A. 138.8 N-m C. 130.2 N-m
B. 141.8 N-m D. 113.6 N-m

19. A series motor, having an armature resistance of 0.1 Q and a field resistance of
0.15 Q , takes 48 A at 230 V and 720 rpm. The total core and friction losses are
650 W. neglecting stray-load and brush-contact losses, calculate the motor
efficiency.
A. 88.9% C. 83.6% -
B. 85.7% D. 78.6 %

20.A 10 hp, 230 Volt, 1000 rpm series motor, having rated efficiency of 85.5 %,
armature resistance including brushes of 0.28 Q and a field resistance of 0.15 £X
W hat value of starting resistance so that the starting torque is equal to 150 % of
the rated-load torque ?
A 4.95 Q C. 5.94 Q
B. 4.52 Q D. 4.67 Q

21. A six pole dynamo, with interpoles, requires 360 inductors on its armature, with
three turns per coil and two coil sides per slot. Determ ine the num ber of
com m utator bars.
•A. 60 C. 120
B. 90 D. 160

22. A four pole generator has 500 conductors on the armature. If the generator is
running at 1200 rpm, find the average voltage generated between brushes for
wave winding. The total flux per pole is 106 lines.
A. 200 V C. 120 V
B. 100 V D. 250 V

23. Assuming a constant field excitation, calculate the no-load voltage of a separately
excited generator the arm ature voltage is 150 volts at 1800 rpm. W hat is the
voltage if the speed is increased to 2000 rpm?
A . 167 C. 133
B. 120 D. 180
24. A 10 kW 250 volts self-excited generator, when delivering rated load, has an
armature-circuit voltage drop of 5% of the terminal voltage and a shunt field
current equal to 5% of the rated load current. Calculate the resistance of the field
circuit.
A. 125 ohms C. 500 ohms
B. 250 ohms D. 62.5 ohms

25.A self-excited s h u n t generator supplies a load of 12.5 kW at 125 V. the field


resistance is 25 £1 and the armature resistance is 0.1 Q . The total voltage drop
because of brush contact and armature reaction at this load is 3.5 V. calculate the
induced armature voltage.
A. 129 V C. 135 V
B. 263 V D. 139 V

26. A 2500 kW 600-V 16-pole generator has lap wound arm ature with 2360
conductors. If the pole face covers 65% of the entire circum ference, calculate the
number of pole face conductors in each pole of a com pensating winding.
A. 6 C. 5
B. 9 D. 4

27. A certain 110-volt shunt generator has an armature and brush resistance of 0.06
ohm at full load of 85 amp. The resistance of the shunt field is 45 ohms, and the
stray power losses are found to be 897 watts. Calculate the full load efficiency.
A. 85.2% C. 86.5%
B. 89.4% D. 84.9%

28. A 250 kW 230 volt compound generator is delivering 800 amp at 230 V. The
shunt field current is 12 amp. The arm ature resistance is 0.007 ohm and the
series field resistance is 0.002 ohm. The stray power at this load is 5500 watts.
The generator is connected long shunt. Determine the generator efficiency at this
load Stray load loss is 1% of output.
/ y 92% C. 95%
B. 94% D. 90%

29. A separately-excited dc generator has the following data: arm ature resistance,
0.04 Q ; field resistance, 110 Q ; total core and mechanical losses, 960 W;
voltage across the field, 230 V. the generator supplies a load at a terminal voltage
of 230 V. Calculate the maximum value of the generator efficiency.
A. 93.8% C. 93.2%
B. 92.7% D. 98.2%

30. Two shunt generators A & B running in parallel share a load o f 100 kW equally at
a terminal voltage of 230 volts. On no-load their voltage rises to 240 and 245 volts
respectively. How will they share the load when the total current is reduced to half
of its original value.
A. 20 kW & 30 kW C. 25kW each
B. 18 kW & 32 kW D. 1 5 k W & 3 5 k W
DO Machines Supplementary Problems
1. Dirt on the motor commutator may produce___
A. isolation C. lower torque
B. excessive sparking D. power loss

2. If a motor frame is found excessively hot, which of the following is the LEAST likely cause
A. very high ambient temperature C. inadequate internal ventilation
B. faulty starter D. excessive loading

3. The simplest form of a motor controller i s ......


A. drum switch C. magnetic switch
B. relay D. toggle switch

4. Rapid stopping of motor by momentarily reversing its connections to the supply is ...
A. jogging C. inching
B. plugging D. sequence operation

5. A dc generator may loose its residual magnetism because o f ......


A. varying loads C. over-excitation
B. vibrations D. heating or overload

6. The simplest way to shift load from one dc shunt generator running in parallel with
another is to ..
A. adjust their fieldrheostatC. insert resistance in their armature circuits
B. adjust speeds oftheirprime movers D. use equalizer connections

7. For both lap and wave windings, there are as many commutator bars as the number of
A. poles % C. armature conductors
B. slots D. winding elements

8. It is a DC motor that tends to over-speed when the field opened while running.
A. shunt motor C. split phase motor
B. series motor D. over-compounded motor,

9. In DC shunt motors, how is the shunt field connected in relation with the armature?
A. parallel C. parallel series
B. series - parallel D. series

10. DC shunt motor are commonly used in what application?


A. high starting torque C. constant speed
B low no-load speed D. high speed

11. Which motor is used to start heavy loads?


A. shunt C. series
B. commulatively compound D. shaded pole

12. When the load is removed, which motor will run at the highest speed?
A. series C. shunt
B. commulatively compound D. Differentially compound

13. Which motor is suitable for elevator?


A. series C, shunt
B. commulatively compound D. Differentially compound
14 If a self-excited dc generator after being installed, fails to build up on its first trial run, the
first thing to do is to.
A. increase the field resistance C reverse field connections
B. check armature insulation resistance D. increase the speed of prime mover

15. The voltage regulation of over-compound generator is always


A. positive C. zero
B. negative D. high

16. A 240 volts dc shunt motor has an armature current of 50 ampere, armature resistance of
0.25 running at 1200 rpm. If it is run at 220 volts, what value of armature current when
driving the same load. The field is adjusted so that the speed remains at 1200 rpm.
A. 55 C. 60
B. 50 D. 57.5

17 A dc shunt motor develops 15 hp at 120 volts. The armature resistance is 0.061 ohm.
What is the armature current?
A. 95.81 Amperes C. 96.15 Amperes
B. 91.85 Amperes D. 98.15 Amperes

18. A dc shunt motor develops 15 hp at 120 V. The armature efficiency is 95%. What is the
back emf?
A. 114 volts C. 118 volts
B. 122 volts D. 117 volts

19. A 120 Volt shunt motor has an armature resistance of 0.08 ohm and field resistance of
60 ohms. What current is drawn when its back emf is 114.8 volts.
A. 65 A C. 63 A
B. 67 A D. 70 A

20. A 440 volt 75 hp shunt motor has an armature resistance of 0.185 ohm and a field
resistance of 350 ohms. The current drawn at full load is 135 amperes. What is the back
emf in volts?
A. 415.3 C. 394.8
. B. 398.3 D. 417.1

21. A 4 poles lap wound, dc machine has 728 active conductors running at 1800 rpm. The
flux per pole is 30 mWb, The armature current is 120 A, what is the electromagnetic
oower developed in the armature in kw?
A 72 4 C. 65.5
B 78.6 D. 62.4

22 The 3'~iature and field circuits of a 230 Volt shunt motor have resistance of 0.31 and 115
ohms 'espectively. It takes 52 Amp when operating at a certain load. What is the
ar"'a :jre power developed?
A 14.4 hp C. 7.5 hp
B 24.6 hp D. 12.4 hp

23 A 500 Volt shunt motor running at 720 rpm takes'an armature current of 50 Amp, its
effective resistance 0.40 ohm. W l^ n a resistance of 1.6 ohm is placed in series with the
armature, what is the new speed ? Assume that load torque is constant.
A. 720 rpm C. 750 rpm
B. 600 rpm D. 680 rpm
24. A dc shunt motor runs at 900 rpm from a 460V supply when taking an armature current of
25 A. Calculate the speed at which it will run from a 230V supply when taking an
armature current of 15A. The resistance of the armature circuit is 0.8 ohm. Assume flux
per pole at 230V have to decrease to 75% of its value at 460V
A. 495 C. 585
B. 485 p } 595

25. A separately-excited motor runs at 1045 rpm, with a constant field current, while taking
an armature current of 50 A at 120 V. the armature resistance is 0.1 Q . If the load on the
motor changes such that it now takes 95 A at 120 V, determine the motor speed at this
load.
A. 1004 rpm C. 1014 rpm
B. 1104 rpm D. 994 rpm

26. A 20-hp, 250-V shunt motor has an armature-circuit resistance of 0.22 Q and a field
resistance of 170 Q . At no-load and rated voltage, the speed’is 1200rpm and the
armature current is 3.0 A. at full-load and rated voltage, the line current is 55 A, and the
flux is reduced 6% (due to the effects of armature reaction) from its value at no-load.
What is the full load speed?
A. 1220 rpm C.)1200 rpm
B. 1188 rpm D. 1260 rpm

27. A shunt motor with an armature resistance of 0.15 ohm is running at 1200 rpm for a load
which requires of armature current of 50 ampere from a 230 volt source. At no load, the
armature current is 5 ampere. If the effect of armature reaction has reduced the air gap
flux 2% from no load to full load, what is the no load speed?
A. 1215 rpm C, 1,209 rpm
B. 1268 rpm D. j,2 1 2 rp m

28. The armature of a dc machine running at 1200 rpm carries 45 A in current. If the induced
armature voltage is 130 V, what is the torque developed by the armature?
A. 46.5 N-m C. 45.6 N-m
B. 4.65 N-m D. 456 N-m

29. Calculate the electromagnetic torque developed in the armature of a 4-pole, lap-wound
dc machine having 728 active conductors and running at 1800 rpm. The flux per pole is
30 mWb. The armature is designed to carry a maximum current of 100A.
A. 34.76 N-m C. 347.6 N-m
B. 3.476 N-m D. 3476 N-m

30. A 4-pole motor is lap wound with 728 conductors, and has a flux of 25 mWb per pole.
The armature takes 50 A in current; the resultant demagnetizing effect due to armature
reaction reduces the air-gap flux by 5%. Calculate the developed torque.
A. 137.6 N-m C. 144.8 N-m
B. 167.3N-m D. 176.3 N-m

31. The armature of a motor has of 400 active conductors, the flux density in the field being
3000 gauss. The axial length of the armature is 20.32 cm. The current is 30 amperes.
Determine the total force exerted in the armature.
A .> 31.5 N C. 0.07 N r - & \l ^
B. 645.9 N D. 439.2 N
32. A dc motor takes an armature current of 105 Amp at 480 volts when running at 720 rpm.
The resistance of the armature circuit is 0.2 ohm and the field resistance is 96 ohms.
What is the torque developed in the armature ?
A. 668 N-m C. 752 N-m
B. 610 N-m D. 639 N-m

33. An electric motor develops 20 metric HP at a speed of 600 rpm. What is the torque?
A: 240 N-m C. 210 N-m
B. 290 N-m D. 260 N-m

34. A 4-pole lap-wound' armature has 144 slotswith two coil sides per slot, each coilhaving
two turns. If the flux per pole is20mWb andthe armature rotates at 720rpm. What is the
induced voitage?
A. 138.24 V C. 69.12 V
B. 34.96 V D. 276.48 V

35. A d.c. shunt motor develops 7.8 hp at 120 Volt, the armature efficiency is 93%. What is
the armature effective resistance?
A. 0.1 ohm C. 0.16 ohm
B. 0 06 ohm D. 0.05 ohm

36. A certain shunt motor has an armature resistance of 0.05. It draws 50 amperes at a
terminal voltage of 120! volts. Assume other miscellaneous losses of 1%. Determine the
output horsepower of the motor.
A. 8.5 hp C. 6.8 hp
B. 7.1 hp D. 7.8 hp

37. In a brake test the effective load on the branch pulley was 38.1 kg, the effective diameter
of the pulley is 63.5 cm and the speed is 12 r.p.s. The motor took 49 ampere at 220 volts.
What is the efficiency at this load?
A. 85% C. 86%
B 80% D) 83%

38. A 240 volt d.c. motor on brake test took 52 Amp when running at 1500 rpm. The spring
balance at the end of the 76 cm brake arm reads 93.5 N. What is the efficiency of the
motor at this load ?
A. 0.85 C. 0.90
B. 0.95 , D. 0.86

39. The shaft power of a shunt motor is 7.8 hp. Its armature draws 50 Amperes from 120
Volts. The field winding draws 1.2 Amp. What is the efficiency of the motor?
A. 94.7% C. 96.5%
B: 93 % D. 98.2 %

40. A 15 hp 220 volt, 1200 rpm shunt motor takes 50 Amperes. Its armature resistance is 0.1
ohm and field resistance is 220 ohms. Brush contact voltage drop is 2 volts, Friction and
windage loss is 400 watts and stray load loss of 1% of the output. Calculate the
efficiency.
A. 91.8% C. 90.4%
B. 92.4% D. 94.6%
41. A 250-V, 10-hp shunt motor has an armature resistance of 0.5 Q and a field resistance of
250 Q . The motor takes a current of 5 A on no-land and 37.1 A on rated load. Determine
its rated-load efficiency.
A. 88.4% C. 86.5%
B. 76.4% D. 79.7%

42. A shunt m otor runs at 1100 rpm, at 230 V, and draws a line current of 40 A.The
armature- and field-circuit resistances are 0.25 Q and 230 Q , respectively.
Calculate the m otor speed if the output power is reduced by 50%
A. 1125 rpm K' , C. 1080 rpm
B. 1344 rpm J K) D. 2200 rpm
v ->& <-‘ Sfg - ? ' :
43. A 440 Volt shunt motor has an armature resistance of 0.8 ohm and field resistance of
200 ohms. What is the armature power developed when giving an output of 7.46 kW at
85% efficiency.
A. 7.56 kW C. 7.46 kW
B. 8.46 kW D. 7.23 kW

44. In a 5 Hp, 230 Volt, 1500 rpm shunt motor, the resistance of the armature including
brushes is 0.175 ohm and that of the shunt field is 610 ohms. The stray power losses
when the motor delivers rated load at rated voltage is 305 watts. The efficiency at rated
load is nearest to.......
A. 98% C. 90 %
B. 89% D. 85%

45. A 10-hp, 230-V shunt motor takes a full-load line current of 40 A. the armature and field
resistances are 0.25 Q and 230 Q , respectively. The total brush-contact drop is 2 V and
the core and friction losses are 380 W. calculate the efficiency of the motor. Assume that
stray-load loss is 1% of output. **** ^ -%:
A. 78.6% C. 96.4%
B. 87.6% D. 83.5%

46. A 15 hp 220-volt shunt motor has an armature resistance of 0.07 ohm. and a shunt
■resistance of 200 ohms. The overall efficiency at rated output is 89%, What is the
rotational loss?
A 688 watts C. 921 watts
B; 1,383 watts Voip i / , D. 242 watts
fh o st 3
47. A 5 Hp, 230 V shunt motor takes 18 Amperes when operating at full load. The shunt field
resistance is 115 ohms and the armature resistance is 0.25 ohm. Calculate the starter
resistance if the starting torque is limited to 1.5 times the full load torque (take a 3-V
brush drop).
A. 13.94 Q C. 9.46 Q
B. 9.2 Q D. 9.33 Q

48. A shunt motor yvith armature resistance of 0.12 Q and designed to operate at 115 V, is
taking 60 A in armature current at full-load. Determine the value of the external
resistance to be inserted inthearmature circuit so that the armature current shall not
exceed twice its full-load value at starting.
A. 0.838 ohms C. 0.808 ohms
B. 0.883 ohms D. 0.788 ohms
49 A 20 Hp, 240 V shunt motor takes 72 Amperes when operating at full load. The shunt
field resistance is 240 ohms and the armature resistance is 0.22 ohm. Calculate the
starter resistance if the starting torque is limited to 1.5 times the full load torque (take a
3-V brush drop).
A. 4Q C. 6 Q
B. 2Q D. 1.8 Q

50. A dc series motor develops 180 lbs.-ft of torque when the is current is 30 Amp. The load
increases so that the motor current rises to 50 Amp. The torque now is...
A 500 lbs.-ft C. 64.8 Ibs.-ft
B. 300 Ibs.-ft D. 108 Ibs.-ft

51. A series motor, having an armature resistance of 0.1 Q and a field resistance of 0.15 Q ,
takes 48 A at 230 V and 720 rpm. The total core and friction losses are 650 W. neglecting
stray-load and brush-contact losses, calculate output power at the shaft.
A. 11.36 HP C. 13.16 HP
B. 16.13 HP D. 113.6 HP

52. For a 4-pole, 2 -layer, dc lap-winding with 20 slots and one conductor per layer, the
number of commutator bars is.
A. 80 C. 40
B. 20 D. 160

53. A 4 pole, lap-wound dc armature, the back pitch is 17, if the winding is progressive what
is the front pitch?
A. 15 C. 17
B. 13 D. 16

54. In a 4 pole, 54 slot d.c. armature, calculate the coil span .


A. 18 C. 14
B. 9 D. 13

55. If the armature of an eight-pole dc machine were wound with a simplex wave winding.
' How many parallel paths would there be ?
A. 16 paths C. 8 paths
B. 2 paths D. 4 paths

56. In a 4 pole, 35 slot d.c. armature, calculate the coil span .


A. 8 slots C. 10 slots
B. 7 slots D. 9 slots

57. A 4 pole, 12 slot lap-wound dc armature has two coil side/slot. Assuming single turn coil,
calculate the back pitch for progressive and retrogressive winding.
A. 5 & 3 C. 7 & 5
B. 3 & 5 D. 6 & 4

58. In a 10 pole, lap wound dc armature, the number of active armature conductor per pole is
50. The number of compensating conductor per pole required is
A. 5 C. 50
B 500 D. 10
59. A 4 pole dc generator with simplex lap winding has 48 slots and four element per slot.
The flux per pole is 2.5 x 106 maxwells and it runs at 1,500 rpm. What is the output
voltage ?
A. 60 Volts C. 225 Volts
B. 360 Volts D. 120 Volts

60. A 4 pole dc generator with duplex lap winding has 48 slots and four element per slot. The
flux per pole is 2.5 x 106 maxwells and it runs at 1,500 rpm. What is the output voltage ?
A. 60 Volts C 225 Volts
B. 480 Volts D. 120 Volts

61. A separately excited generator generates 230 volts when driven at 1200 rpm, If the flux is
decrease by 10%, What is the new emf with speed remains the same.
A. 230 volts C. 207 volts
B. 256 volts D. 253 volts

62. The input current to a 220 volts, long shunt compound motor at no load is 6 amp. The
shunt field circuit resistance is 220 ohms, the armature resistance is 0.1 ohm and series
field resistance is 0.08 ohm. What is the stray power loss?
A. 1095.5W —" C. 1086.5W
B. 1097.5W D. 1189.6W

63. A 20-kW 220-volt shunt generator has an armature resistance of 0.07 ohm and a shunt
resistance of 200 ohms. Determine the power developed in the armature when it delivers
its rated output.
A. 27.9 kW C. 19.2 kW
B. 20.83 kW D. 25.7 kW

64. The pulley of an old gen-set has a diameter of 20 inches. The belt exerts a pull of 353 lbs
on the pulley . the gen-set runs at 900 rpm. What is the approximate kW rating of the
gen-set ? , < y
A. 75 C. 200
B. 250 D. 35

65. A 288 volt shunt generator has constant losses of 2880 Watts. The armature circuit
resistance is 0.20 ohm and field resistance of 144 ohms. If the generator is delivering
power at maximum efficiency, then its armature current is.
A. 10 A C. 130 A
B. 120 A D. 64 A

66. A 400- kW, 600 volt, d-c shunt generator has an armature resistance of 0.03 ohm. The
field resistance is 48 ohms. The stray power loss for this machine at rated voltage and
speed is 7200 watts, What is the maximum efficiency?
A. 93.4% C. 95.3%
B. 96.3% D. 98.5%

67. A shunt generator delivers 195 Amp of terminal of voltage 250 Volts. The armature
resistance and shunt field resistances are 0.02 ohm and 50 ohms respectively. The iron
and friction losses equal to 950 watts. What is the overall efficiency?
A 94 % C. 96 %
B. 98% D. 95%

J
68. A 440 Volt long shunt generator has a full-load current of 200 Amp. Its armature
resistance, series field resistance and shunt field resistances are 0.02 ohm, 0.04 ohm
and 100 ohms respectively, the stray load losses are 2000 Watts. What is the full-load
efficiency of generator ?
A. 97% C. 98%
B. 95% D. 93%

69. A 25 kW, 500 volts long shunt compound generator delivers a full load. It has armature,
series field and shunt field resistances of 0.05fi, 0.03Q and 250Q respectively. What is
the generated voltage allowing 3 V for brush contact drop?
A. 496 V C. 503 V
B. 507 V D. 484 V

70. A long-shunt compound generator supplies 50 kW at 230 V. the total field- and armature-
circuit resistances are 46 Q and 0.03 Q , respectively. The brush-contact drop is 2 V.
Determine the percent voltage regulation. Neglect the armature reaction.
A. 3.97% . C. 2.9%
B. 3.57% D. 3.77%

71. A 100-kW, 230-V, shunt generator has Ra= 0.05 Q and Rf= 57.5 Q . If the generator
operates at rated voltage, calculate the induced voltage at half full-load. Neglect brush-
contact drop.
A. 241 V C. 252 V
B. 230 V D. 238.2V

72. A 50-kW, 250-V short-shunt-compound generator has the following data: Ra = 0.06 Q ,
R xe = 0 04 Q and R f = 125Q. Calculate the induced armature voltage at rated load and
terminal voltage. Take 2 V as the total brush-contact drop.
A 270.12V C. 272.12 V
B. 268.24 V D. 274.12 V

73. A 50-kW 460-volt shunt generator has a voltage regulation of 8.7 percent, Calculate the
no-load terminal voltage.
A. 475 C. 505
B. 500 D. 490

74. Two shunt generators A and B, with ratings of 150 kW and 100 kW respectively having
identical straight-line voltage vs. percent kW output characteristic, are connected in
parallel. The no-load and full-load voltage of 255 and 230 volts, calculate the kW
output of each when the total load shared is 200 kW.
A. 110 kW & 90 kW C. 1 2 0 k W & 8 0 k W
B. 150 kW & 50 kW D. 1 2 5 k W & 7 5 k W

75. Two shunt generators A and B, with ratings of 250 and 400 kW respectively having
equal no-load voltage of 260 volts and full-load voltage of 240 volts, calculate the load
of each when the total load shared is 600 kW.
A. 231 kW & 369 kW C. 242 kW & 358 kW
B. 238kW & 362 kW D. 234 kW & 366 kW

Practice Makes Perfect!!!


This Free Quality Manual is
Found only at POWERLINE

E-MAIL: poweriinereview@ya hoo.com

POWERLINE REVIEW CENTER


M EZ Z A N IN E FLO O R , D O N A A M P A R O B U ILD IN G
C O R N E R E S P A N A & G. M. TO L E N TIN O S T R E E T S
S A M P A LO C , M A N IL A
TE L . NO S. 7 3 5 -7 3 -0 2 & . 7 3 3 -2 1 -1 8

CEEIB-POWERLINE REVIEW CENTER


TE L . NOS. (0 3 )2 6 1 -2 2 4 4 & (0 3 2 )2 6 1 -8 4 5 2
1
P O W E R L IN E R E V IE W C E N T E R , INC-
IL ,1^U M I N A T I O N ~ may be defined as the light of proper color, quantity and so directed as
to allow quick and accurate vision without discomfort to eyes.

TERMS & DEFINITIONS


1. Luminous Flux (<b or F) - is the light energy radiated out per second from the luminous body in the form of
light waves. Its unit is lumen (Lm) one lumen is defined as the flux emitted per unit
solid angle from a standard candle.
A </> - I (Aco) where Aa) - a solid angle subtended at the center of the sphere in steradian

A(f> = I in lm :A 0) - - — —- = 4n sr ( for isotropic source)


r (radius)~
&<j> - An I
2. Luminous Intensity ( I ) - or candle power of the source in any direction is given by the luminous flux
radiated out per sec, on its unit solid angle in that direction . Its unit is candela (cd)
one candela is defined as one lumen per steradian.

I= in candela note : 1 candle power (older unit of I) - 0.981 candela


A co
3. Mean Spherical Candela power ( MSCP )
<f> . lumen
MSCP = — i n ------- or cd.
An Sr
3. Mean Hemispherical Candle power ( MHSCP )
. <t> . lumen .
MHSCP = — in ------- or cd.
In Sr
4. Illuminance (ILLUMINATION) (E) - is a measure of how light falls on unit area of the surface.
Its unit is dependent on Area:
, ' f lumen ^n , lumen , , , , x , lumen
1 lux ( Ix ) = 1 ---- — : 1 foot candle ( fc ) = 1 — r—: 1 phot ( ph ) = 1 -----r~
m ft1 cm
thus: 1 fc = 10.76 lux & 1 phot = 10,000 lux
£. - !L ^ iux, phot or fc

5. Specific Output (Efficacy) - it is the ratio of luminous flux to the power intake “W”.
lm^ 4 n x M.S.C.P
W watts
6. Principle of Photometry - the illumination (E) due to a point source decreases inversely as the square of
the distance from the source.

E^ a —I ; h
— - h -
- y -> mverse square law
r ri r2
For Horizontal surface For Vertical surface:
I cos9 - fY - / sin #
E 2 : 2
dl dl
n h . : t? \ (X u „ s
but cos 0= — n \ but sin 0 = —
d d
^ Ih . c H ,1 _ Is . ~
•- E = ——lux or fc 7 " I E - —tt lux or fc
d3 t d3
8- The space/height ratio is given by:
horizontal distance between lamps
mounting height o f lamps
2
9. The utilization factor or coefficient of utilization ( 77)

lumens actually received on working plane


7= ---------------------------------------------------- -
lumens emitted by the light source
Factors:
a) kinds of light
b) surface where light is coming ( walls,ceilings etc.)
c) color of the surface and surrounding
it varies from: 0.10 to 0.60 depending upon the nature of light
a) 0.10 to 0.40 for indirect lighting system
b) 0.40 to 0.60 for direct lighting system.
10. Maintenance Factor or Depreciation Factor ( p )
sometimes called Light Loss Factor.
illu min ation under actual condition
P= — ............ ...................................................................
illumination when everything is perfectly clean
Factors:
a) Because of dust.,dirt etc.
b) Blacking of filament the light emitted decreases
it varies: 0.60 to 0.80
also p = LLD x LDD x RSDD x LBO
where:
LLD ~ lamp lumen depreciation factor RSDD - room surface dirt depreciation factor
LDD = luminaire dirt depreciation factor LBO = lamp burnout (only for 2 or more luminaires)

LIGHTING DESIGN
- Lighting design is a combination of applied art and applied science.
GOALS OF LIGHTING DESIGN :
1. Lighting levels should be adequate for efficient seeing of the particular task involved.
2. Lighting equipment should be unobstructive, but not necessarily invisible.
3. Lighting must have the proper quality.
4. Lighting design must be accomplished efficiently in terms of capital & energy resources.

METHODS USED FOR LIGHTING DESIGN :

1. Illumination Methods ( Point-by-Point method)


a) General Liehtins - this is as system designed to give uniform and generally,
though not necessarily. Diffuse lighting throughout the
area under consideration.
b) Local and supplementary - these are two terms that are used interchangeably but
have slightly different meanings, uLocal Lighting “ provides
a small, high level area without contributing to the general
lighting. “Supplementary lighting “ also provides a restricted
area of high intensity, but supplements die general lighting.
s
c) Combined General Local Lizhtinz - is used in space where general visual
task is low, but supplementary lighting is required in a limited
area of particular task.
2. Lumens ( Flux ) Method
Lumen method of calculation is a procedure for determining the average maintained
illumination on the working plane in a room. The method presupposed that luminaires will be
spread so that uniformity of illumination is provided.
Z Q N A .L . C A V I T Y M E T H O D
In this method coefficient of utilization was determine through different tables related
to luminaires light distribution characteristic, to room size and its surface reflectance, to its
mounting height and its relationship to the working plane (i.e. Cavity Ratios)

£+ w
RCR = 5h RC room cavity ratio
ixw

t +w
CCR = 5h cc = ceiling cavity ratio
£xw

£+ w
FCR = 5h FC = floor cavity ratio
£xw

Where : hRC~ height of room cavity, hcc = height of ceiling cavity, & hFC^ height of floor cavity
FioodLighting
It means “ flooding u of large surface with the help of light from powerful projectors. Its
uses are for aesthetics, advertising and industrial/commercial lighting schemes.

FLOOD LIGHTING DESIGN (2x - 4x rule)

DISTINCTION BETWEEN TUNGSTEN FILAMENT LAMP &


FLUORESCENT TUBE:

TUNGSTEN FILAMENT L A M P ----FLUORESCENT LAMP

1. Its Light is close to the natural light. 1. Its light is not close to natural light
2. Actual color can be judged 2. Actual color can not be judge.
3. Initial cost is low 3. Initial cost is high
4. Life is about 1000 hrs. 4. Life is about 4000 hrs.
5. Maintenance cost is more 5. Maintenance cost is low
6. Brightness is more 6. Its light is cool and pleasant
I. Light output is reduced with time 7. Light output is also reduced but very less
8. Heat radiation loss is there 8. As the temperature is less soless radiation losses
9. Less lumen output per watt 9. More lumen output per watt (40 Im/watt)
10. Lumen efficiency is poor because of colored 10. The color of light depends upon the gas
glass. in the case of colored lamp.
II. No stroboscopic effect 11. It has stroboscopic effect.
P O W E R L IN E r e v i e w C E N T E R Inc.
The Ultimate E.E. Review Center
ILLUMINATION, R .A .7 92 0& IR R REVIEW PROBLEMS
1. Polar graph representing variation in candle power of illumination.
A. Photometric distribution C. Photo sensor
B' Photometric report D. Photometric brightness
2. It is a quantitative measure of the ability of a light source to reproduce the colors of
various objects faithfully in comparison with an ideal or natural light source. Also called
color accuracy.
A. spectral power C. luminous emittnace
B. color rendering index D. chromatic index

3. What do you call the interior lighting where the roof trusses or ceiling height is greater
than 7.6 m (25 ft) above floor level? r —^
A. High pressure sodium (HPS) High bay
B. High Intensity discharge (HID) D. High ballast
4. A lamp gives an average output of 628 lumens. What is the M.S.C.P.?
A. 45 tAvp r r>T C) 50
B. 48 _. D. 40
5. A 150 Watt light source has a mean spherical luminous intensity of 87.5 candelas.
Compute the luminous efficacy of the lamp.
A. 4.77 Im/w * 1 - _ *>V& y /iir C. 5.33 Im/w
B. 7.33 Im/w f “ IbQ D. 6.77 Im/w
6. A light has a candle power of 100 at a given direction at a point 10 ft distance from the
source, if the receiving plane is tilted through 30 degrees from the. perpendicular, what
is the illumination upon the surface? r *rh
A. 0.7 fc .. vv. *■ £■ 0.5 fc
B. 0.34 fc -J T ~ QD^0.866fc
7. A road is illuminated by two 500 cd lamps placed 6.1 m above the centerline of the
road and 18.3 m apart. Find the illumination produced on the surface of the road at a
point on the centerline midway between the lamps. £r ■ 2 f.
A. 2.3 lux , C. 4.6 lux - L ^ ,
B. 3.4 lux La. -boo ^ D. 6.8 lux r w ------
8. The illumination of a room is being designed. The illumination intensity required is
50 fc. The maintenance factor is 70% and coefficient of utilization is 0.6. If the room is
20 ft. by 15 ft. What is the total required lumen output of the lamp?
A. 10,500 lumens £ C. 6,300 lumens
B. 12,800 lumens w - D. 35,000 lumens 0 ^
C-Va M .f 'J *
9. A room 100 ft x 300 ft requires 80 foot-candle. It is to be illuminated by duplex
fluorescent lamp. The 2-T20 Watt lamp has an output of 3200 lumen, maintenance
factor is 0.7 and coefficient of utilization is 0.6. Calculate the number of luminaires.
k ) 1786 C. 894
B- 1144 ' j * i T d - 1228
10. A room with dimension 75 ft x 36 ft x13 ft requires 50 foot-candle, lamp lumen output
3200 maintenance factor is 0.7, coefficient of utilization is 0.46. Arranged in 4 rows and
4.bays. How many lamps in bays? iO ') / {'Cf.j
A. 8 M vi 3U C. 4/ r ® r —£T"
B- 6 * D. 10 Mo * *
x
11. The illumination in a room is being designed, the data are as follows :
Room length = 30 ft Room width = 20 ft
Coefficient of utilization (r|) 0.60 Maintenance factor (p) = 70 %
Required foot-candle = 50 Lumens per lamp = 3300
Watts per lamp = 53 MM* Supply Voltage = 220 V
Power factor = 75 %
What is the total current, of the lamps required to satisfy the requirements?
A. 21 Amperes —- r — C. 15 Amperes
B 7 Amperes D. 25 Amperes

12. A 150 candle power tamp give sufficient light to read a book at a distance of 6 feet. If
the lamp depreciates to 75 candle power, how much closer should the book be placed
to be read with same luminance in foot-candle.
A. 3.4ft x T C. 2.6ft -^ 5 , ^ ^
B. 2.2 ft : ft). 1.8 ft " 5

13. How much should a 100-Watt lamp be lowered to double the illumination on the object
which is 70 cm under it?
A, 20.5 cm 2£ ,
<4. C. 49.5 cm
B. 16.7 cm D. 2.42 cm tS <t>j s *
Gi - -j
14. The illumination on a certain object 2.5 m from the light source is 6,648 lux, using the
same light source, what is the illumination on the object at 5 m?
A. 1,226 lux p, 2 . p _ cA-2 c 1,662 lux
B. 3,324 lux &S' D. 4,700 lux

15. Two lamps of 20 cd and 40 cd are 10 m apart. At what point between them will the
illumination produced by them be equal?
A. 5.86 m from 20 cd lamp %o C. 4.14 m from 20 cd lamp
B. 4.14 m from 40 cd lamp D. at midpoint

16. Cavity factor is applied in which field?


A. Video system C. Audio system
B. Computer 6 j Illumination system
17. A classroom 15 ft x 20 ft x 10 ft height, having 2 ft ceiling cavity and 3 ft floor cavity.
What is the height of the room cavity for the said classroom?
A. 300 sq. ft 5 ft
B. 3,000 cu. ft D. 3 ft

18. A classroom 6m x 8m x 3.70m height, having 1.0m ceiling cavity and 0.75 m floor
cavity. What is the floor cavity ratio of the said classroom?
A. 1.95 _ L r <33 1.09
B. 1.46 3»Ve j ’S. D 2.84
~ 4 XM) )
19. VVhat component is necessary to ignite and lit up a preheat fluorescent luminaire?
A . Starter C. Capacitor
B. Ballast D. Tube holder

20. A choke coil (ballast) is generally used in series with the tube in the fluorescent lamp
circuit in order to _______.
A. prevent flickeringC. suppress radio interference
B. limit current to a proper valueD. improve overall power factor
21. The purpose of coating the fluorescent tube from inside with white phosphor powder is:
A. to improve its life
B. to improve the appearance
C. to change the colour of light emitted to white
£T\to increase the light radiations due to secondary emissions

22. The following lighting system are HID (High-lntensity-Discharge) general groups of
Jamp EXCEPT.
(A^ rapid-start lamp C. metal-hallide lamp
B. mercury lamp D. high-pressure sodium lamp

23. It is desired to flood-light the front of abuilding 42 m wide and 16 m high. Projectors of
30° beam spread and 1,000-Watt lamp giving 20 Im/watt are available. If thedesired
level of illumination is 75 lux and if the projectors areto be locatedat ground level 17 m
away. Estimate the number of lamps required.Assume thefollowing:coefficient of
utilization, 7 = 0.40, maintenance factor, /?= 0.77, waste light factor, co = 1.2
A. 12 ca * y (c: 10
B -i4 1 ----r.. D. 8
u*f I ' / *» ft* w. A
I*
24. _______is an establishment or a system for the production and modification of electric
energy.
A. Business establishments C. Electric plant
B. Industrial plant D. Commercial establishments

25. The Board of Electrical Engineer shall composed o f.


A. one chairman C. one chairman & two members
B. one chairman & one member D: three member

26. A Board of Electrical Engineering (BEE) will serve his term in how many years?
A) 3 C. 2
B. 5 D. 6

27. Vacancies in the Board shall be filled by th e ____ from the list of recommendees
selected by the commissioner who where chosen from the list of nominees submitted
by the integrated and accredited association for the un expired term only.
A. commissioner — rct/ 3vrsc.hr* C. board member
(B,- president D. IIEE tw m ' H

28. R.A. 7920 is a consolidation of House Bill No._____Senate Bill No.


A. 11063 & 1766 C. 16013 & 1667
B. 10163 & 1676 D. 13106 & 1776

29. Sec.2 (a)(6) Art. 1 of R.A. 7920 is .


A. Consultation, investigation, valuation and management of services requiring
electrical engineering knowledge.
B. Supervision of erection, installation, testing and commissioning of power plants.
C. Taking charge of sale and distribution of electrical equipment.
D. Teaching of electrical engineering professional subjects.

30. Sec. 4, Art. II states the powers and duties of the board. What does it mean by quasi­
legislative powers?
A. evaluation C. supervisory
B. control , D. rule-making
31. How many years does a high school graduate need to qualify for RME?
A. 4 yrs. C. 2 yrs.
B. 3 yrs. D. 5 yrs.

32. The E.E. Board Chairman when the R.A 7920 and Code of Ethics was approved.
A. Paul Joseph E. Woo C. Gerardo V. Gaor
B. Rafael F. Florentino , D. Roy F. Gil

33. Associate Electrical Engineer (Asso. EE) can apply for registration as Professional
Electrical Engineer (PEE) without a degree of BSEE?
A. True C. can apply
(B, False D. can interview

34. In what rule does R.A. 184 - Old Electrical Engineering Law (1947) stated?
A. Rule 2(e) - C. Rule 2 (p)^ *-c . <7«,
B Rule 2(f) D. Rule 2 (j)

35. Electric Power Industry Reform Act of 2001 (EPIRA) is ______.


A. R.A. 7832 ' C. R.A. 9136
B R.A. 7920 <i<c j •*j ■ R-A. 8495
t«wp 0 a
36. Calculation of reliability standards set by Philippine Distribution Code shall include
outages that occur on the_____.
A. Generation C. Secondary side of distribution
B. Transmission lines D. ^Primary side of distribution

37. If a tropical storm is going to the Philippine area of responsibility within 24 hrs, what
kind of alert status to be raised in accordance to Philippine Distribution Code?
A. Blue alert C. Weather disturbance alert
B. Yellow alert D. Red alert

38. It is a distribution reliability index defined as the total duration of sustained customer
•power interruptions within a given period divided by the total number of customer
served within the same period.
A. SAIFI - C. MAIFI *■ :“ -v i
SAID I - a O a> D. DAIFI
v' “'
39. Who is the board of trustee or board member of TRANSCO?
A. President of TRANSCO C. President of DENR
B. President of DOE Department of Finance

40. What refers to end-users who do not have choice of a supplier of electricity?
A. Contestable market C. Market operator
B Captive market D. Small power UtilityGroup

Revprobs Sept. 2013

4. Lire is aouui iv /w — 5 Maintenance wum j


ILLUMINATION, RA 7 9 2 0 «&IR R
SUPPLEMENTARY PROBLEMS
1. For indoor lawn tennis, what is the recommended level of illumination for tournaments?
A. 30 fc C. 20 fc
B. 10 fc (JD>50fc
2. Recommended illumination level of general office for reading.
A. 40 fc C. 30 fc
B. 100fc 70 fc

3. A light source emits a total luminous flux of 1000 lumens distributed uniformly over a
quarter of a sphere. What is the luminous intensity at 2.5 m?
A. 270 cd _ 0 C. 357 cd
BA 318 cd 1 •" D: 400 cd
4. A light meter reads the illumination received from the sun as 1x105 lm/m2. If the
distance from the earth to the sun is 1.5 x1011 m, compute the luminous intensity of the
sun.
A. 2.25 x1027 cd • -j. Q* ® ) C. 2.25 x1021 cd
B. 1.5x1021cd ' D. 1.25 x1027 cd
5. A 100 watt light bulb emits a total luminous flux of 1500 lumens, distributed uniformly
over a hemisphere. What is the illuminance at a distance of 2 m?
A. 10.6 lux +_ -s 46.8 lux
B. 21.2 lux ©I 59.7 lux
X 9. 's~r
6. A 100-Watt, 110 Volts lamp is rated 11.0 Im/watt. At what distance from the lamp is the
maximum illumination 10 lux?
A) 2.96 meters t C. 11.73 meters - -0
7.00 meters r s /SV r - 1 D. 1.91 meters

Given 800 lux, what is the lux when the distance to the point is doubled?
A - 300 X\ 9< 400 c U -
B. 100 ' ' 1 ' ' - DJ 200
.D. ' * r
8. The required illuminance for a given working plane is 100 foot-candles. Find the
illuminance “E “ in lux.
A. 1496 jf , V0. V * VU* C. 1204
B, 1075 * D. 1360
t ix<- •
9. 100 footcandle at 10 ft distance from point “P”. What is the intensity?
A. 40,000 1 C. 20,000
B. 30,000 1 * s ; ; ‘ ~~77vr ' ^ 10,000
■ V | 0 ^
10. A light source has a candle power of 100 in all directions. At a 10 ft distance from the
source, what illumination in foot-candle is being received on a plane perpendicular to
the source? ’ r r.
A. 2.5 C. 2
B. 1.5 r " ,R 1

11. A 150 candela lamp at a distance of 12.37 m gives a sufficient illumination on a certain
working plain. How far a 250 candela lamp be placed to have the same illumination as
before?
A. 9.58 m £ £» C. 49.5 m
B. 15.97 m ’ D 895m
12. A lamp giving 200 candelas uniformly below the horizontal plane is suspended 3 m
above a horizontal plane. Calculate the illumination at a point 3 m from the vertical
axis.
A. 7.66 tr . XV (j£ )7 .8 6
B. 7.75 ' cp D. 7.55

13. A street lamp 7 m above the sidewalk dissipates 200 Watts and has a luminous
efficiency of 60 lumens per watt. If the the source is assumed to be isotropic find the
illuminance at a point on the sidewalk 10 m from the base of the lamp.
/M 3.68 lux C. 4.52 lux
'~B. 6.42 lux t ~ D. 5.68 lux

14. A room 20 ft x 30 ft is illuminated by five 200-Watt, 3700 lumen incandescent lamps.


The average illumination intensity at the working plane as measured by a foot-candle
meter is 10 lumens/sq. ft. Calculate the utilization factor of the lamps.
fA. 0.32 . C. 0.23
B 0.45 D. 0.56

15. The illumination of a room is being designed from the following data:
Room length = 40 ft Room width = 24 ft
Coefficient of utilization= 0.60 Maintenance factor = 0.70
Required foot-candle =200 Lumens per lamp = 3,300
What is the number of lamps per row ? Provided the number of rows is 4.
A. 40 C. 30
. B'.- 35 D. 140
16. A 50 ft. by 20 ft office is to be illuminated with duplex fluorescent luminaires at level of
100 foot-candle. The maintenance factor and coefficient of utilization are estimated to
be 70% and 60% respectively. Each fluorescent produces 3300 lumens. Estimate the
total power if the wattage of each lamp is 53 watts
A. 1800 watts 'jSp 3800 watts
B. 2800 watts D 3500 watts
17. A 40 ft by 80 ft office is to be illuminated with duplex fluorescent luminaires at level of
60 foot-candle. The maintenance factor and coefficient of utilization are estimated to be
•75% and 65% respectively. Each fluorescent produces 3300 lumens. How any
luminaries are required per row if there are 4 rows ?
M 15 C. 30
'B . 13 D. 8

18. A room 40 m by 20 m needs to be illuminated by duplex fluorescent lamp at level of 60


cp. Each lamp has an output of 3300 lumens. Depreciation and maintenance factor are
0.65 and 0.75 respectively. If the lamp consumes 53 Watts, power factor is 75%, what
is the feeder current? Take supply voltage of 230 Volts, 3-phase.
A, 92 Amperes C. 75 Amperes
B j 57 Amperes D. 72 Amperes

19. A room measures 4ft by 60 ft needs 80 foot-candle of illumination. Each lamp has an
output of 3300 lumens. Depreciation and maintenance factor are 0.6 and 0.7
respectively. If the lamp consumes 53 watts, power factor is 75%, what is the feeder
current. Take supply voltage of 220 volts singtephase.
A. 36 Amperes 44 Amperes
B. 22 Amperes D. 26 Amperes
20. How much should a 100-W lamp be lowered to double the illumination on the object
which is 60 cm under it ?
O fr 17.6 cm C. 42.4 cm
B. 16.7 cm D. 2.42 cm
21. Two lamps 5 cd and 20 cd are 150 cm apart. At what point between them will the same
illumination be produced by each?
A. 50 cm from20 cd lamp C. 50 cm from 5 cd lamp
B. 100 cm from 5cd lamp D. 80 cm from 20 cd lamp

22. lyfe burning hours of a typical 40 W fluorescent lamp is _____.


A. 24,000 C. 16,000
B. 20,000 D. 12,000

23. Burning hours of incandescent lamp is about.


'Jk) 1500 C. 1800
-6. 750 D. 650

24. Average life of 28 V, 75 W tungsten halogen lamps is ______.


A - 2000 C. 4000
B. 1500 D. 500

25. Prior to starting of fluorescent, heating of electrode is done by__


A. Ballast C. Starter
B. Heater D. Filament

26. The direct lighting scheme is most efficient but is liable to cause _
A.) monotomy C. hard shadows
B7 glare D. both b) and c)

27. Estimate the number of 1000-Watt floodlight projectors required to illuminate the upper
75 m of one face of a 96 m tower of width 13 m if approximate initial average
illumination is to be 86 lux. The projectors are mounted at ground level 51 m from the
base of the tower. Utilization factor is 0.2 and efficiency of each lamp is 18 Im/watt.
A. 22 £ £ ) 24
' B. 25 D. 21
28. It refers to choice of electrical systems, including planning and detailing of
requirements for protection, control, monitoring, coordination and interlocking of
electrical systems among others.
A. system utilization design C. power plant design
B. practice of electrical engineering . 0. Electrical system design

29. Article III contains:


A. Sec. 3 to 6 C. Sec. 10 to 20
B. Sec 10 to 30 D. Sec. 10 to 28

30. Sec.17(a), of R.A. 7920 : Examination and registration qualifications of applicants for
Registered Electrical Engineer states that an applicant is a _____of the country
A. leader C. citizen
B. professional D. known person

31. On what particular section in article IV of the RA 7920 is Foreign Reciprocity stated?
A. Sec. 39 s ® Sec. 38
B. Sec. 40 D. Sec. 37
32. According to Sec. 12 of R.A. 7920; all applications for oral examinations for PEE and
written examinations for REE and RME shall be subject to payment of fees prescribed
by the commission: Provided, th a t__% of the fees is to be treated as special fund for
programs, projects and activities of the commission and the remaining__% shall be
set aside as a trust fund for the establishment and maintenance of the center for
continuing education and research.
A. 80,20 B. 85,15 C. 90, 10 D. 95,5
33. For a person to be considered to be a member of the Board of Electrical Engineering he
must, at the time of appointment: be a natural-born Filipino citizen and a resident of the
Philippines for at least____consecutive yrs; be atleast____ yrs of age, of proven
integrity with high moral values in his personal and professional conduct; have practiced
electrical engineering for a period of not less than____yrs prior to his appointment.
A. .5,45,10 B 5, 35, 10 C. 10, 35, 5 D. 5, 45, 15
34. Under Penalty Clause: Any person who shall violate any of the provision of this Act
shall be guilty of misdemeanor and shall, upon conviction, be sentenced to a fine:
A. not more than P60,000 C. not more than P10,000
B. not more than P20,000 D not more than P50,000

35. Rule 4, Article lll:The Board of Electrical Engineering in the enforcement and
administration of Republic Act. No 7920, and in the execution and performance of its
powers, duties, functions and responsibilities set forth therein shall rely or seek the
review, approval, supervision, assistance, support and/or advice of th e ____ through
the latter’s office, divisions sections and units.
A. TESDA B. Commission C. CHED D. DECS
•V y ^ *■
36. In Philippine Distribution Code Performance Standards, Which of the following does
NOT belong?
A. Collection standards C. Reliability standards
B ' Safety standards D. System efficiency standards
37. In Philippine Distribution Code Purposes, Which of the following does NOT belong?
A. To ensure quality of electric Power
B. To specify customers services for the protection of end users
C. To specify standards for the protection of personnel in the work environment
p . To ensure distribution system will be operated by only one distribution company
38. What is SoLR from Philippine Distribution Code (PDC)?
A, Supplier of last resort C. Supplier of load resort
B. Source of load resort D. Source of line reserve
39. In Philippine Distribution Code Power Quality Standards, power quality problem exist in
any of the following EXCEPT:
A. The system frequency has deviated from nominal value of 60 Hz
B. Harmonic frequencies are present in the system
C. Voltage magnitude are outside their allowable range of variation
D. Phase voltage magnitude are equal and 120 degrees out of phase
40. What is the meaning of WESM?
A. Wholesale Electricity Spot Market C. Wholesale Energy Saving Market
B. World Energy Spot Market D. World Electricity Saving Market

P r a c tic e m a k e s p e r f e c t !
Supplprobs. Sept. 2013
P,EC 2009. ETHICSI
This Free Quality Manual is
Found only at POWERLINE

POWERLINE REVIEW CENTER


M E Z Z A N IN E F L O O R , D O N A A M P A R O B U IL D IN G
C O R N E R E S P A N A & G. M . T O L E N T IN O S T R E E T S
S A M P A L O C , M A N IL A
T E L . N O S . 7 3 5 -7 3 -0 2 & 7 3 3 -2 1 -1 8

T E L . N O S . (0 3 )2 6 1 -2 2 4 -4 & (0 3 2 ) 2 6 1 -8 4 5 2
1
POWERLINE REVIEW CENTER, INO.
P.E.C. 2009, Obligation 8cContracts R eview L ectu re

Part I: COPE - A set of requirements (rules)


The P.E.C, 2009 consists of two (2) parts :
PART I (volum e 1) - Consists of rules which regulates electrical installation done “inside” the building.
PART I (volum e 2) - Consists of rules which regulates electrical installation applied to “watercraft”.
PART II - Consists of rules which regulates electrical installation done “outside” the building.

Mandatory Rules. Permissive Rules* & Explanatory Material :


1.Mandatory Rules - Mandatory rules of this Code are those that identify actions that are specifically
required or prohibited and are characterized be the use of the terms “shall ” or “shall not "
2. Permissive Rules - Permissive rules of this Code are those that identify actions that are allowed but
not required, are normally used to describe options or alternative methods, and are characterized by the
use of the terms “shall be permitted’ or “shall not be required *\
3. Explanatory Material - Explanatory material, such as references to other standards, references to
related sections of this Code, or information related to a Code rule, is included in this Code in the form
of fine print notes (FPN). Fine print notes are informational only and are not enforceable as
requirements of this Code.
Purpose ofP.E X . 2009 ;
1. Practical Safeguarding - The purpose of this Code is the practical safeguarding of persons and
property from hazard arising from the use of electricity.
2. Adequacy - This Code contains provisions that are considered the minimum requirements necessary
for safety. Compliance therewith and proper maintenance will result in an installation that is essentially
free from hazard but not necessarily efficient, convenient, or adequate for good service or future
expansion of electrical use.
3. Intention - This Code is intended for exclusive use of licensed electrical practitioners (PEE, REE and
RME). This Code is not intended as a design specification nor instruction manual for a non-licensed
electrical practitioner, unless under the supervision of a licensed electrical practitioner
4 Relation to Other International Standards - The requirements in this Code address the
fundamental principles of protection for safety contained in Section 131 of International
Electrotechnical Commission (IEC) Standards 60364-1, Electrical Installations o f Buildings.
FPN: IEC 60364-1, Section 131. Contains fundamental principles of protection for safety that
encompasses protection against thermal effects, protection against overcurrent, protection against
fault currents, and protection against overvoltage. All of these potential hazards are addressed by
the requirements in this Code.

Who enforces the P.JELC, 2009 ?


1. This Code is intended for mandatory application by government bodies exercising legal jurisdiction
over electrical installations.
2. These government bodies, only through a licensed electrical practitioner, shall have responsibility of
implementing the provisions of this Code in deciding on the approval of equipment and materials and
for granting special permission contemplated in this Code, where it is assured that equivalent
objectives can be achieved by establishing and maintaining effective safety.
3. The authority having jurisdiction may waive specific requirements in this Code or permit alternate
methods where it is assured that equivalent objectives can be achieved by establishing and maintaining
effective safety
4. This Code may require new products, constructions , or materials that may not yet be available at the
time this Code is adopted. In such event, the authority having jurisdiction may permit the use of the
products, constructions, or materials that comply with the most recent previous edition of this Code
adopted by the jurisdiction
2
Who authorized the P.E.C. 2009 ?
1.
This Code has been approved and adopted by the Board of Electrical Engineering, (BEE), Professional
Regulation Commission (PRC).
2. By virtue of authority vested in the Board under RA 7920, it hereby directs strict adherence to the
provisions of this Code.
3. Where deviations from these provisions are necessary, such deviations shall not be made, except with
written permission from this government bodies exercising legal jurisdiction applicable only to the
particular job for which such permission was granted.

What are the coverage or scope of P.EX\ 2009 ?


Covered: This Code covers the installation of electrical conductors, equipment, and raceways; monitoring,
signaling, and communication conductors, equipment, and raceways; and optical fiber cables
and raceways installed within or on, to or from:
1. Public and Private Buildings, including but not limited to residential, commercial, industrial,
institutional, cultural, agricultural, agro-industrial, planned unit development and all other
building/premises that may require practical safeguarding of persons and property from hazards arising
from use of electricity.
2.. Electric generating plants 9. Quarries and mines
3. Industrial Plants 10. Watercraft
4. Transformer stations 11. Dockyards
5. Permanent and temporary substations, etc. 12. Trailers
6. Airfields 13, Mobile homes and recreational vehicles
7. Railway switchyards 14. Offshore facilities
8. Yard, Carnivals, parking and other lots.
Not Covered: This Code does not cover the following:
1Installations in railway rolling stock, aircraft, or automotive vehicles
2. Installations of railways for generation, transformation, transmission, or distribution of power used
exclusively for operation of rolling stock.

What Covers Electrical Inspection ?


Inspection - criteria for examination of equipment for :
1. Suitability for the installation
2. Mechanical strength and durability
3. Wire bending and connection space
4. Electrical insulation
5. Heating effects produced by the equipment under normal and abnormal condition of use.
6. Arcing effects.
7. * Class, type, size, voltage, ampacity, specific use of equipment.

SERVICE - the conductors and equipment for delivering energy from the electric supply system
to the wiring system of the premises served. Which consists o f:
FOR OVERHEAP SERVICE:
1 SERVICE DROP - the overhead service conductors from the last pole or other aerial support
and including splices if any, connecting to the service entrance conductors in a building or other
structures, And shall not be smaller than 8 mm 2 Cu or 14 mm2 A1 or Cu-clad Al.
/ Clearances of Service Drop:
a) Above roof - 2,400 mm
b) Vertical Clearances from ground :
• 3,000 mm - at electric service entrance to buildings, also at the lowest point of the drip loop.
• 3,600 mm - over residential property and driveways, and those commercial areas not subject
to truck traffic where voltage does not exceeds 300 Volts to ground.
• 4,500 mm - for those areas listed in the 3,700 mm classification where the voltage exceeds
600 Volts to ground.
• 5,500 mm - over public streets, alleys, roads parking areas subject to truck traffic
3
c) Clearance from building opening - 900 mm from windows that are designed to be opened.
d) Clearance from swimming pools-6,900 mm
in any direction from water level, (see table 6 80.1.8)
2. SERVICE ENTRANCE - the service conductors between the terminal of the service equipment and
a point usually outside the building, clear of bldg. walls, where joined by a taps or splices to the
service drop, and snail be of size adequate to carry the load intended to serve.
Note: A building or other structure served be supplied by one service only.
EXCEPTIONS: For the following:
(a) Special Conditions r
1. fire pumps (high pressure) for the building
2. Emergency electrical systems
3. Legally required stand-by systems
4. Optional stand-by systems
5. Parallel power production systems
6. Systems designed for connection to multiple sources of supply for the purpose of enhance
reliability.
(b) Special Occupancies
1. Multiple-occupancy building where there is no available space for service equipment
accessible to ail occupancies.
2. Multiple-occupancy building or group of singledetached buildings owned or
operated/managed by one (1) person or entity andwhere spaces areavailable, amain
service shall be allowed to serve the service entrance equipment of each occupant and
common loads, if any.
3. A single building or other structure sufficiently large to make two or more services
necessary.
(c) Capacity Requirements
1. Where the capacity requirements are in excess of 2000 Amp at a supply voltage of 600
Volts or less.
2. Where load requirements of a single-phase installation are greater than the serving
agency normally supplies through one service.
(d) Different Characteristics
Additional services shall be permitted for different voltages, frequencies, or phases, or for
different uses, such as for different rate schedules.
(e) Identification
Where a building or structure is supplied by more than one service, or any combination of
branch circuits, feeders, and services, a permanent plaque or directory shall be installed at
each service disconnect location denoting all other services, feeders, and branch circuits
supplying that building or structure and the area served by each (see Section 2.25.2.8)

3. SERVICE EQUIPMENT - the necessary equipment, usually consisting of a circuit breakers) or


switch(es) and fuse(s) and their accessories, connected to the load end of service conductors to a
building or other structure, or an otherwise designated area, and intended to constitute the main
control and cutoff of the supply. This shall be located inside or outside wall of the building served
or to the nearest point of entry of non-building structure served.

B. FOR UNDERGROUND SERVICE :


1. SERVICE LATERAL - the underground service conductors between the street main including any
risers at a pole or other structure or from transformers, and the first point of connection to the
service-entrance conductors in a terminal box or other enclosure, inside or outside the building
wall. Where there is no terminal box, meter, or other enclosure, the point of connection is
considered to be the point of entrance of the service conductors into the building, and shall not be
smaller than 8.0 mm2 Cu & 14 mm2A1 or Cu-clad Al.
Clearances of Service Lateral : (from ground, see Table 3.0.1.5)
• 600 mm- directly buried cable
• 150 mm or
- RMC IMC installation
• 450 mm - PVC approved for direct burial without concrete encasement.
• 450 mm- other approved raceway for direct burial.
4
2. SERVICE ENTRANCE (Underground) - the service conductors between the terminals of
the service equipment and the first point of connection to service lateral.
3. SERVICE EQUIPMENT - (Same definition as for overhead service equipment)
Note: When distribution is overhead, the utility company usually secures the service drop conductors
to the building. All design and construction beyond this point is the responsibility of the owner, and the
PEC regulations now apply. If distribution is from an underground system, the underground service
installation is usually made by the utility company.
NOMINAL VOLTAGE - the lowest ratings of voltage under which a load can operate under normal condition.
Total voltage drop allowed: 5 %, For Feeders: 3 % and for Branch circuits: 2 - 3 %
FEEDER - A set conductors that supply a group of branch circuit over current devices.
BRANCH CIRCUIT - A set of conductors that extend beyond the last over current device in the system.
SWITCHBOARD - A large frame assembly that contains a number of over current device for feeder. It
may also contain instruments and may be accessible from both front and rear.
PANELBOARD - A smaller assembly of over current devices, contained in a cabinet and accessible only
from the front. The over current devices up to a maximum of 48 (excluding main CB), in this case 24
branch circuits (or feeders) for l(j) and 16 branch circuits (or feeders) for the case of 3(J>.
A P P R O V E D T Y P E O F W IR IN G M E T H O D B Y P .E X . :
A) Conductors on insulator Method :
1. Open wiring on insulator - installed exposed. •
2. Concealed Knob and tube work- installed hidden by portion of building. •
B) Cable Wiring Method :
1. Armored (BX) cable. • 6. Service entrance cable.
2. Metal- clad cable. 7. Underground feeder & branch ckt. cable
3. Mineral insulated metal sheathed cable. 8. Power and control tray cable.
4. Non-metallic sheathed cable. 9 Flat conductor cable.
5. Shielded non-metallic sheathed cable. 10. Medium voltage cable.
11. Integrated Gas spacer cable.
C) Raceway Method :
1. Intermediate metal Conduct (IMC) • 11. Under floor raceways
2. Rigid metal conduit (RMC) • 12. Cellular metal floor raceway
3. Rigid non-metallic conduit (PVC) • 13. Cellular concrete floor race way
4. Electrical metallic tubing (EMT) • 14. Wireways •
5. Flexible metallic tubing ( FMT) • 15. Busway s •
6. Liquid-tight flexible metallic tubing 16. Cable trays •
7. Liquid-tight flexible non-metallic tubing 17. Cable bus
8. Metallic surface raceway • 18. Flexible metallic conduit •
9. Wooden surface raceway • 19. Electrical non-metallic tubing
10. Non-Metallic surface raceway • 20. Multi-outlet assembly
• The most commonly used methods done by Practitioners.

M E T H O D S O F C O M P U T A T IO N F O R W IR IN G D E S IG N
A. Design Load Method:
• 100 VA per lighting & convenience receptacle (LO)
• 180 VA or 200 VA per convenience outlet (CO)
B. Area Method:
• 24 VA per m2 for residential excluding toilet & porches.
• 28 VA per m2 and 8 VA per m2 for office general lighting and office general purpose
receptacle outlets respectively.
• 16 VA per m2 for industrial plant.
Note: A single residence with 10 kW or more initial load requires a 100 Amp service. Lighting outlet
shall be separated from the circuit of convenient outlet except in occupancy where the total
connected load does not exceed 15 Amp.
C. For Motor Loads i
• Ampacity of conductors: Use 125 % of the FLA ( ftill-load ampere)
• Rating of branch over-current protective device :
Use: 250 % of FLA for Thermal Magnetic CB
300 % of FLA for Non-time delay fuse.
N ote: The above ratings were based on full voltage or across the line starter and can go lower to
150 % of FLA and 200 % of FLA for CB & Fuses respectively if starters were used.
HAZARDOUS LOCATION:
* Class I - Locations in which flammable gases or vapors are or may be present in the air
sufficient to produced explosive or ignitable mixtures.
( e.g. gasoline terminals, distillery, pharmaceuticals etc.)
2. Class II ~ Locations m which are hazardous because of the presence of combustible dust.
( e.g. firecracker maker, match maker, dynamite plant etc.)
3. Class III - Locations in which are hazardous because of the presence easily ignitable fibers
and fyings. ( e.g. textile plant, paper, saw mills etc.)
GRQLNDING SYSTEM
1. System grounding - a system or circuit conductor that is intentionally grounded.
2. Equipment grounding - the conductors used to connect the non-current carrying metal parts
of equipment to the system ground conductor at the service equipment.
3. Grounding conductor - a conductor used to connect equipment to grounded circuit of a
wiring system to a grounding electrode.

Part II - OBLIGATION and CONTRACTS


1. Define OBLIGATION
it is the juridical or legal necessity to give (a thing), to do or not do (a service).
2. W hat are the requisites o f a valid obligation ?
(requirements to establish a legal obligation)
a. active subject - the party that can demand that the obligation be done or fulfilled; sometimes
referred to as “ creditor or obligee
b. passive subject - the party to fulfill the obligation; sometimes referred to as “ debtor or obligor
c. object or prestation - the subject matter of the obligation.
d. legal or juridical tie ~ this binds the parties to the obligation; sometimes referred to as the
efficient cause or vinculum or source o f obligation.
3. W hat are the sources o f obligation (legal tie) ?
a. from law - example ; taxes
b. from contract - as long as the contract is legal
c. from quasi-contract - relation between parties from lawful, voluntary and unilateral acts.
No formal agreement, but valid to prevent injustice viz.
- voluntary management of another person’s property creating the obligation for proper
remuneration (negotiorum gestio)
- obligation to return things or objects delivered by mistake and there is no right to demand
(solution indebiti)
d. from delicts - crime or omission punishable by the Law.
e. From quasi-derelicts - omission due to negligence or accideul.
4. Who will be liable for damages in cases o f quasi-derelicts ?
a. the person himself (if not incapacitated)
b. in cases of minor, the father then the mother
c. for employees/workers on official works, the employees
5. W hat are the two (2) general classification o f obligation ?
a. natural b. civil
(once the natural obligation is put into a formal agreements, it becomes a civil obligation)
6
6. What is meant by fortuitous event /"fo rce m ajeure" ?
events that can not be predicted or prevented by man
furfural: typhoon, earthquake, flood, etc man-made : riot, strike, war, etc.
7. What are the d ifferen t kinds o f (civil) obligation ?
a. pure - no conditions attached
b. conditional - with conditions attached
c. with a period - covered by a period or time
d. alternative - specifies several prestation andthe debtor or passive subject canselect
e. facultative - specific object but substitution may be allowedas agreed upon by the parties
f. divisible - can be performed partially
g. joint-to be fulfilled proportionately by the different debtors
h. solidary - each debtor is liable for the whole obligation
i. others terms - jointly and severally
j. with penal clause - to insure performance liability is provided in case of breach
8 . How are obligation be extinguished or se ttle d ?
a. by payment or performance - total substantial or incomplete as long as acceptable to the creditor
b. by loss of the thing due - if the object disappears and can not be recovered
(Note: applicable only if the object is specific and not “generic”)
c. condonation or remission of debt - the creditor abandons his right to collect.
d. confusion or merger - the quality of the creditor and debtor merges into the same person
e. compensation or set-off- equivalent or the same amount of the obligation.
f novation or substitution - a third party substituted to the debtor and accepted by the creditor
g. others, (i.e. death in case of personal obligation, compromise agreement, fortuitous event,
decion- en- pago, cession)
9. To whom should the payments o f deb t be made ?
a. the creditor
b. the successor -in-interest (or heir)
c the authorized persons
10. W hat is the e ffe c ts o f paym ent to unauthorized or incapacitated persons ?
payment is valid only up to the amount received by and / or beneficial to the creditor
11. How may debt in money be se ttle d ?
By payment in the agreed currency or the legal currency (PESO in the Philippines)
(Note: P 1.00 to P 0.10 coins are legal tender only up to P 50; P 0.05 or less for P 20 or less)
12. W hat is "consignation" o f payment ?
assignment of payment to the Court of Law to protect the debtor against claims for damages/
default from the creditor.
a. creditor unreasonably refuse to accept payment
b. creditor is absent or unknown on time of payment
c. without just cause, creditor refuses to accept payment
d. two or more persons claim the same right to collect payment
e. title of obligation is lost
13. What is a CONTRACT ?
meeting of minds between two persons, where one binds himself, with respect to the other to
give some thing of render some service.
14. COMPARE contracts with obligations
contract is the agreement while obligation is the remedy that the law affords for the enforcement
of said agreement.
contract is a source of obligation
15. What are the elem ents o f a contract ?
a. essential element - without which the contract is not valid, viz. consent, object and cause
b. natural element - those presumed to exist in some contracts “unless” excluded, viz. warranty
c. accidental element - terms and conditions agreed upon by the parties, (viz. interests, damages
and the like).
16. What are characteristics o f a contract ?
a. obligatory force - once perfected, the parties have respective obligations to fulfill
b. autonomy - parties are free and have the legal capacity to enter into the agreement
c. mutuality - both parties agreed on the terms and conditions of the contract
d relativity - contract is effective only between the contracting parties, their heirs or assigns
17. What are the stages o f a contract ?
a• Preparation or conception or generation
parties are negotiating towards an agreement or contract
& Perfection or birth
parties have signed the contract or have agreed on the contract.
c. consummation or termination
parties have fulfilled their respective obligation or have agreed to cancel the contract.
18. When can consent in a contract be considered valid ?
a. when consent is intelligent
b. when consent is free and voluntary
c. when consent is conscious and spontaneous
19. When is consent "intelligent" ?
- When the party is NOT a minor, nor INSANE, nor DEAF and MUTE who can not reador write.
20. When is consent M
free and voluntary"?
When consent is obtained/given without the use of FORCE, VIOLENCE or INTIMIDATION.
21. When is consent "Consious and Spontaneous" ?
When consent is obtained/ given without the use of FRAUD, UNDUE INFLUENCE or there is
error of MISTAKE.
22. When can the o b ject/s o f a contract considered NOT VAUO ?
a. when it is not within the commerce-of-men
examples : plaza, street, river, parks, etc
b. when it is contrary to law, morals , good customs, public order or policy
c. when it is physically and legally impossible
d. when it is not determinate as to its kind
23. What is the difference between onerous and gratuitous contracts ?
in onerous, contract, both parties are benefited while in gratuitous, only one party derived -
benefit, example; donation
24. W hat is the difference between unilateral and bilateral contracts ?
in unilateral, only partylias the obligation in the contract, while in bilateral, both parties hsve
respective obligations.
25. W hat are th e defective contracts ?
a Recissible c. Unenforceable
b. Voidable d. Void or Incxistent
RESCISSIBLE CONTRACTS- have the essential elements and valid but may be set aside on the
basis of justice or equity.
if the affected party suffer damage by more than 25% of the value of the object,
when there is fraud and the creditor cannot in any manner collect the claims due them,
those things under litigation if entered by the defendant without the knowledge and approval
of the litigants or the Court
VOIDABLE CONTRACTS- one of the parties is incapable of giving consent, or consent is
vitiated by mistake , fraudundueinfluence, force, violence or intimidation.
UNFORCEABLE CONTRACTS-r
a unauthorized contracts- without or in excess of authority of contracting party/ ies
b. both parties are incapable of giving consent
C. not complying with the statute of Fraud.
/ s/ ; \V \
POWERLINe r e v i e w c e n t e r Inc.
/ P O W E R U K E \
T h e I_ J Itim a te E .E . R e v ie w C e n t e r
P .E .C .2 0 0 9 REVIEW PROBLEMS /QUESTIONS

1- ____ are used to provide explanatory material, references to other sections,


precautions to be observed, and further mandatory requirements.
A. mandatory rules c ) fine print notes
B. exceptions D. listings
2. Legally required standby systems are typically installed to serve toads such as _____.
I. -Sewerage disposal
II. Data processing
III. Refrigeration system
A. I and II only C. I and III only
B. II and III only D I, II and III
3. According to Philippine Electrical Code, at ambient temperature of 30°C, 8.0 mm2 copper
conductors with the following insulation have the following ampacities: TW insulation= 30
Amp; THW insulation= 45 Amp; THHN insulation= 50 Amp. If each of these insulated
wires carry the same 40 Amp, which will generate heat the fastest?
A. Each will generate same heat C. THHN insulated wire
B. THW insulated wire D. TW insulated wire
4. Code name for polyethylene insulation or PVC 75°C is?
A. TW $ THW
B. TF D THHN
5. Which is the safest type of insulation for indoor wiring installation especially residential
and commercial buildings?
A. XLPE q ) Polyvinyl chloride
B. Polyethylene D. Ethylene propylene rubber
6. What is the advantage of PVC over Polyethylene ?
A. .moisture resistance C. flame retardant
B. temperature resistant D. weather resistant
7. What is the minimum size of wire in lighting outlet?
A. 3.5 sq. mm C 2.0 sq. mm
B. 5.5 sq. mm D. 8.0 sq. mm
8. What is the maximum bend of Rigid Metallic Conduit (RMC)?
A. 2 quarter C. 3 quarter
B. ?4 quarter D7 5 quarter
9. What is the required Volt-Amp for lighting of an office building with 500 sq mts area?
A. 16,500.' C. 23,500
B. 14,000 D. 50,000
10. A single family dwelling unit having a floor area of 45 sq. meter has a total connected toad
of 2575 VA. What is the minimum size service entrance conductor?
A. 2.0 sq. mm C. 5.5 sq. mm
B. 3.5 sq. mm D. 8.0 sq. mm
11. Sign and outline lighting outlets shall be computed at a minimum of VA for each
required branch circuit.
A. 1500 C. 600
B. 180 D. 1200
12. A 3-way and 4-way switches shall be so wired that all switching is done in th e __
conductor.
A) ungrounded C. neutral
B. grounded D. grounding
13. What is the demand factor for a single 8 kW household electric range?
A. 65% C. 90%
B 80% D. 70%
14. What is the branch circuit demand for two-14 kW range in a residence of 1-phase, 230
Volt?
A. 19.8 kW C. 12.1 kW
B. 14.4 kW D. 28 kW
15. Calculate total load for a 1-phase, 230 Volt branch circuit supplying two-3kW and two-
4kW wall-mounted oven.
A. 60.9 Amp C. 42.2 Amp
B. 48.7 Amp D. 34.4 Amp
16. For dwelling unit having a floor area more than 150 sq. meters, the feeder and service
loads shall be computed with_____percent demand factor on the air-conditioning load
and the lighting load.
A. 50 C. 100
B. 125 D. 40
17. What is the neutral load for a 1(j> system having 150 Amp of electric-discharge lighting
and 250 Amp of incandescent lighting?
. A. 325 Amp (5) 385 Amp
B. 280 Amp D. 400 Amp
18. What is the maximum current allowed on a # 12 AWG (3.5 sq.mm) THHN copper
conductor in an ambient temperature of 122°F (50°C) with a total of six (6) current-
carrying conductors in the conduit? (use: derating factor of 80% for 6 conductors and
correction factor of 0.82 at 50°C)
A. 24.6 Amp C. 32.8 Amp
B. 19.68 Amp D. 20 Amp
19. What size of wireway is required for 6-# 2/0 THW @ 0.2781 in 2 each, 8-# 6 THW @
0.0819 in2each and 22-# 12 THWN @ 0.0117 in 2 each copper conductors?
A. 4 in. x 2 in C. 3 in. x 3 in.
B 4 in. x 4 in. D. 3 in. x 2 in.
20. What is the maxi~*_~ allowable distance interval between supports securing busways?
A. 1,000 mm C. 2,000 mm
B. 1,800 mm D. 1,500 mm
21. What is the current-carrying capacity of each of the three copper bus-bars. The cross-
sectional area of each is 1.30 cm x 2.55 cm, if each bus-bar were replaced by aluminum,
what is the corresponding current-carrying capacity?
A. 415 Amp & 316 Amp C> 514 Amp & 361 Amp
B. 500 Amp & 350 Amp D. 400 Amp & 325 Amp
22. What is the minimum ampacity of conductor for room air conditioner with 12 Amp rated
current?
A. 10 Amp
B. 12 Amp
23. What is the minimum ampacity of feeders with motor and other load connected?
A. 250% motor FL current plus other current' C. 125% total current
B. 125% motor FL current plus other current D. 250% total current
24. A hermetic motor-compressor has a rated load current of 20 Amp. What is the allowable
ampacity of branch circuit conductor?
40 Amp C. 30 Amp
B. 20 Amp D. 25 Amp
25. A hermetic motor-compressor has 25 Amp full load current. What is the rating of
disconnecting means?
A. 31.25 Amp r§) 28.75 Amp
B. 31.75 Amp D. 25 Amp
26. Three squirrel-cage induction motors rated as follows: 25 HP, 34 Amp full load,20 Hp,
27 Amp full-load, and 10 Hp, 14 Amp full-load all rated at 460 volts, 3cj>, 60 Hz are to be
served by a feeder circuit for an ambient temperature of 30°C. What is the net computed
current for feeder conductors and using non-time delay (NTD) fuse, what is the largest
protective device for feeder circuit protection?
A. 83.5 Amp & 141 Amp resp. C. 86.3 Amp & 145 Amp resp.
B. 105.6 Amp &125 Amp resp. D. 90.5 Amp & 121 Amp resp.
27. A generator has a nameplate rating of 30 Amp, what is the minimum ampacity of
conductor from generator terminals to the first overcurrent device?
A. 30 Amp Q- 34.5 Amp
B. 37.5 Amp D. 60 Amp
28. The primary winding of a step down transformer shall be protected on the primary side
by an overcurrent device rated NOT more than a certain percentage of the primary
current. What is this percentage?
A. 110% C. 125%
B. 135% D. 150%
29. The ampacity of capacitor circuit conductors shall not be less than_____percent of the
rated current of the capacitor.
A. 135 C. 150
B. 125 D. 167
30. On solar photovoltaic system; ampacity of conductors and overcurrent devices shall not
be less than_____percent of the computed current.
A. 150 C) 125
31. P.E.C. requires that the minimum area exposed surface affected by a plate electrode is:
A. 1/6 sq.mts CJ 1/5 sq.mts
B. 1/4 sq.mts D. 1/3 sq.mts
32. P.E.C. requires that resistance to ground of a single electrode consisting of rod, pipe or
plate shall be______..
A. 30 to 50 ohms C. above 100 ohms
B. 25 ohms or less D. 50 to 100 ohms
33. What is the minimum size of grounding electrode copper conductor?
A. 3.5 sq. mm c. 5.5 sq. mm
B. 8.0 sq. mm D. 14 sq. mm
for circuit protected by 30 Amp circuit
breaker?
A. 2.0 sq. mm C. 3.5 sq. mm
i3. 5.5 sq. mm D. 8.0 sq. mm
35. What is the minimum size of grounding rod?
A. 10 mm C. 13 mm
B. 12 mm D. 16 mm
36. The main purpose of bonding jumper is to connect equipment grounding conductors
and_______to the grounded conductor of the system.
A. supply transformer C. load side of service drop
B.
' ^
service-disconnect enclosure D. line side of service drop
37. Which of the following is a standard rating of fixed-trip inverse time CB?
A. 50 C. 750
B. 1500 D. 75
38. The maximum number of overcurrent devices that may be installed in a lighting panel is:
A. 24 C. 42
B. 48 D. 36
39. What is the maximum time of delay permitted for the GFCI (Ground Fault Circuit
Interrupter) to operate where the ground-fault current is 3000 Amp?
A. >2 sec C. 100 msec
B. 1 sec D. 2 sec
40. Where transformer vault is to be constructed with other buildings below it, the minimum
requirement for fire resistance rating is____ hours.
A. 4 C. 2
B. 1 1/2 D. 3

RevProbs Sept.2013
P.E.C. 2 0 0 9 , CODE o f ETHICS SUPPLEMENTARY
PROBLEMS

1. Name the voltage class when the nominal voltage is 120-600 Volts.
A^LowVoltage C. High Volage
B. Medium voltage D. Extra High Voltage
2. The short circuit temperature rating of PVC insulated building wires for 30 second period
A. 200°C C. 300°C
B. 250°C (IS) 150°C
3. Common cause of failure in termination of cable is_______________.
C^y^Proper insulation & penciling C. Wrong clearance
B. Splices D. Tracking
4. Before installation of power cables the basic DC test voltage in ascending mpre
according to ICEA (Insulated Cable Engineer Association) specification for 100%
insulation level in terms of rated voltage is not above?
(fcP 5 times C. 4 times
B. 3 times D. 2 times
5. What is the normal bending radius from inner edge of conduits with relation to diameter?
X. 8-9 times C. 4-6 times
B. 6-7 times D. 3-4 times
6. The breakdown voltage of Insulation depends upon_____value.
A. RMS C. average
R.'>peak D. twice the RMS
7. Short circuit temperature of Polyethylene cable in degree centigrade is.
A. 200 C. 300
250 D. 350
8. What is the minimum distance receptacle outlet of a show window in meters?
C. 2
B. 4 D. 5
9. Receptacle outlet shall be located not more than____ mm above countertop.
A. 300 C. 250
5X50 D. 600
10. Branch circuit larger than____Amp shall supply only non lighting outlet load.
WC 50 C. 20
B. 30 D. 15
11. What is the distance from the flooring to the receptacle outlet in dwelling unit in mm?
X 1800 C. 1700
B. 3700 D. 4000
12. In dwelling units, at leastone wallreceptacle outlet shall be installed within_mm of
the outside edge of basinor bath tub.
A. 1200 mm C, 600 mm
B. 1600 mm jo. 900 mm
13. Injjwelling units, at least one receptacle outlet for hallways more than meter.
3 C. 2
B. 4 D. 5
14. The lighting load for a dwelling unit expressed in terms of volt-ampere per sq. meter
must be at least_____.
A. 16 24
B. 28 . D. 12
15. The lighting load for an office expressed in terms of volt-ampere per square meter must
be,at least_____ .
ty 28 C. 16
B. 12 D. 24
16. Wh^t is the general lighting load for industrial?
p / 16 VA/sq. m C. 6VA/sq. m
B. 28 VA/ sq. m D. 24 VA/sq. m
17. What is the branch circuit demand for a 12 kW range and a 5 kW cook top in a residence
of 230 Volt, 1-phase ?
11kW C. 13 kW
B. 17 kW D. 14.6 kW
18. The ampacity of a #10 AWG (5.5 sq.mm) TW copper conductor when there are not more
than three (3) conductors in the raceway and the ambient temperature is 97°F (36°C)
would be: (use correction factor for given the conductor of 0.82 at 36°C)
A 22 Amp C. 30 Amp
.JK 25 Amp D. 27 Amp
19. In rigid metal conduit wiring, conduits shall be supported at least every______.
A. 2000 mm C. 2500 mm
Br 3000 mm D. 3500 mm
20. One of the following raceways NOT suitable for installation in wet locations? ,
A. rigid non-metallic conduit - C. rigid metal conduit
B. intermediate metal conduit Q/flexible metallic conduit
21 Feeder supply several motor load shall have an ampacity equal to the sum of the full
load/6urrent rating of all the motors plus____% of the highest rated current in the group.
t y 25 C. 100
o. 80 D. 125
22. A 3<|> motor draws 50 Amperes at full-load. What shall be the maximum inverse time
circuit breaker that may be used for short circuit protection ?
A. 50 Amp C. 100 Amp
£ /1 25 Amp D. 75 Amp
23. Branch circuit conductor to one or more units of a data processing system shall have an
ampacity of___ % of the total connected load.
A. 80% C. 200%
jj/l2 5 % D. 100%
24. Equipment for hazardous locations shall be approved for the following property, EXCEPT
JC durability C. combustibility
B. ignitability D. explosibility
25. What is the minimum ampacity of supply feeder for a window type air conditioning unit
with 8 Amp rated load current with other load supplied?
A. 15 Amp C. 12 Amp
B^20Amp D. 10 Amp
26. Ferroresonance is destructive phenomenon associated with
A'" distribution transformer C. capacitor
B. distribution line D. transmission line
27. Its primary function is to provide ground source.
y t f zig-zag transformer C. sectionalizer
B. surge arrester D. tertiary transformer
28. A concrete encased electrode consisting at least_____mm zinc galvanized coated steel
reinforced or other electrically conductive or rods not less than_____mm diameter.
A. 5,000 mm, 13mm dia. C. 5,000 mm, 12 mm dia.
B /6 ,000 mm, 13mm dia. D. 6,000 mm, 12 mm dia.
29. A single electrode consisting of a _______which does not have a resistance to ground
of 25 Q or less shall be augmented by one additional electrode.
I. rod II. pipe III. plate
A. I only ^C . I, fl or
B. Ill only D. II only
30. Self contained device that automatically opens an electric circuit, on severe overload
without getting damage is:
A. fuse molded circuit breaker
B. disconnect switch D. cut-out
31. It is use to disconnect the system at same time protect the system
Ar'Safety switch C. Magnetic contactor
B. Magnetic starter D. Magnet wire
2. For circuits above 1 kVolt, what is the minimum conductor between surge arrester and the
line and to the grounding connection?
A. 3.5 sq. mm 14 sq. mm
B. 8.0 sq. mm u. 5.5 sq. mm
!3. Name the throw away protection device.
A. Manual starter CTFuse ^
B. Circuit breaker / D. Magnetic contactor
'4. Advantage of Magnetic starter over magnetic contactor.
^ Over current C. Over voltage or High voltage
B. Under voltage D. Undercurrent
5. How are multiple start push buttons arranged in a control circuit?
A. combination series-parallel C. gang
jMsarallel D. series
6. Which of the following is not accepted as if they have the same meaning?
low voltage C. over voltage
B. no voltage D. under voltage
37. What is called an electrically operated switch rated in amperes but also provide overload
Bcotection for motors ?
Qy Magnetic starter C. Safety switch
B. Magnetic contactor D. Manual starter
38. What is the most reliable and safety installation of switchgear?
A. Indoor C. Outdoor
B. OCB JD/Unit switchgear
39. Where else is the third wire in the service conductor terminated aside from S/N of the
panelboard?
A. Line 1 ground rod
B. Line 2 D. air terminal
40. Purpose of third wire in 3-wire, single phase 60 Hz, 240-Volt service for residential.
A. mechanical support grounding
B. 110-Volt D. harmonic drain
41. Communication cables and wires shall have a voltage rating not less than:
A. /400 Volts C. 600 Volts
B / 300 Volts D. 250 Volts
42. Code of Ethics is Resolution No.___ series of>995.
A. 18 (CJ22
B. 23 LI 19
43. Which of the following establishes ethical behavky: among professionals?
A. legal contracts ( jj ) Hippocratic oath
B. taxation regulations D. rules of play sports
44. Sec.7, Article III, R.A. 7920: An Electrical Engineer shall not a____ , vilify, or destroy the
' good name and reputation of others.
A. modify C. compete
B. divulge malign
45. An obligation may be extinguished or settled by the following EXCEPT:
A by the lost of the thingdue C. by condonation or remission of debt
B( by consumation or termination D. by novation or substitution

Practice makes perfect

SupplProbs Sept.2013
This Free Quality Manual is
Found Only at POWERLINE

E-MAIL:power linereviewcenter@yahoo.co m

POWERLINE REVIEW CENTER


M E Z Z A N IN E F L O O R , D O N A A M P A R O B U IL D IN G
C O R N E R E S P A N A & G. M . T O L E N T IN O S T R E E T S
S A M P A L O C , M A N IL A
TE L. N O S . 7 3 5 - 7 3 - 0 2 & 7 3 3 - 2 1 - 1 8

CEERS - POWEEINE REVIEW CENTER


TEL. NOS. (0 3 2 )2 6 1 -2 2 4 4 & (0 3 2 )2 6 1 -8 4 5 2
POWERLINE ENGINEERING REVIEW CENTER, INC.
AC CIRCUITS- 2 (THREE PHASE CIRCUITS)

I - BALANCED 3cp CIRCUITS :


The magnitude of the line currents are the same and they have 120 0 phase difference.

(1 ) WYE (STAR) Connected System :


One-line equivalent circu it:
T ¥ T7 ^ T~ 1 1 -------------

(0h€ phase of bolflncfcd


W g| 0 - Connected ^ • s je w
Relationships :
V% "'if-
u= h v

v>o a N it * * m
(2) D E LTA {MESH} Connected System :

\
sV
/>

Relationships

NOTE : To analyzed De/ra-connected system using one-line equivalent circuits, It is first transforme
Into an equivalent vvye-connected load making used o f :
VOLTAG E T R IA N G L E S :

Positive phase sequence: (ref. Pab) Negative phase sequence : (ref. FAB)
AB-BC-CA , A B -C , AN-BN-CN AB-CA-BC, A -C -B , AN-CN-BN
Note : If the phase sequence i$ not given, you can give the positive phase sequence.
Alternators are designed to operate with positive sequence voltage.

BALANCED 3m POWER Equations :


kW
PT = S V LI L cos9 = 3 cos9 = 3l}R+ where : cos 6 - —
S' kVA
kVAR
Qt = 43Vl I l sin& = 3VjIf si n9 = sin 8 ■ Qt Hi
zA kVA

S T = & VL1L = 3V^I^ = 31 =-JPt +Q t 0 - angle...between...V* & I a

POWER MEASUREMENT in a 3m System ;


L One-wattmeter Method - used to measure total “ balanced” 3<p power using one wattmeter only.
Methods used :
a.) T- Method
b.) Artificial Neutral Method
c ') Potential Leadshiff Method
d.) Current Transformer Method .
Note : for further discussion, pis. refer to Electrical Circuits by; Charles S. Siskind 2nd. Ed.(p.494~498)
2. Two-wattmeter M ethod - used to measure power drawn by a 3cp , 3 wire system.
( a.) For balanced 3cp system: Diagram of Connection as Shown : / Vet
n r —
4 -0

a.
3

L ~ _____1
WMR] = VL1L cosl tan 6 =

WMR2 = VLI L cos(30° + 9) PT = WMR, + WMR,


Where : Px - total real power of the balanced 3cp load
0 = p.f. angle of the balanced 3(p load.
A*
Note : If WMRj+ WMR2 is negative, you can carry the negative sign.
CO Consider the Following :
1. If p.f = 1.0 , both Wattmeters reads positive
W,=W2
2. If p.f. > 0.50 , both readings are positive
A PT - W, + W2
W) 3. If p.f. * 0.50 ,only one wattmeter gives reading, other will not read
se PT “ W,
A* 4. If p.f < 0.50, only one wattmeter will read positive
and other will give reversed reading (i.e. negative)

II - UNBALANCED 3<p CIRCUITS :


The line currents will not be equal nor will they have 120 0 phase difference as was the case of
6c
with balanced loads.

( b.) For unbalanced 3cp system: Consider the first Wattmeter’s current coil in line a ,and potential coil between
fines a and c. While the second Wattmeter’s current coil in line b and potentia
coil between lines b and c. The Wattmeter readings will be :

W M R ,= V J , c o s S , }"■ WMR2 = V J , c o s tf, ] J


3. Three wattmeter Method —used to measure power drawn by a 3(p,, 4 wire system.
In the case o f balanced and unbalanced loads : Diagram of Connection as Shown
« -

in

A
irr

WMR, = V J a c o s 0 , £ WMR2 =V hllIhc o s0 2 f WMR, = V J r c o s 9, f

BLONDEL’S THEOREM :
States that the minimum number of wattmeters required to measure power in a polyphase
System is “ONE - LESS” than the number of wires in the system.
4

ita i
S Y M M ETR IC A L C O M PO N EN TS : ( F O R T E SC U E ’S T H E O R E M )
rms
a. a
B 2 Any unbalanced 3<p system o f vectors can be resolved into three (3) balanced system o f vectors
N a m ely:
elta (L ) A balanced set o f 3<p vectors having the same phase sequence as the original unbalanced system of
mec vectors. This balanced system o f 3cp vectors is known as “Positive Sequence System ”...
ictar (2,) A balanced set o f 3cp vectors having phase sequence which opposite to that o f the original
line
unbalanced system o f vectors. This balanced system o f 3cp vectors is known as
A.
“Negative Sequence System”,
B.
(3.) A set o f three (3) lq> vectors which are equal in magnitude and which have exactly the same 3cp
120 position with respect to any given reference axis. This system o f lcp vectors is known as
nne “Zero Sequence or Uniphase System '\
cii
A. For Positive Sequence components : For Negative Sequence components :
B
Va V b i'
v„
bal<
sist I20 I-------- > Vb|= a2 V.,
20 Vfc
/i ' b2
E
Vc, =a V„ VC2 =a2Va2
thr
VH
For Zero Sequence components :
X)nn
onr
v a()= v a() Where : a = 1 / 120” = - 0.5 + j 0.866

v b0 = v a„ a2 - 1 / 240° - - 0.5 - j 0.866


Thr<
bate
seo Vc0 = Va0 Iden tity :
the
N O TE : Symmetrical components do not have separate existence, they are only mathematical
components which does not exist at a ll

At HARMONICS in different 3cp systems :


kV\
is I
In 3q> systems, harmonics may be produced in the same way as in l <p system. Hence.for all calculations
each harmonic is treated separately. Usually, EVEN harmonics are absent in 3<p system. But care must be exercised
when dealing with ODD harmonics, especially with 3rd. and all triple-n harmonics.
A
1C The following points are important in harmonic analysis for 3 <psystems :
ta (i) All triple-n harmonics i.e. 3rd.,9* ,15th. ...etc. are in phase.
(ii) The 7 .,13th.,&19th. Harmonics have positve phase rotation of A-B-C.
(iii) The 5th,,11th., &17th. Harmonics have negative phase rotation o fA-C-B
(iv) For Star-connected system ,the line voltages will be the difference between the
successive phase voltages and hcnce will contain No 3r<l. harmonic terms, bccausc
0. T being identical in each phase, thus it will cancel out.
tl
F (v) For Delta-connected system ,the resultant emf acting around the closed loop would be
the sum of the phase emfs. It is zero in the case of fundamental,5th.,7th., 1i J}\, 17,l\,ete.
but for 3rd.,9th.,15th.,etc,the resultant emf is 3 times the phase value acting in the loop.
(vi) For 3(p, 4 wire system, there will be No 3rd. harmonic component in the line voltage.
If the load is balanced, the resulting 3rd. harmonic line currcnt will all be in phase so
11. J
that the Neutral wire will have to carry 3 times the 3rd. harmonic line current. •
POWERLINe r e v ie w c e n t e r Inc.
The Ultimate E l Review Center
POLYPHASE CIRCUITS REVIEW QUESTIONS
1. In a balanced 2-phasev.oltage system, the phase voltages are 120 volts, what is
the line to line voltage?
A. 120 volts C. 240 volts
B^ 170 volts D. 60 volts

2. The three phase rural distribution system has a phase-to-neutral voltage is 7,967
volts, what is the iine to line voltage?
A. 13,200 volts C. 11,000 volts
B. 7,967 volts D. 13,800 volts

3. A balanced three phase wye connected source has using positive phase
sequence, Vca = 230 Vrms with 90 degrees angle. Determine Vbc.
A. 230 + j 0 V C. -115 - j199 V
B .-1 9 9 —j115V D. - j230 V

4. A balanced three phase wye connected source has using negative phase
sequence, Vab = 381 V rms with 60 degrees angle. Determine Vcn
A. 190.5 —j1 10 V C. —190.5 —j1 10 V
B. -110 + j190.5 V D. —]220 V

5. The wye connected three phase ABC voltage source has Vc= 240 Vrms with -24(
degrees angle. Find the line to line voltage Vab.
A. 208 + j120 C. 208 - j120
B. 360 + j 208 D. 360 - j208

6. Three equal resistors of 100 ohms are connected in delta across 240 Volts, 2
phase lines. What is the total power with the same resistors wye connected fror
the same supply?
A. 240W C. 1728W
B. 100W D. 57 6W
v

7. Three 100 ohm resistors are connected in wye across 480 Volts, 3-phase source
If one of the resistor is open, what is the power absorbed by the two resistors?
A) 1,152 W C. 768 W
B. 1,536 W D. 2,304 W

8. Three identical resistance connected in delta carry a line current of 300 Amp.
the same resistances are connected in wye across the same supply, what is th
line current ?
A . 150 A C. 300 A
B. 600 A D. 100 A
9. A circuit consisting of three resistors rated: 10 ohms, 15 ohms and 20 ohms are
connected in wye. What would be the resistances of the equivalent Delta
connected load ?
A. 33, 23 &15 ohm C. 70, 44.4 & 36.7 ohms
B. 30, 40 & 50 ohms D. 65, 43.3 & 32.5 ohms

10. Three capacitors, each having a capacitance of 75 uF are connected in star to a


400V, 3ph, 50Hz supply. Calculate the delta equivalent capacitance so that when
they are connected in the same supply, the line current will remain the same.
A. 150 |iF C. 225 uF
B. 25 jaF D. 75 |.tF

11. A balanced delta connected load of 4 + y'3 ohms per phase is connected to a 3-
phase 230 V supply. What is the line current?
A^SOA C. 23 A
B. 46 A D. 33 A

12. The metering of a 3-phase inductive load indicated the following values: 4000 kW,
34.5 kV, 80% power factor. What is the kVA? _
A. 3000 C. 5000
B. 4000 D. 7000

13. A balanced wye connected load of impedance 8+j6 ohms is connected to a three
phase balanced source of 400 volts 60 Hz. Find the power per phase.
A. 4267W C. 5333 W
B. 12800W D. 3198 W

14. The phase b voltage and the phase b current of a balance 3-phase system are: V
= 220 sin(oot+210°) and 1=10 sin (ot-1800). What is the power of the system?
A )5 7 16 watts C. 1905 watts
B. 3810 watts D. 3300 watts

15. A balanced delta connected load draws 10 Amp line current and 3 kW at 220V.
The reactance of each phase of the load in ohms is.......
A. 30 C. 38.1
B. 23.5 D. 20

16. A three phase source of 230-V supplies the following three phase loads.
20 kW at 90% power factor leading
5p kW at 80% power factor lagging
10 kW at unity power factor
What is the total kVA and current?
A. 85.7 & 225 A C. 84.7 & 212.6 A
B. 94.7& 182.8 A D. 84.7 & 92.88 A
17. Three identical impedances of 15 cis 60° ohms are connected in star to a 2
phase, 3 wire, 240 V system. The lines between the supply and the load hav
impedance of 2 + y'1 ohms. Find magnitude of the line voltage at the load.
A. 220 V C. 240 V
B. 185V D. 213V

18. A balanced delta connected load of impedance 13.5+j9 ohms is connected to a


three phase balanced source of 4160 volts 60 Hz through a line with 0.5 ohm
resistance and 2 ohm reactance . With Van=2400Z0, find the line B current usinc
ABC sequence.
A. 87.8+j327.8 A C. -327.8-j87.8 A
B. 69.3-j52 A D. 240-j240 A

19. A balanced 3-phase, 3 wire, 240 V, 60 Hz source supplies power to a star


connected load ZA = 10 + JO Q, ZB = 10 + y'10 f2 and Zc= 10 + ;0 Q. What is
the line current lB?
A. 11.53 Amp C. 15.93 Amp
B. 10.02 Amp D. 16.97 Amp

20. The power input to a 100 hp, 480 Volt, 85% efficiency and 75% power factor 3-
phase motor at full load is measured by two wattmeter connected at lines A & C.
What are the readings in kW?
A. 66.23 & 21.55 C. 87.76 & 11.7
B. 69.5 & 18.26 D. 67.89 & 30.4

21 .A three phase induction motor takes 3000 W at 80 percent power factor. Two
wattmeters are used to measure the power, Wa and Wc . Wa reading is _____ .
A. 2000 /C:)2150
B. 3000 D. 2500

22. In a balanced three phase, 208 volts circuit, the line current is 100 amperes. The
power is measured by two wattmeter method, one reads 18 kW and the other is 9
kW, What is the power factor of the load?
A. 0.8 C. 0.866
B. 0.5 D. 0.95

23. An unbalanced wye connected loads on a 3 phase 4-wire system consist of Zi =


10 + j5, Z2 = 8 + j4 and Z3 = 20 + jO. The wye connected generator load has
balanced voltage to neutral of 120 volts. What is the wattmeter reading at Z 1 ?
A. 720 watts C. 1440 watts
B. 3312 watts Q. 1152 watts

24. It is required to increase the power factor of a 750 kW three phase balance load
from 70% lagging to 90 %lagging. The line voltage is 6,900 volts, 60 Hz. Specify
the capacitor required to increase the power factor of this wye connected load in
micro-farad per phase.
A. 20.45 C. 22.39
B. 18.58 D. 17.22
25.A 3-phase 150 hp, 480 Volts synchronous motor with rated efficiency of 90% is
operating at rated load. It is in parallel with 3-phase balanced load of 400 kW at
0.75 p.f. lagging. In order to improve to p.f. to 0.95 lagging, the excitation of the
motor was increased. Calculate the operating power factor of the motor.
A. 0.65 C J 0.57
B. 0.38 D. 0.82

26. A 7.5 hp wound-rotor, 3-phase induction motor draws a line current of 18.4 Amp
at 230 V line when operating at rated output and efficiency of 88%. Applying
artificial neutral-Method of measuring 3-phase power using only one wattmeter,
What is the wattmeter reading ?
A. 3179 W C) 2119 W
B. 2126 D. 1590 W

27. What is 1+A+A2 in rectangular form?


A. 0.5 +j0.866 C.O+j 1.732
B.-0.5-j0.866 D. 0
28. The three phase unbalance currents are :
la = 10 cis (-30°) Amp, lb = 0, Ic = 10 cis 150° Amp
Find the negative sequence current of phase b.
A. 8.66 cis 30° C. -5.77
B. 5.77 cis (-60°) D. 5.77cis 60°

29. Given the following phase currents: la=5 cis 45°p.u. Ib=3 cis(-90°)p.u. and
Ic=3cis150° p.u. Determine the positive sequence symmetrical component of
current for phase A.
A. 3.635 cis 276.8° p.u. C. 3.635 cis 156.8° p.u.
B. 3.635 cis 36.8° p.u. D. 0.748 cis 65.3° p.u.

30. The line currents of unbalance delta connected load are:


la=50 cis 0 degree, lb=40 cis 240 degrees, lc=45.8 cis 130.9 degrees.
Determine the phase B zero sequence component.
A. zero C.16+j7.25
B. 45-j.2.9 D. 5-j2.9
Supplementary Problems

1. In a balanced12-phase; voltage;system, the phase voltages are 100 volts, what is the line to line
voltage?
A. 120 volts C. 141 volts
B. 170volts D. 60 volts

2. In a balanced 3-phase voltagje generator, the different phase voltages are____ degrees apart.
A. 120 C. 60
B. 240 D. 30

3. In a 3-phase 4-wire system, the line voltage is 4160 volts. What is the line to neutral voltage?
A. 1390 volts C. 2400 volts
B 2080 volts D. 4160 volts

4. In ABC sequence, What is the equivalent of Van ?

A. VbcZ9Q° C .~ V b cZ -9 0 °
a/ 3 V3

B. ~ V b c Z \2 0 ° ( D. j= VbcZ - 120°
V3 . ■ V _ A /3

5 In a balance three phase voltage, Van=120/30°, What is Vcn using ABC sequence?
A.-103.9+j60 C.-60-j109.3
B. —60— j103.9 D. 0—j120

6. The secondary of a 3-phase transformer bank is connected delta, the line to line voltage is 480V,
the midpoint of the winding between terminal A & B is grounded. What is the voltage across
terminal C & the ground?
A. 240 volts C. 720 volts
B. 277 volts D. 416 volts

7. A three phase, balanced delta connected resistive load draws 30 ampere phase current from
balance three phase supply. An open circuit fauit occurs in one of the lines. Determine the line
current.
A. 45A C. 90A
B. 60A D. 100 A

8. • Three 30 ohms resistance are connected in delta across 220 V, 3-phase supply. What is its
equivalent wye connected resistance?
A. 90 Q C. 45 Q
B. 30 Q D. 10 Q

9. Three identical capacitance, each of 150 ^F are connected in delta. The value of capacitance in
each phase of the equivalent star connected load would b e .......
A. 50nF C. 150fiF
B. 450fiF D. 100nF

10. A balance 3-phase load takes 20 amperes at 230 volts line to line. The phase A line current lags
the line voltage Vba by 50 degrees. What is the power factor of the load in percent?
A. 86.6 C. 94
B. 64.3 D. 76.6

11. Three identical resistors are connected in delta across balance, 3-phase source. If one of the
resistor is open,
A. Powerwill becomedouble C. Power will become three times
B. Power is reduce by 33% D Power will increase by 67%
12. The secondary of a transformer bank is wye-grounded, it has a balance voltage and the balanced
load. The voltage to ground is 132V and its line current is 40A. What is kVA of the load?
A. 9.1 C. 27.4
B. 19.2 D. 15.84

13. A 170 kV, 3-phase electric source delivers 200 MVA to a balanced load which has a power factor
of 90 % lagging. What is the line current ?
A. 257 A C. 402 A
B. 502 A ^ D. 679 A

14. The phase b line voltage and the phase a line current of a balanced 3-phase system are : V =
220 Z 210° Volts and I = 10 Z 30° Amp respectively. What is the power of the system ?
A. 1905 Watts C. 5716 Watts
B. 3300 Watts D. 3810 Watts

15. The phase b voltage and the phase a current of a balanced three-phase system are : V = 220 sin
(a)t + 210°) Volts and I = 10 sin (co t + 60°) Amp respectively. What is the power of the system ?
A. p C. 1905
B-/3300 D. 6600

16. The phase A line voltage and the phase C line current of a balanced three-phase system are : V =
220 sin (a t - 30°) Volts and I = 10 sin (co t + 90°) Amp respectively. What is the impedance of the
system?
A. 19.1 —J11 C. 19.1 +j11
B. 33 - j19.1^r D. .33+j19.1

17. A balanced delta connected load of 4 - j3 ohms/phase is fed by a 380 V, 3-phase system. Find
the reactive power.
A. 52 kVARS C. 69 kVARS
B 29 kVARS D. 87 kVARS /

18. A balanced three-phase wye-connected load of 150 kW takes a leading current of 100 A, when
the line is 2400 V, 60 Hz. What is the capacitance per phase?
A. 21 mF C. 205 mF
B. 21 pF D. 205 pF

19. A balanced delta connected load consisting of pure resistance of 18 ohms per phase in parallel
with a purely resistive balanced wye connected load of 12 ohm per phase. The combination is
connected to a three phase balanced supply Of 346.42 volts line to line via a three phase line of
inductive reactance j3 ohm per phase. Determine the line to line terminal voltage of the combined
load.
A. 332.6 V ' C. 277.1 V
B. 320.8 V D. 294.3 V

20. A symmetrica! three phase 240 volts supplies a balanced delta connected load of 8-j6 ohms per
phase. Using ABC sequence with Vab as the reference voltage, What is the line A current ?
A.-25+J32A C. 16.3-p8.2A
B. 24.9+j33/2A D.' 41.27+j4.97A

21. A three phase line has an impedance of 2+j4 ohms. The line feeds two balanced three phase
loads that are connected in parallel. The first load is wye-connected and has an impedance of
30+j40 and the second is delta connected load and has an impedance of 60-j45 ohms. The line is
energized at the sending end from a balanced supply of line voltage 207.85 volts. Taking phase
Va as reference determine the line current la.
A. 5 - j 8.66 A D. 5+j 8.66
B. 5 A D. 0 - j5
22 A wye connected load has a per phase impedance of 10 +j 1 ohms. It is supplied by a wye
connected balance source . The voltage Van=120/0! volts. What is the line A current?
A. 11.88+j1.19 A C. 11.88-j1.19 A
B. 13.86-j13.86 A B. -18.93+j5.07 A

23. A delta connected load has a per phase impedance of 29.8757 -j 25.0687 It is supplied with a
wye connected balance source through a 3-phase line with 4 ohm resistance. The voltage
7« sequence is given by Va=277 cis-30deg Vb=277 cis -150deg Vc=277 cis 90deg. What is the line
A current?
A 17-jO.267 .C. 6 5+j0.75
B 17+j0.267 D. 6.5-jO.75

24. The load on a 230 volt three phase generator consist of (a) three groups of incandescent lamps,
each group containing twenty 100 watt lamps and (b) a 9 hp motor operating at 82 percent P.F. at
87 percent efficiency . Find the total load current in amperes?
A. 37 C. 45
B. 25 D. 30

25 A 3-phase 480 volts supply delivers power to a wye-connected load with impedance per phase of
10+j5 and a wye connected pure resistive load of 15+jO. What is the average power of one phase?
A. 11.2 kW ' /( , C. 12 kW
B. 3.3 kW S • D. 33.8 kW
*0 r i ' O
26. Three identical impedance are connected' in delta to a 3-phase supply of 400 Volts. The line
current is 34.65 Amp and the total power taken from the supply is 14.4 kW. What is the impedance
in each phase ?
A. 1 2 + / 1 6 Q C. 16+ /1 2Q
B 20+/0Q D. 0+/20Q

27. A 25 kVA delta connected balance load is supplied from 230 volts, 3-phase source. What is the
impedance per phase?
A. 3.5 ohms C. 10.6 ohms
B. 2.3 ohms D. 6.3 ohms

28. In an unbalanced wye system, the phase voltages are:


Vna = 132 cis 0°
Vnb = 130 cis235°
Vnc = 133 cis 117°
What is the line to line voltage Vcb?
A. 227 cis (-15.2°) C.225.44 cis (-93.6°)
B. 218 cis 37.2° D. 232.4 cis 27.3°

29. A generator supplies 3-phase power to a balanced load. The voltage is 230 Volts, the current is 18
Amperes and the power factor is 85%. What is the power?
A. 3.6 kW C. 6.1 kW
B. 1.6 kW D. 1.4 kW

30. A three phase motor takes 10 kVA at 0.6 power factor from a source of 230 V. It is in parallel with
a balanced delta connected load having 16 ohms resistance and 12 ohms capacitive reactance in
series in each phase. Find the total volt-amperes
A. 12.8 C. 7.9
B. 17.9 D. 15.3

31. Find the total power in a delta connected load with 6+j8 ohms per phase and supplied from a
220V, 3-phase source.
A. 8712 W C. 8367 W
B. 8376 W D.X8617W
32. A delta connected load of 9+j12 ohms per phase is supplied by a 3-phase balance source of 230
volt rms through a line with an impedance of 0.06+j 0.12 Ohm. Find phase voltage at the load.
A. 288 volts C. 132 volts
B. 224 volts D. 228 volts

33. A delta connected load has a per phase impedance of 10 +j 10 ohms. It is supplied by a wye
connected balance source through a 3-phase line with 1.0 ohm resistance and 1.0 ohms inductive
reactance. The voltage sequence is given by Va=120/-30° Vb=120/-150° V c -120/90°. What is
the line A current?
A. 13.86+j13.86 A C. 5.07-j18.93 A
B. 13.86-j13.86 A D. -18.93+j5.07 A

34. A 120 volts per phase, 3-phase wye connected source delivers power to the following delta
connected load. Phase 1 :40+j0 ohms, Phase 2: 20 cis (-60°)ohms, phase 3: 15 cis45° ohms. Find
the current at line 1. (Use1-3-2 sequence)
A. 18.5+j 3.57 C.-15.2+jO
B .-2.9-j3.57 D. -2.6+j 0.8

35. A balanced 3-phase, 3 wire, 240 V, 60 Hz, ABC source supplies power to a star connected
resistance 10 Q, 15 £2 and 20 Q. What is the current through 20 ohms resistance ?
A. 11.23 Amp C. 15.93 Amp
B. 9.77 Amp /'D ^ 8.05 Amp

36. A three phase ABC source with Van= 240cis 30degrees volts supplies unbalanced wye-
connected loads: 15 ohms resistance connected across lines AN , a 12 ohms resistance
connected across lines BN and 20 ohms resistance across CN. Determine the current at line B.
A. 17.32-j10 A C. 0-j20A
B.-10.39+j6A D. 13.85+j8 A

37. Three unequal single-phase induction motor are connected between lines and the neutral of
balanced 3-phase , 202-V line to neutral. The first load takes 20 kW at 0.82 power factor, the
second takes 28 kW at 0.75 power factor and the third takes 36 kW at 0.8 power factor. What is
the current in the neutral conductor?
A. 105.5 A C. 85.6 A
B. 125.4 A D. 133.4 A

38. A balanced 3-phase, 3 wire system supplies two balance wye-connected loads. The first draws 6
•kW at 0.8 pf lagging while the other requires 12 kW at 0.833 pf leading. If the current in each line
is 8 A. Find the supply voltage in kV.
A. 1.544 C. 2.291
B. 1.682 D. 1.323

39. A balanced 3-phase source supplies three loads. Load 1 is 15 kVAat 75% pf lagging, Load 2 is
10 kW at 90% pf lagging andLoad 3 is unknown.If the generator delivers 35.52kVA at 80% p.f.
lagging, what is the unknown load?
A. 8.37kVA at 76.4% pf lagging C. 8.37kVA at 76.4% pf lagging
B. 9.3kVA at 89.4% pf lagging p>9.7kVA at 73.8% pf lagging

40. Three star-connected impedances Zi = 20 + /37.7 ohms per phase are connected in parallel with
three delta-connected impedances Z2 = 30 - y'159.3 ohms per phase. The line voltage is 398 V.
Find the line current.
A. 3.36 A C. 5.8 A
B. 7.26 A D. 4.24 A

41. A three phase induction motor takes 3000 watts at 80% power factor. Two wattmeter Wa & Wc
are used to measure the power. What is the reading of Wc?
A. 800 watts C. 950 watts
B. 850 watts D. 900 watts
42. The power input to 440 Volt 3-phase motor is measured by two wattmeter method, the readings
are 460 W and 780 W. What is the input current?
A. 1.78 A C 1.63 A
B. 0.94 A D. 2.82 A

43. A balance three phase resistive load is supplied by 230 volts 60 Hz and draws a current of 90A. If
two wattmeter is used, what will be the reading of each wattmeter.
A. 20. 7 kW C. 25.7 kW
B. 35.9 kW D. 17.93 kW

44. The ratio of the wattmeter readings in the 3-phase inductive system is 5:3 when power is
measured by these two wattmeter. What is the power factor of the load ?
AJ 0.92 C. 0.80
B. 0.50 D. 0.866

45. A three phase balanced load id connected across 220 volts three phase source. A wattmeter
reads 600 watts when its current coil is connected in line a and potential coil across line a and b.
Next, the voltage coii is connected between lines b and c, but the current coil remains at line a.
The wattmeter agam reads 600 watts. Calculate the load power factor.
A. 0.8 C. 0.806
B. 0.866 D. 0.5

46. A 4,160 V, 3-phase, 3 wire- ABC source supplies power to an industrial load that draws the
following line currents : la = 110 cis (-36.87°) Amp, lb = 166.5 cis (-168.2°) Amp. Using two
wattmeter method, what is the total power drawn by the load?
A. 882.5 kW C. 1,281 kW
1|) 828.5 kW D. 97.5 kW

47 One wattmeter method of measuring power of a balanced system, The indication of wattmeter by
T-method is _____ .
A. Pt/3 LC> Pt/2
B. Pt/CTR D. Vlcos(30+#)

48. An unbalanced wye connected loads on a 3 phase four wire system consist of Z1 = 10 + j5, Z2 =
8 + j4 and Z3 = 20 + jO. The wye connected generator supplying the unbalanced load is balancec
voltage to neutral of 120 volts. What is the wattmeter^ reading at Z2 , watts?
' A. 720 o <^>40
B. 3312 D. 1152

49. The 3-phase, 4-wire source voltage of Van =230 sin(cot+30deg.) volts supply loads .with the
following line currents: la = 4.7 sin(cot +30 deg.),lb = 6.8 sin(ot-60deg) and Ic = 3.4
sin(o)t+120deg.)
Determine the total power.
A.3113 watts C. 2786 watts
B. 2768 watts D. 2733 watts

50. A three phase system delivers 50 MVA of a balanced delta load of power factor 78%. Determin
the size of capacitor bank necessary to raise the power factor to 90%.
A. 17.3 MVAR Q, 11.4 MVAR
B. 13.5 MVAR D> 12.4 MVAR
Vy
51. A 132 kV line, three phase system delivers 70.7 MVA of a balanced delta load of power factc
70.7%. Determinethe reactance necessary to attain unity power factor.
A. 1092 ohms Cs 1142 ohms
B. 965 ohms D. ')1045 ohms
52. A balanced Y-connected load of 30+j10 ohms per phase is connected to a 3-phase 208 volts 60
Hz supply. Determine the value of each capacitor in // F which will yield to unity power factor.
A. 88.4 C. 26.5
B. 8.84 D. 2.65

53. What will be the rating of a phase advancing plant in kVA if it improves the power factor from 0.8
lagging to 0.95 lagging? The consumer load is 500 kW and the current taken by phase advancer
lead the supply voltage at power factor of 25%
A. 150 kVA C. 175 kVA
B. 200 kVA D. 250 kVA

54. Given the following sets of per unit symmetrical component of phase A currents, determine the
phase C current.
Iao=1.0 cis 75 deg, Ia1= 1.0 cis 15 deg., Ia2= 1.0 cis(-45deg)
A. 0.259 + j0.966 p.u. C. 1.932+j0.518 p.u.
B. 0.516 +j1.931 p.u. D. -1.414 + J1.414 p.u.

55. The sequence component of phase 2 currents are:


lo= 0 lpos=11.55 I neg.=11.55 cis (-60deg)
Determine the phase A current.
A. 22.3 cis 15 deg. C 22 cis 120 deg.
B. 20 cis 150 deg. (CO 20 cis -30 deg

56. The sequence components of three phase current are:


I zero = 0 I positive = 90 + j5.78 amp. I negative = 10-j 5.78 amp
Determine phase C current?
A. 100 cis 120° C. 73.22 cis 118°
B. 82 cis 122° D) 91.65 cis 131°

57. A delta connected transformer has the following line current:


la=50 cis 0 degree, !b=40 cis 240 degrees, lc=45.8 cis 130.9 degrees.
Determine the phase C negative sequence component
(2>5-j2.9 • C.0+j5.78
B. 8-j4.22 D. 5-j2.9

58. In a three phase balanced delta connected load with one line open, line A has 20 A. Assume lin<
A is the reference and the line c is open. What should be the positive sequence of line a?
A .O . ( c ) 10-j5.77 A
B. 30+j17,32A D. 10+j5.77 A

59. The line currents of unbalance deltaconnected load are:


la=50 cis 0 degree, Ib=40 cis 240 degrees, lc=45.8 cis 130.9 degrees.
Determine the phase B zero sequence component.
A. zero C.16+j7.25
B. 45-j.2.9 D. 5-j2.9

60. A three phase unbalanced current are:


la = 10 cis (-30°), lb = 0, Ic=10cis150°
What is the phase b positive sequence current?
A. 8.66 C. 5.77
O
B. 5.77-cis240° D. 8.66 cis 120'
:1 '
J

PRACTICE Makes Perfect HI


This Free Quality Manual is
Found Only at POWERLINE

E-MAI L: Powerl inereviewcenter@yahoo.com

POWERLINE REVIEW CENTER


MEZZANINE FLOOR, DONA A M P A R O BUILDING
CO R N ER ESPA N A 8 t G. M. TOLENTINO STREETS
SAM PALO C, M ANILA
TEL. NOS. 735-7 3-02 & 733-21-18

CEERS - POWERLINE REVIEW CENTER


TEL. NOS. (032)261-2244 & (032)261-8452
POWERLINE REVIEW CENTER, INC,

POWER PLANT REVIEW LECTURE


Pja^L^xxLEugm ^m
An of designing and installing generating plant that will result in maximum return on investment
(profit) over the expected life of the equipment. And also operating this equipment to achieve reliable,
continues & cheap power service.

Power Plant - A station or establishment which houses the pnme-movers, electric generators and
auxiliaries, for conversion of mechanical, chemical and / or nuclear energy into electrical
energy.
/ Types o f Power Plants ( As to source o f energy )

. 1. Therm al P ow er P lant - an electric generating station using heat as a source of energy

a} makes use of heavy fuel oil light fuel oil or bunker oil for
production of energy.

b) Coal-fired thermal plant - makes use of pulverized coal as fuel

f - makes use of wood (ipil-ipil)

d) Nudesr-steam plant - makes use of steam generated in a reactor by heat from the fission
process of nuclear fuel ( uranium. 235, Uranium-238)

e) Gas turbine plant - makes use of combustible gasses as fuel from a gas turbine engine
priniemover,

f) Geothermal Plant - makes use of generated heat from the inherent steam from the earth’s
magma fuel

9) Solar-steam Plant - makes use of steam generated from solar radiation.

ric Power Plant - an electric generating station using flowing water to drive the
prime-movers (hydraulic turbines) either impulse or reaction type.

a) Impulse type - use for high-heads and low volume, example is the 4*Pelton wheel”

b) Reaction type - use for low-heads and high volume, examples are “ Francis >& Kaplan ‘
Turbine

IXQSkl.
a} R un-of-river using pondage orsteam jlow as it occurs, more power can be generated in a
rainy season than in dry season,

b) Plant with storage e a o jc ttv - associated with a large water reservoir. This permits
regulated supply of water so that the power output is
constant throughout the year.

C) P um p-sioraae P la n t- where energy is generated during periods of high system demand


f using water which has been pumped into a reservoir usually during
periods of relatively low system demands.
3. DiSSfiLEfiWSdQaBt - 3 plant of internal combustion engine (ICE) prune-roover using diesel as fuel
in producing energy .

4. Windmill Plant - using a series of windinills as prirae-mover.

5. ^g^L-VVaves/Ocggn,JM&5JF?a n tm a k e s use of the natural rising of tide to simulate flowing water.

6. S.Qiar...Q.r Pi?.Qto- .Vtojlaj cJP[ant - chemical conversion of radiant energy of the sun to electric (dc)
energy.

U - Types o f Power Pi ant ( /is ic use )

1. B_5se-Loacf Plant - plant that assumed load requirements under normal conditions.

2- B raking P l? nt - plant that is normally operated to provide power only during peak load periods.

3- EggMfeting Plant - plant capable of carrying load for the time interval cither during off-peak or peak
periods and usually responds to changes in system frequency.

4. Reserve or Stand" by plant - for peak or system deficiencies.

Types o f Plant Reserves

a) C o M j^ e r v g - portion of the installed reserve kept in operable condition and available for
service but not for immediate loading

O perating ~ refers to capacity in actual service in excess of peak load

c) ijQJL££$gF£L~ refers to units available, maintained at operating temperature and ready for
service although not in actual operation.

3} SpilinjnQ j:esejyti^- generating capacity connected ro the bus ready to take load.

STEAM POWER IM.aNT —operates in the principle of “Rankine Cycle”.

cold
w ater
from
riv e r

LEGEM D circulating
pump
- w ater
• s te a m
Regenerat - The process of extracting or bleeding-off steam from the turbine to open -heater
and back to the boiler for steam production.

Thermal E fficiency ( e t )

Heat equivalent o f mech'L energy transmitted to the turbine


Q ZZ------ i----- ------------- --------------------------—--- —
-- —------------------
Heat o f fu el combustion

Where: Wnet = Wt - IW p
Qa Qa Qa
IW p -sum of all pump work

O verall E fficiency (eQ)

Heat equivalent o f electrical output


g ——-— — ...... - ' ' ■■------■------------“
Heat o f fuel combustion
Where : eg = electrical efficiency = alternator efficiency

H E A T R A T E (HR)

HR * — (K ) ; Where : K = constant conversion factor

Values of K based from the ff. Conversion factor

1 KWh - 3600 KJ 1 KWh = 3413 BTU


1 H P - h r -2 5 4 5 BTU i KWh - 860 Kcal.

HYDRO-ELECTRIC PO W ER PLANT ( SCHEMATIC ARRANGEMENT )


Surge tank ( open a t lo p )
Gross head efev. r use to elim inate w ater hammer

Valve house
Reservoir

E ffe c tiv e head;


(excluding head loss)

: Power house

Tail racm elevt


£ center line o f turbine runner
4

IMPORTANT TERMS & PLANT FACTORS :

1. Connected Load ( CL) ~ it is the sum of all continuous rating of all utilization equipment in
the consumer premises connected to the supply system.

2. Maximum Demand ( or Peak Load) (MD) - it is the greatest demand of load on the power station
during a given period ( say 1 5 - 3 0 minutes)

3. Average Load (AL) - the average ofloads occurring on the power station in a given period ( say day,
month or year), sometimes called “ average demand”.

Average Load (AL) = ------- T o tM n e ^ r o t o c e d J ^ _____


Period hrs in a day, month or year (Pr)

4. Demand Factor (DF) - it is the ratio of maximum demand on the power station to its connected Load.

df = — - < i.O
CL
5. Diversity Factor (Div F} - the ratio of the sum of individual maximum demand to the simultaneous
maximum demand on the power station .

DJv.F = — — S tM P ll ______ 2 , 0
simid tan eous MD

6. Coincidence Factor fCO.F) - reciprocal of diversity factor .


simul tan eous MD 1
Co.F = ---- --------------------= ----------- < 1.0
Z MD's Div.F

7. Load Factor (LF) - it is die ratio of average load to the maximum demand during a given period.
rp AL TER
I f - ----- -------------- Where:
MD MD x S r
Sr = service hours or operating hours

8. Capacity Factor ( CF) - it is the ratio of actual energy produced to the maximum possible energy that
could have been produced a during a given period.
TEP
CF ~ ----------- Where:
IC x Pr
IC ~ installed capacity or rated capacity of
the machine as an individual.

9. Name plate or Capacity rating - ability to carry load under condition specify by the manufacturer.

10. Capability Mating - maximum output of equipment obtained by test under specific operating condition
( not specified by manufacturer).

Plant Use ( or Output) Factor- it is the ratio of the energy generated to the product of plant capacity
and the number of hours for which the plant was in operation .
TEP
P U F - ------ -----
IC x Sr
i 2. Utilization Factor (VF) it is the ratio of maximum generator demand to the generator capacity
MD
UF = -----
IC
! 3. Operating Plant Factor - (OPF) it is the ratio of average load for a given period to the operating
capacity in actual service only ( for multi-set p lan t)
TFP
OPF ------ : r £ L _ Where:
OPC x Sr
OPC —operating plant capacity

14. Operating Load Factor (OLF) ~ is the ratio of the average load for a given period of the time to the
average of daily maximum demand during the same period.
( say for 30 day period),
TEP! 30(24)
OLF
M L +... 4-
+MO^
M DX+
30

15. Operating ( or Service Factor ) —it is the ratio between Capacity Factor (CF) to the use factor (PUP),
or it is the ratio of service hours (Sr) to the period hours (Pr).
CF Sr
GF = ------ -- —
PUF Pr

16. Plant Ratio (PR) - it is the per annum measure of the plant’s capacity factor, it is the ratio of Net
kWh output for the year to the name-plate capacity of the plant times hours of th>
years.
TEP per year
PR = ------- ----------—
-------
IC x Pr (or 8760)

ECO NO M IC S ON POW ER G ENERATION

The art of determining the per unit (or per kWh) cost of production of electrical energy.

a) Fair b) Simple c) Reasonable

The following Elements enters into the cost of Electric Energy to consumers :

1. Fixed Elements (FE) — to start the enterprise, includes the following :


a) power- plant, lands, building structures etc.
b) primary distribution lines and substation including supports.
c) Management, administrative staff
d) Depreciation
e) General maintenance, required whether the plant is in operation or not.
Note: FE is dependent on the size of enterprise.

2. Energy Elements (EE) - Sometimes called running cost, it is operating expenses to generate power/
energy which includes the following.
a) Fuels
b) Labor for operation
c) Facilities, water, oil supplies
d) Maintenance required when the plant is in operation only.
Note: EE is dependent on the voiume/Level of generated kW or kWh.
3, Customer Elements (CE) - cost to bring power to the users/ consumers which includes the following:

a). Sesmdm <Mnkutm. system including ssiyics deeps & m im ,


b) Personnels for lines, meter readers, collection of bills.
c) Advertisement
d) Franchise (50 yr period)
Note: CE is dependent on the number of customers

Investors Profit / Return-on~Investments (IP) ~ Controlled by the government.

RATE ( o r T a riff) —rate at which electric energy is supplied to a consumer,

TYPES OF TARIFF:

Uniform rate tariff - there is a fixed rate per kWh consumed.

Flat rate ta riff - whenjlifferent types of consumers are charged at different uniform rates.

3. Block rate tariff - also called room rates” when a block of energy is charged at a specified rate and
the succeeding blocks of energy are charged at progressively reduced rates.

- also called “two-charge rate” charge on the basis o f maximum demand of the
consumer and the energy consumed.
Total charges = ^ b x kW (of MD) + # c x kWh (consumed)
Where : P b = charge per kW of MD
f* c = charge per kWH of energy consumed.

it is similar to two-part tariff with only difference is that the max.


demand is actually measured installing a maximum demand meter in the
premises of the consumer. It is mostly applied to large users.

Few er factor tariff - power factor of consumer’s load is taken into consideration
Billing kWh - [ Meter Reading ] x K
Where : K = 1.0 if p.f is from 80,1% to 85.0%
K > 1.0 if p.f, is lower than 80.1 % (k is from 1.01 to 1.09)
K < 1.0 if p.f. is higher than 85.0 % (k is from 0.95 to 0.99)

Econom ical lim it o f pow er fa cto r

Bi
~ most economical p ,f.
100 A
Where:
A = charge per annum per kVA maximum demand
i - interest and depreciation charges
B = cost of synchronous condenser per kVAR

7 Three -P art T ariff - also called “ three-charge rate” or Doherty Rate “ total charge is split into
three.
Total charge = f a + ? b x kW (of MD) + f ' c x kWh ( consumed)

Where: f*a - fixed charge made during each billing period, includes interest, depreciation on the
cost o f secondary distribution and labor cost.

KELVIN'S ECONOMIC LAW :


The most economical cross-section o a conductor is that which makes the interest on the
capital outlay plus depreciation due to the conductor in the cable equal to the annual cost of energy loss.
P O W E R L i n e r e v i e w c e n t e r Inc.
The Ultimate E.E. Review Center
POWER PLANT REVIEW PROBLEMS

1. Which of the following is the least efficient power plant?


A. Diesel :\f - £C) Coal fired I ~ 2H ~3S/a
B. Gasoline D. Combined cycle

2. Which of the following power plant will take least time in starting from cold condition to
full load condition?
A. Nuclear power ( c ) Gas turbine
B. Steam turbine D. Hydro-electric

3. Geothermal power plant is most suitable for .


A. Stand-by plant C. Regulating plant
B. Peak load plant fxTj Base load plant

4. Which power plant can have a single unit up to 100 MW?


A. Hydro electric power plant C. Steam power plant
B. Diesel power plant D. Nuclear power plant

5. An effective remover of dusts, carbon particles and others from the flue gas of power
plant?
,7V; Electrostatic precipitator C. Mechanical collector
~BC Soot blower & collector D. Dust scrubber

6. What is the reason why thermal power plant decrease capacity during summer?
A. decrease water level at the lake C. taking a bath or swimming on lake
i temperature rise of the lake D. there is no rain

7. The efficiency of the thermal power plant improves with____ .


A. increased quality of coal burnt C. lower load in the plant
use of high steam pressure D. larger quantity of water used

8. In fusion, energy is produced when


A. fuel burns C. nuclear particle divide
B. chemical compounds fused together ^ nuclear combines particle

9. In fission, energy is produced when


A. chemical compound splits into its constituents C. nuclear particle combine
B J nuclear particles split D. molecules combine

10. The net energy required of a certain country in yr 2000 was approximately 3 x106 GWh.
What is the equivalent value in quad? 7 i is ~ i
A; 12.42 C. 124.2 = 10^ 1
10,24 ^ D' 102,4 ;(\*t\0Vr • 2>Ht3 etfy:
\ \\iQhr • V.A'
11. In a power station, 4 x10 GWh of energy is to be produced in TyrrhalHrom coal and
half from natural gas. The energy content of coal is 900 W-yr per ton, and that of natural
gas is 0.03 W-yr per ft3. How much natural gas will be required?
A. 152.2 x109 ft3 C. 125.2 x10s ft3
12. Suppose that the consumption of energy in a certain country has a growth rate of 4%
per year, in how many years will the energy consumption be tripled?
A. 24.47 yrs „ q 27.47 yrs
B. 17.33 yrs 0 r'' '- ' ' 5 34.66 yrs

13. Natural gas reserve in a certain country is estimated at 100 x109 ft3, with an energy
content of 0.025 W-yr per ft3, if the present peak power demand ia 0.5 GW and the
power demand growth rate is 5% and all the energy is to be supplied by natural gas,
approximately how long will the reserve last?
A. 4.64 yrs t ^ ^ f + \ C. 4.46 yrs
B. 6.44 yrs I— Po u. 6.64 yrs

14. A 100 MW power plant has a heat rate of 2.88x106 Calories per kWh. it is a base load
plant and runs at full load 24 hrs a day. How many tons of coal is needed per day?
A. 250 ' C. 830
B. 625 ( D ) 960

15. One of the following methods of producing emf NOT yet in commercial use.
( A. Fuel Cell C. Magnetic
B. Thermal D. Solar

16. Which of the following energy generators is NOT an atmosphere pollutant?


A. Light crude power plant * 'C. Solar power plant
B. Gas-fired plant w/ electrostatic dust collector d ? Biogas plant

17. Out of the following, which ONE is not unconventional source of energy?
A tidal power C. nuclear power
( m geothermal power D. wind energy

18. Which of the following generating station has the minimum running cost?
A. Nuclear power - C. Diesel plant
B. Thermal plant D.) Hydro-electric plant

19. How does the output of hydraulic turbine vary with the diameter of the blade?
A n It varies directly as the square of the diameter of the blade.
B.‘ It varies directly as the two-third power of the diameter of the blade.
C. It varies directly as the diameter of the blade.
D. It varies directly as the cube of thediameter of the blade.

20. A hydroelectric plant generates 100 MW at an available head of 200 m, and at an


overall efficiency of 75%, what quantity of water in cubic meter per second is required?
C. 1220
a toh * % *-TIq D. 26

21. A wind generator with an efficiency of 80% has a blade diameter of 20 m. If the wind
velocity is 35 kph, how much power is obtainable from the generator?
A. 4.32 kW j ! C. 5.47 kW
t(3 54 7 kW — ____ ^ D- 43 2 kW
22. The maximum tidal head available for a proposed tidal-power station is 7m. What must
be the area of the tidal bay to generate an average of 1,200 MWatts of power?
A) 112 sq km. f Pduvj - n.s-in A W 3- c - 126 sq km.
B. 102 sq. km — —------— —— 1— --- D. 121 sq. km.
23. A power customer is applying for source listed the loads as follows:
5-15 hp motors, 3-5 hp motors, lighting loads 8 kW, miscellaneous loads 3.5 kW
The demand factor is 65% and the power factor is 80% wherein the load factor is 46%.
What is the maximum kW demand for the custorr^r?
A. 78 ( C 15 1
B. 64 D 69

24. The power customer has four circuits of 220 Volts, three-phase. The circuits have the
maximum demand as follows: Circuit-1 = 35 Amp, Circuit-2 = 46 Amp, Circuit-3 = 72
Amp, & Circuit-4 = 57 Amp. The diversity factor is 1.5, the load factor is 50%, and the
power factor is 80%. What is the approximate kW demand of the customer?
jA 15 kW C. 43 kW
( iT ) 21 kW D. 53 kW

25. A power plant has a maximum demand of 15 MW. The annual load factor and capacity
factor are 50 % and 40 % respectively. Determine the reserve capacity of the plant.
7530 kW C. 5730 kW
B.j3750kW D. 3075 kW

26. A power plant is said to have a use factor of 48.5% and a capacity factor of 42.4%. How
any hours did it operate during the year?
7,568 hrs C. 7,658 hrs
B, 1,102 hrs D. 8,000 hrs

27. An industrial plant has a peak load of 80 MW, daily load factor of 60%, daily capacity
factor of 80% and plant use factor of 75%. Calculate the daily energy consumption.
A. 1198 MWhr ('Cj 1080 MWhr
B. 1208 MWhr u. 1350 MWhr
28. Depreciation charges are high in the case of what power plant?
A. Windmill plant Diesel plant
B. Hydro-electric plant f CX) Thermal plant

29. An industrial plant has a maximum demand of 400 kVA at 78% p.f. lagging consumes
100,000 kWh per month. Calculate the monthly bill of this plant, Given the following
rates: Demand charge: P220.00/kVA maximum demand per month;
Energy charge: P2.22/kWh per month
Power factor adjustment rate:
0.6% surcharge for every percentage point lower than 85% p.f.
0.3% discount for every percentage point higher than 85% p.f.
A. P88,000.00 fo> P323,020.00
B. P222,000.00 u: P277.561.00

30. An industrial plant has a maximum demand of 750 kVA at 80% p.f. lagging. It is desired
to improve the p.f. to 95% using shunt capacitors which cost P800/kVA with annual
interest and depreciation of 10%. The demand charge is P500/kVA maximum demand.
What is the annual savings after putting capacitor?
A. P41,667.@§, C. P13,467.00
B. P12,755.@® < 0 p P38,983.00

Revprob Sept.2013
Power Plant: Supplementary Problems
I. With which of the following is enthalpy associated?
A. coal C. steam
B. electrostatic filter D. oil

2^/What is the meaning of “heat rate” of a generating unit ?


A. The amount of fuel being used per hour
B. The amount of fuel energy being used per hour by the boiler
C. The amount of fuel energy used to produced one kWhr.
D. The amount of heat energy per hour being absorbed at the condenser.

3. A diesel power plant is best suited as...


A. Base load C. Peak load
B Stand by D. General purpose

The energy of present nuclear plants comes from:


A. the fussion of radioactive materials
B. the friction of high speed electron against protons
C. The destruction of very tiny particles released when atoms split
D. The heat produced by the collision of high speed protons

5 / Which of the following element is a radioactive?


/ A. Cobalt C. Californium
B. Uranium D. Plutonium

6. In the list below, which one is not a type of present day nuclear power plant?
A. low pressurereactor C. boiling water
B. fusion-fissionreactor D. fast breeder reactor

y f ' Among sources of power which power in commercial quantity?


A. wind C. geothermal
B. solar energy D. biogas

JX ' Reflecting mirrors used for exploiting solar energy in a solar power plant is......
A. mantle C. ponds
B. heliostats D. diffuser

9. Of the various methods of utilizing the heat from the sun, one appears to be the most
efficient so far. Which one is this?
A self-contained collector C. direct application of lenses
B. mirror and tracking system D. flat plate collector

10. The following are the essential parts of a hydroelectric power station. EXCEPT:
A. spiral case C. surge tank
B. throttle valve D. impeller

II. A certain amount of fuel can produce 100 quads of energy. In how many days will the
fuel be totally consumed if it is used to satisfy a demand of 1014 BTU/day at a power
plant with an overall efficiency of 20%?
A. 400 C. 100
B. 2 0 0 ^ D. 300

12. To produce one kWh a power plant burns 0.9 lb of coal with a heating value of 13,000
BTU per lb. What is the heat rate of the plant?
A. 11,700 BTU/ kW-hr C. 9,500 BTU/kW-hr
B. 6,250 BTU/kW-hr D. 8,700 BTU/ kW-hr
13. In a power station, 4 x104 GWh of energy is to be produced in 1 yr, half from coal and
half from natural gas. The energy content of coal is 900 W-yr per ton, and that of natural
gas is 0.03 W-yr per ft3. How much coal will be required?
A. 5.07x106tons C. 2.53 x106 tons
B. 7.05x10®tons D. 5.23x10® tons

14. In a certain country the equivalent fuel reserve for power generation is 3 x10® MW-yrs.
The present peak power demand is 200 GW, and the expected power consumption
growth rate is 2.1%. How long will the fuel reserve last?
A. 13yrs C. 11 yrs
B. 14 yrs D. 12 yrs

15. A 100 MW power plant burns 0.4 kg coal with a heating value of 7.2 x10® cal/kg to
produce one kWh. The plant is a base load plant and runs 24 hrs at full load. How many
tons of coal are needed per day?
A. 830 C. 250
B. 960 D. 625

16. A power plant consumes 100,000 lbs of coal per hour. The heating value of the coal is
12,000 BTU per pound. The overall plant efficiency is 30%. What is the kW output of the
plant?
A. 175,000 kW C. 142,500 kW
B. 205,000 kW D. 105,500 kW

17. A diesel power station has a fuel consumption of 0.37 lb/kWh, the calorific value of fuel
being 30,000 BTU/lb. Determine the engine efficiency if the generator efficiency is 95%
neglect brake power on the engine.
A. 29% C. 36%
B. 32% D. 34%

18^stimate the average power output of a wind turbine having a blade diameter of 35 ft if
the wind velocity ranges from 10 to 30 mph?
A. 3.014 kW C. 24.11 kW
B. 81.364 kW D. 42.20 kW

19. One million cu meter of water is stored in a reservoir feeding water turbine. If the center
of mass of the water is 50 m above the turbine and losses are negligible, how much
energy (in MWh) will that volume of water produced? The density of water is 993 kg/m3.
A. 135.3 C. 136.3
B. 153.5 D. 163.6

20. A ten-year investigation of a river’s potential gave an average water flow of 25 cm per
second at the bottom and 90 cm per second at the surface. The average cross-section
at the same location is 80 m2. What is the average flow rate in cubic meter per hour ?
A. 5,760 Q ) 165,000
B.' 57,500 D. 46,500

21. A power plant gets water from a dam from a height of 122.45 meters at the rate of 1,000
cubic meters per minute. If the output of the plant is 15,000 kW, what is the efficiency?
A. 75% ' C. 80%
B. 70 % D. 65 %

22. A hydro-electric plant generates 100 MW at an available head of 200 m, and at an


overall efficiency of 75%, what quantity of water in cubic meter per second is required?
A. 122 C. 1200
B. 68 D. 26

' ojgfc **2^=3 rw er nousa


23. Hydroelectric power is generated at a dam that produces a head of 180 ft and a
reservoir containing 3 x1Q6 gal of water. How much energy can be generated from this
reservoir by a turbine-generator system whose overall efficiency is 20%?
A. 7761.5 MJ C. 6126.5 MJ
B. 1552.3 MJ D. 1225.3 MJ

24. A water pump discharges 100 gpm for a depth of 900 ft. friction loss is 4.4%.The motor
has an efficiency of 70%. If the cost of energy is P4.00 per kWh, what is the total cost of
electric energy to have one cubic meter of water?
A. P4.46 C. P1.12
B. P70.45 D. P12.25

25. A diversity factor of 4.2 gives a savings o f_____ percent in the generating equipment.
A. 80 C. 26
B. 40 D. 76

26. The current loads of four circuits are as follows :


Circuit No. 1 = 25 Amp Circuit No. 3 = 18 Amp
Circuit No. 2 = 38 Amp Circuit No. 4 = 45 Amp
If the diversity factor is 1.5. What is the minimum ampacity of the feeder conductor?
A. 126 Amp C. 84 Amp
B. 189 Amp D. 152 Amp

27. The power customer has four circuits of 220 Volts, three-phase. The circuits have the
maximum demand as follows:
Circuit No. 1 = 35 Amp; Circuit No. 3 = 72 Amp;
Circuit No. 2 = 46 Amp; Circuit No. 4 = 57Amp
The diversity factor is 1.5, the load factor is 50%, Determine the maximum kVA demand
on the customer?
A. 46 kVA C. 80 kVA
B. 26.5 kVA D. 53 kVA

28. Daytime capacity when only 300 MW pump storage plant and 600 MW Nuclear Plant is
operating.....
A. 500 MW C. 600 MW
B. 900 MW D. 300 MW

29. The metering of a power customer was read and gave the following data :
kW-hr = 200,000, Maximum demand = 380 kW, kVAR-hr = 180,000, billing days = 30
What is the load factor?
A. 68% C. 73%
B. 80% D. 62%

30. A generating plant rated 100 MW 90% power factor has a load factor of 65% for a
certain day. The minimum output was 40 MW. How many kWh generated for that day?
A. 2,160,000kWh C. 1,560,000 kWh
B. 960,000 kWh D. 2,400,000 kWh

31. The annual peak load on a 15 MW power plant is 10.5 MW. Two substations are
supplied by this plant. Annual energy dispatched through substation A is 27,500 MWh,
while 16,500 MWh are sent to substation B. Neglecting line losses. What is the annual
capacity factor of the plant?
A. 35.3% C. 33.5%
B. 47.8% D. 30.5%
32. A generating station has a maximum demand of 40 MW, annual load factor of 75%,
annual plant capacity factor of 65% and a plant use factor of 85%. What is the annual
energy produced?
A. 200.9x10® kWh C. 148.9x10® kWh
B. 195.6x10® kWh D. 812.3x10® kWh

33. A power plant has an annual factors as follows: Load 58.5 %, Capacity 40.9 % and
Use 45.2 %. The reserve carried over and above peak load is 8900 kW. The hours per
year not in service and installed capacity of this plant are respectively.
A. 792 hrs & 20,346kW C. 833 hrs & 39,147 kW
B. 933 hrs & 49,417 kW D. 822 hrs & 30,463 Kw

34. The annual load duration curve of a certain power station can be considered as a
straight line from 20 MW to 4 MW. To meet this load, 3 turbo-alternator units, two rated
at 10 MW each and one rated 5 MW are installed. Determine the plant use factor.
A. 80 % C. 48 %
B. 60% D. 38%

35. A power piant having three turbo-alternators rated 25 MVA, 0.9 p.f.,10 MVA, unity p.f.,
and 20 MVA, 0.85 p.f. has a maximum demand of 40 MW. What is the utilization factor
of this power plant?
A. 0.88 C. 0.81
- B. 0.90 D. 0.73

36. A generating station has the following data : Installed capacity = 300 MW, Annual load
factor = 60%, Annual Capacity factor = 50%, Annual interest & depreciation = 10 %
Annual cost of fuel, oil etc. = P90 Million, Capital Cost = P1,000 Million. What is the cost
per kWn generated?
A. P0.24 C. P0.20
B. P0.14 D. P0.10

37. A diesel plant has an overall efficiency of 30% the heating value of diesel is 18,000
BTU/lb. if diesel cost P6.00 per liter. What is the fuel cost of production in peso per
kWh. Take density of diesel to be 900 grams per liter.
A. PI.85/kWh C. P2.12/kWh
B. P1.91/kWh D. P2.45/kWh

38. A diesel generator set burns diesel with a heating value of 18,000 BTU per lb. The
diesel engine has an efficiency of 30% and the alternator has an efficiency of 95%.
Determine the fuel cost component of producing one KWh if diesel costs P2.80 per lbs.
A. P0.15 C. P2.15
B. P1.86 D. P3.28

39. Which of the following is the disadvantage of static capacitor for power factor
improvement?
A. easily damaged by high voltage C. short service life
B. cannot be repaired D. all of these

40. The type of switchgear assembly commonly selected for use in a turbine-generator
power plant is the ...
A. Vertical switchboard C. Duplex benchboard
B. Dual benchboard D. Enclosed switchboard

P r a c t i c e makes p e r f e c t I
Suppiprobs Sept.2013
This Free Quality Manual is
Found only at POWERLINE

E-MAIL: powerlinereview@yahoo.:com

POWERLINE REVIEW CENTER


M E Z Z A N IN E FLO OR, D O N A A M P A R O B U ILD IN G
C O R N E R ESP A N A & G. M. TO LE N TfN O S T R E E T S
S A M P A L O C , M A N ILA
T E L . N O S. 7 3 5 -7 3 -0 2 & 7 3 3 -2 1 -1 8

T E L , N O S . (0 3 )2 6 1 -2 2 4 4 & (0 3 2 )2 6 1 -8 4 5 2
POWERLINE REVIEW CENTER, INC.
POWER SYSTEM REVIEW LECTURE'

POWER SYSTEM:
Is the study concerned with generation, transmission, distribution and utilization of
electric power.

Classification of-Overhead Transmission Lines According to length


3. Short - 80 km (50 miles) cr less
2. Medium - 81 to 240 km (150 miles)
3. Long - above 240 km

1. Series resistance of the line } short } }


2. Series inductance of the line} }Medium }
3. Shunt capacitance of the line } }Long
4. Shunt conductance of the line }

' OF THE LINE


D C Resistance AC Resistance
£
R<!c ~ p ~ Rjc - k R,|c
A

Where,: |< l± -? A + J L - n W , t

f --0.0I05 d2f for copper VS


F —0.0063 d?.f for aluminum
d - diameter of conductor, inch
f - frequency, Hz

Skin Effect - the tendency o f current to move outward the surface of conductor.
Skin cffect depends upon the following factors :
a. Nature of material
h. Diameter of wire
c. Frequency
d. Shape of wire
Note: Skin effect is negligible when the supply frequency is low (less than 50 Hz) and conduc
diameter is less than 1.0 cm

SE R IE S INDUCTA N C E OF THE LINE


For 1<{>Solid Lines:
L ™2 x 10 In ~~ H I m per conductor
For 3<J> lines: ~ 9
I. - 2 x 10 1 in /A t. f 7 Z per phase —
Whcra. 1 IrC* \*} \
L r series imlucta.icc of the lm c \_ C c ',H )
D ~ disiancc between conductors • ""..... 1
r ' - self Geometric Mean Distance of solid conductor
' r ' = re ' 1' 4 -=0.7788 r 1 W* « M K */*1 U >
r - radius of conductor w X "ri J
Ds - self Gee metric Mean Distance of solid conductor or GeonTetrrc Meart^
Radius (GMR) of conductor.
Deq - equivalent spacing of conductors or Geometric Mean Distance (GMD
conductors.
Tjo ;-* <\LQoiuLurJ&rsJb rxDugzmGJit

% —

G M D or O cq - \J D i: D 1} • />„

Transposition - is the alternating of position of'each phase of the line at regular interval to balance the
inductance and 'he capacitance of the line.

EQLBsmdkdSsmd.ucio.c:
- it r e d u c e s the ic a e la n e e o f the lin e am i e ffe c t o f c o ;n n a . ^
A
d , d / V d

4 $
d O f
3 - stran d 4 - stran d
L I x 10 In /J- L M / m per phase
Ds"

Where: D sb • OMR o f the bundled conductor


IX1' ■ f n t (1 -■:> Pol ?;-'strar.d hnn/Hr
Dsu « ^£>y • d 2 rr> For 3 - strand bundle

Dr.1, - 1.(}c) xJ d , •f/1 Fur 4 - strand bundli*

HibVlll.C$fixt;itnr)c*f o f t!i<> liny


F o r 1 fj>, 2 w i r o l i n o :

— '— 1( ■
-0 -

2/ft:
F /111 C3n ~ Cp„ •' Cn ~ ---- 7- TVrn to neutral
In ■-- iU

C o n s id e r in g th u y ffe c t o f v a r th

-0 , -----------^

U - D -M
D/ /•

V 4 ;,1
7>a\17yA'\\V77XVv^r/7/^^X"7/rw?C
7JOL///d
f o r 3 t/j f i n e * ;

Cn ~ — F/ m to neutral
i„ D a '1

IVJ1 err: j.tt A,y.’> :c 1 0 IJ f / n i p e i n i t l i v i l y foi tre e s p a c e


3
P erform j?/!£C. o f Tran s m ission Lin e 5 11r.ing pcr phase basis :

SHORT TlM N £M ISSiaCL.lJMES :


• Rl Xl

——-'A
AAV
----------------------------------
"A r " 7
Vsrv m?) l^/oad^ww
Ir fs

Notation:
V o U d fs *-• /:< /n a t io n : V%f “ Sending end 1i?\c voltage
VK- Receiving end line voltage
Vs = v3 VSN VSN- Sending end line to neutral voltage
is“J
r
VUN Receiving end line to Neutral voiugc

Voltage Regulation : h ~ Sending end current


\y 1_ ! f / J
%VR = L"lL_Li*: x 100% IH - Receiving end current or loud current

%VR ' \W tA x :co% Pf r - Receiving end power factor


~ n ~
r f jk 'i e n c y o f T r a n s m is s io n : Pf.s " Sending end pmvev factor
Kc ceivini' t'/ui power
.< i 00% 0 s : Sending end. pfanj.de
Sc/i'fin;^ :'}\(l power
(J K- Receiving end pi inu>le
t e n d i n ',' e n d P o w e r ,.

?s ' v 3 Vo{s fJJs -- 3 VSN ls i)f^ Pu 1 I.i/ie I nss


iieceivittg end Power , P (>

P|{- n/3 V !( Ir P fK - 3 V kN I;i P fK


PtjY ver F a c t o r a t s e n d i n g a n d r e c e i v i n g , P f 3 &. ]>f K

Pfs = cos 9S «*cus j0,.. - 0Vyi | <t Pf* “• cos 0„ =-- cos jfl - 0„bv j

N o m ine! T-C irc n il R o n ro s e n tnt/on /7om in ^ l IT-C irc u it R qp ro s a lt a t i o n


Zl /2 zl
—V\AA— -VvW '-/”0W
---------------
■•vvvwjir^
Js
I-
^ " 1 j *
: Y /2
(flj
*
I- [1 v
Jc Jr
L i
^'t?I.it;g e k <;guIatio 11 F.fjicirncy
LONG T RANSMISSION MNKS :
---------------------------- ---- -—

P*
y pf? y
n Vrn

4-
z = series impedance per unit length
y =* shunt admittance per unit length
v sn - VKKcosh JzyC +I*ZC sinh JzyC
Z ,.~ z * & Y = y £
^sn ^ rn cosh ^ ( .sinn a^K

Is - Ik cosh yj/f V 4 sinh K

jZ
Zc ~ characteristic i m p e d a n c e - o h m s
S /

y - propagation constant'' J y:< ~ & jp


p ~ phasic .shift constant in radian per unit length
a - attenuation constant neper per unit length.
Wavelength; X •
2jt,
" if
Velocity oi p to p in io n , o - / • /
/ ’•- frequency in c,ps

£LLIRG.ZS
in transmission lines (he effect of lighiuing discharge (i.c .external causc)or Similar interim! causc
such as switching S irgcs, insulation breakdown, arcing ground and resonance when impressed
suddenly on a part of transmission line a quantity of electrical energy which moves along the. line a!
approximately the speed of light a> a "tm \ I'Jimy wuvc’* or "hue sm ’vv " which can.»c;; high rate ot
chance of voltage.

Surge Jtnpi'ilance (Z0) : Z 0 - .1 -—------ bn; fo; lossless line, it becomes : Z.Q •• /
V G + jo )(. VC

\y ! \y «'
S v r z c InweManre LnsniimL (S.I.L.) : S.f.L. - jj/, | - | ^ Wails
; iA I
Vc Vr
Re flucuon o f Truvelliuif Jt'Vn1'/ :
Orse 2 : Line open circuited at the other cud (Z w)
fg, j) — v
Reflected current: (e;0
Reflected voltage : A' open
R e s u 1ta n t e u rre n t : / ” 0 c ir c u it
R esultant voltage ; 2E

Ca.sc 2 : fane short circuited at the other end (/. • 0)


Reflected current : i ':z i
Reflected voltage : E
Rcsu!f:mt current 2/ oh or f circuited
Resultant volui^e : 0
Case 3 : Line terminated by resistance (Z„ - R)
Reflected current : j '- 0
Re flee led voltaic : A*'-1*0
RcyuJum: cuneni : i"-- t
>R
Resultant voltage : xT"~ E
o~ i
Q ENEBAU ZLO CIRCUIT CQ liS J A N T

V s.v- A V 1W+ 5 lR ; Is - C'V»m* 3 lR

At no load, IR«0, V*N-@ no load - VNL

Q Q R D m .J lQ .W m

Qcrvih'L- a luminous discharge due to ionization of air surrounding a conductor caused by high voltage
gradient exceeding a certain critical value. The phenomenon of violet j»low production of ozone
”as in an overhead lines.
/ 'actor T.f'fectinx { 'oran/t ;
1) Atmosphere
2) Conductor size
’>) Spacing between conductor
4) Line voltqgc

Disadvantages o f Corona :
■; Corona is accompanied by loss uf energy.
2) Gvonc is produced and may cause corrosion of conductor due to chemical reaction.
3 } Nomsmusoidal current cause irrteifcrence wuh communication lines.

Remedy:
1. Increase conductor size
2. Increase conductor spacim/,

Corana I'ower LossxC'.V


?4 * ' i■
CP ~ (1 25 ) (V (j) - Vc j~ x 10“' kW/kn: per phase
Where :
f - frequency i:i eps
r - radius of conductor
L) - distance between conductor
V4- applied voliajje/phase
8 ~ air density factor
s - H 2A.
"7+273
b r" barometric pressure m cm of mercury'
t r-‘ temperature in '5<j
V c ~ disruptive critical voltage/phase (k V )

Disruptive Critical Voltage^ Vi:


- the minimum phase to neutral voltage at which corona occurs.
Vc - mog0 S i' In — Vcks /phase
r
m0 - irregularity factor (1.0 for polished wire)
g0 - breakdown strength of air (30 kV rnax /cm or 21.1 kV rms 'em)
6
Underground Cables - essentially amoi.r.t of one or more conductors covered with suitable insulation and
surrounded by protecting core.

Single Core
co n d u cto r

in su lation
^ _______ J

jns_u 1st Hon K osistonce o f u n .d m y n m tu l slqP.Is


<K —- In
in — ohms
-L
2nt r.
Where :
p ~ specific resistance of insulation
r, ~ conductor radius

r: - cable radius ~ )] + t
t - thickness of insulation
I! - length of cables

QdPJ-titeoc e o fu n ^ o rg ro u o ^ s s h l s
2m: c
C" ...----- l'/n\
In ' ■

Where : •-,■■=8.85 x I !) 12 t'/m


r.t relative pcmuUivity of insulation

LI-JX 0M 3M L.TE D C A B L E S

Co - capacitance* between core


Cs - capacitance between each core and the sheet
B y d e l t a to w y e t r a n s j h r m a t i a n :
. Ca - Cs 3- Cc J;/m to neutral

SA G & S TR E SS A N A L Y S IS

i - J'o r L e vai Sappcrts :

(a)Parabolic Cable- ffor span of 300m or less) (b)Catenary Cable (for span of more than
i 300m)
DISTRtRUTiON SYSTEM
- The part of powe r system which distributes electric energy. It generally consist of
feeders, distributors. and service mains,

CLASSIFICATION OF DISTRIBUTION SYSTEM


1. Nature of Current
a. DC distribution system
b. AC distribution system
2. Types of Construction
a. overhead system
b. underground system
3. Scheme of Connection
a. radial system
b. ring system
c. interconnected system

FAULT CALCULATION

Faults on a Power System - it is the unintentional or intentional c onnecting together of two or more
conductors which ordinarily operates with a difference of potential between them.

PER-UNIT CALCULATIONS :
MVA x 1000 V0 KV2 . 'S k ’’2
Using : IL - —---- pz-------- Amp ; Z<? = — --------—--------— = -------ohms
b 7 3 kV H' MVA x_ 1000 A VA
T ik V
Actual Value
In general: Per Unit (p.u.j Values ^ — -------—
Base Vaiue
r r/2* MVA. X 1000
,7 base . f. = ____base______
Z base isrrj ' 1 base rrt l ,
MVA, <j3kV.
base base
o/o/,
/ y kV . i:
given given MVA,
____ oase i
For change o f base : Z pu new = • •--—
10 0 % k V, MVA . I
base given j

For C ircuit B reaker ( 600 V & above)

Interrupting Capacity oi Rating ~ Factor x Symmetrical Jsc

Factor : 1.0 —-------- 3 cycle duty


\ . \ ---------------- $ cycle duty
1. 3 ---- --------- 3 cycle duty
1.4 ......................I Vi cycle duty

Momentary Current Duty —Factor x Asymmetrical Isc 3*

Factor : 1.6 —-------------- for HV above 5 kV


1 5 ------------- — directly feed from generator

Interrupting Capacity or Rating - 100 % Asymmetrical isc :t+

Momentary Current Duty .== 125 % -v Asymmetrical Isc


1. Sa<* at any point, y 1. Max. sag, set x -
27’, 2 ’
@ max. sag, set x ’ | ; y nux -
o/n Ymax COSll ----
*> 2r0
2. Approx. length of cable in the span , 2. Approx. length of cable in the span ,
Lr = L + iil=2_. L c - -2- - ~ s i n h
3 I. " 2T,
3. Tension ul the point of supports, 3. I eru-ion at the point o f supports,
rp ... I... 2 ( ^
+( r |
Note: f i= I‘0 lens ion a I the luvve.st point.
H - S u n i w m o f D i f f e r e n t I f A w m 'w n : ( p a r a b o l i c c a b le )

7
[< -..... — ..... .......------------ s p a r ,, Jt .............. ..... - — .......... >•!
2. Difference in elevation :
h y,
2 ;>
h , iu! i_ **1
? .r„ 27
- ;,

cat2 .cax.f
~2T0 ’ " ~2T~ 27

h “ ~ ( x }-x,)(.u ix\)
** *
Solving fo r Xj & x : : Ih = * - r ((.Vj - X , )\
2j0

v . v -• — — bur x 2 - ^ —jc,; also ,v, - / - . v .


Ai (Oi
2hTo
( ^ .T, A', ) - ^
cot rod
2hT0 2X ^ W
^ ~ 2 a:, -
tvf ~ C "rttf ’
Xi ~- LAlk X2 - -
2 a* 2 toC
W h art:
x - any horizontal distance from lowest point of cable
y - any vertical distance from lowest point of cable
T„ - tension at the lowest point
CO ~ unit 'weight of cable
POWERLINe r e v i e w c e n t e r Inc.
The Ultimate E.E. Review Cento*
POWER SYSTEM REVIEW PROBLEMS

1. Which of the following standard transmission system voltages is classified as


Extra High Voltage?
A. 115 kV C. 138 kV
B.; 500 kV D, 230 kV

2. A 230 kV line has a horizontal configuration. The distance between adjacent


conductor is 20 ft. W hat is the geometric mean distance of the line?
A. 40 ft. C. 20 ft.
B. 28.3 ft. D. 25.2 ft

3. What is the GMR of 3-strands conductor of radius 1 mm of an individual strand ?


A. 2.1 mm C. 1.46 mm
B. 2.304 mm Uf 2.177 mm

4. A single phase transmission line is 35 km long, consist of two solid round


conductor, each of 0.9 fcm in diameter, the conductor spacing is 2.5 m. Calculate
the equivalent radius of a fictitious hallow, thin walled copper conductor having
the-same inductance as in the original design.
'A JO .35cm C. 1.6cm * v /'
B. 0.623 cm D. 1.25cm

5. A single phase transmission line is 35 km long, consist of two solid round


conductor, each of 0.9 cm in diameter, the conductor spacing is 2.5 m. Calculate
the inductance pe.r conductor v . v| . . ~
. A. 36 mH C. 46 mH * * S ,7 ~ - '
B. 63 mH D. 15mH

6. A 500 kV three phase transposed line is composed of one A CSR 1272 MCM,
45/7 Bittern conductor per phase with horizontal conductor configuration, 35 ft
spacing. The conductors have diameter of 1.345 in and G M R is 0.5328 in. Find
inductance per km. ~
A. 1.38 mH/km * C. 1.56m H /km V/
B. 2.54 mH/km D. 3.46 mH/km

7. A three-phase circuit, 60 Hz, 230 kV transposed transmission line is composed


of two 1272 MCM, 54/19 conductors per phase with horizontal configuration. The
bundle spacing is 50 cm while phase spacing between the centers of the bundle
is 10 m. If the G M R of the A CSR conductors is 0.0466 ft, find the inductive
reactance in ohm/km/phase of the transmission line?
A. 0.3775 C. 0.3628
B. 0.3398 D. 0.3964
8. A single circuit three-phase transmission line is composed of four (4) ACSR,
1,272,000 cmil conductor per phase with horizontal configuration. The bundle
spacing is 45 cm. and the adjacent phases are 14 m apart. Determine the
inductance in millihenry per km. The GM R of the AC SR conductors is 0.0466 ft.
0.88929 C. 0.59856
B. 0.77829 D. 0.72837

9. A three phase 60 Hz transposed transmission line has a flat horizontal spacing


configuration. The line reactance is 0.486 ohm per km. The conductor geometric
mean radius is 2.0 cm. Determine the spacing D in meter.
A.; 10 m C. 18 m
B. 15 m D. 20 m

10. The three phase untransposed 20 km transmission line has 4 m horizontal


spacing and a telephone line 2 m spacing is 5m below phase c and the line are
supported on the same tower. The power line carries a 60 Hz balanced current of
la=320cis 0°, lb=320cis 240°, Ic=320cis120° A. Find the magnitude of the voltage
induced in the telephone line. „.-~v
A. 70 V C /9 0 V
B. 60 V D. 80 V

11. A 13.2 kV, 3 miles transmission line is composed of 336,400 Cmil, 26/7 (Linnet)
acsr has GM R of 0.0244 ft and a resistance of 0.306 ohm per mile. The
conductors are spaced horizontally 3 ft apart. W hat is the series impedance?
A. 0.918 + j 1.385 C. 0.918 + j 1.538
B. 0 .9 1 8 + j 1 .8 3 5 ^ D. 0.918+j1.358

12. A double circuit 3-phase line are arranged at the vertices of a regular hexagon 20
ft on sides, Calculate the equivalent inductive reactance per mile of the parallel
line. Conductor has G M R of 0.0403 ft.
' A. 0.843 C. 0.368
' B. 0.278 D. 0.569

13. A transmission line has a triangular configuration of 4ft spacing. The conductor
336.4 M CM acsr. The outside diameter of 336.4 M C M acsr is 0.721 in. If the
length of the line is 30km, W hat is the shunt capacitive reactance per phase.
A. 7782 C. 3000
B 4833 D. 5000

14. A 500 kV three phase transposed line is composed of two AC SR 636 MCM, 24/7
Rook conductor per phase with horizontal conductor configuration, 35 ft spacing.
Bundle spacing is 18 inches. The conductors have diameter of 0.977 in and
GMR is 0.3924 in. Find capacitance per km.
A. 0.0107 //F /k m C. 0.01157 //F /k m
B. 0.0101 fi F /km D. 0.01264 /iF /k m
15. A 220 kV three phase transmission line is 40 km long. The resistance per phase
is 0.15 ohms per km and the inductance per phase is 1.3263 mH/km. The shunt
capacitance is negligible. Determine the voltage regulation when supplying 381
MVA at 0.8 power factor lagging at 220 kV.
A. 13.6% C. 12.4%
B. 15.8% D. 19.4%

16. A three phase short transmission line having a per phase impedance of 2+j4 has
an equal sending and receiving end voltage of 115 kV while supplying a load at
0.8 power factor leading. Calculate the power supplied by the line.
A. 780 M W C. 839.2 M W
B. 688.2 M W D. 670 M W

17. A 3-phase, 230 kV, 60 Hz 80 km medium transmission line has an impedance of


Z= 0.05+j 0.45 ohm per km and shunt admittance of j3.4x10~6 siemens per km.
What is the transmission line efficiency when the line delivers 200M VA, 0.8 pf
lagging at 220 kV?
A?98.75% C. 96.47%
97.92% D. 98.05%

18. The per phase parameters for a 60 Hz, 200 km long transmission line are R =
2.07 Q and L = 310 mH and C = 1.4474 j^F. The transmission line supplies a 100
MW, wye connected load at 215 kV (line to line) and 0.9 power factor lagging.
Using nominal n model, determine the sending end voltage line to line.
A. 229 kV C. 236 kV
B. 242 kV D. 233 kV

19. What is the sending current in the medium line when the when supplying a load
of 75M W at 132 kV, 0.8 power factor lagging. The line has series impedance of
15+j40 ohms and shunt admittance of 3.2x10.4 mhos.
A. 435 A C. 393 A
B. 550 A D. 410 A

20. A 3-phase, 115 kV, 60 Hz 120 km medium transmission line has an impedance
of Z= 0.05+j 0.45 ohm per km and shunt admittance of j3.4x10'6 siemens per km.
W hat is the transmission line equivalent constant D?
A 0.989+j0.001224 C. 0.979+j0.001212
B. 0.999+j0.001248 D. 0.969+j0.001199

21. A three phase, 60 Hz, 500 kV transmission line is 300 km long. The line
inductance is 0.97 mH per km and its capacitance is 0.0115 micro-farad per km
per phase. T h e receiving end rated load is 800M W , 0.8 power factor lagging at
500 kV. Determine the sending end voltage. Assume a lossless line.
A. 646.48 kV C. 625.42 kV
B. 656.81 kV D. 617,53 kV
22. The ABCD constant of a three phase 345 kV transmission line are:
A =D =0.98182+j0.0012447
B=4.035+j 58.947
C=j0.00061137
The line delivers 400 MVA at 0.8 power factor lagging at 345 kV. Determine the
sending end voltage.
A. 387.025 kV C. 367.045 kV
B. 376.087 kV D. 398.085 kV

23. If iliJ = it *1 = 33 kV (three phase) and X = 13 ohms per phase, what is the
maximum power transmitted per phase?
A. 29 M W C , 60 M W
B. 30 M W D. 28 M W

24. A three phase power of 875 M VA,0.8 power factor is to be transmitted to a


substation located 315 km from the source of power. For a preliminary desigr
assume the following parameters: V s=1.0 pu, Vr=0.9 pu, wavelength of 500C
km, characteristic impedance of 320 ohms and a power angle of 36.87 degrees
Based on practical line loadability equation, determine the line nominal voltage.
A. 420 kV C. 400 kV
B. 480 kV D. 380 kV

25. A 6000 volts line supplying three loads A, B & C. If the loads are located 4, 6 arid
10 km respectively away from the supply having a resistance of 0.05 ohm per km
& currents la=50A, lb=75A and lc=100A, W hat is the voltage delivered in load A
A) 5955 volts C. 5970 volts
B ' 5975 volts D. 5962 volts

26. Loads are tapped along a single phase primary distribution line as follows:
Distance from Sending end 1.0 km 2.5 km 4.0 krr
6.0 km
Load at unity power factor 20 kW 15 kW 15 kW 10 kW
Determine the equivalent length of the line, with a load at that end equal to the
total load that will give the same total moments of load. Using the equivalent lire
with an impedance of (0+/10) ohms per km, determine the sending end voltage Vs
if the line end voltage is 4,000 Volts. Vectorially, Vs = Vr + Ir Z and l=PA/ at u n r/
power factor.
A. 4,424 Volts^" C. 4,625 Volts*
B) 4,021 Volts D. 4,222 Volts

27. The capacitance per km of a 3 -w ire cable are 0.9 uF between the three
bunched conductor and the sheath, and 0.4 uF between conductor and the other
two connected to the sheath. Determine the line to ground capacitance in uF for
20 km length of cable.
A) 9 v C 6*
4A D. 7 X
28. The capacitance between any two conductors of a three conductor cable, the
third conductor is grounded is 0.6 uF/km. Calculate the line to ground
capacitance for a 25 km length of cable.
A 3 0 ju F C .6 0 //F y
B. 45 D. 1 5 / / F v

29. Calculate the maximum span between level supports for 500 M CM acsr
conductor weighs 4,122 lbs/mile if the allowable sag is 2 ft. The ultimate
strength of conductor is 24,400 lbs and safety factor is 4.0.
A 460 ft C. 300 ft *
'B>, 350 ft v D. 250 ft >

30. In a certain circuit analyses, the bases chosen are: 69 kV and 100 MVA, what is
ffjeJmpedance base?
A. 47.6 ohm s^" C. 23.8 ohms
"B. 7.5 ohm s y D. 5 ohms

31. The percent impedance of a line is 6% at 34.5 kV and 100 M VA bases. What is
the ohmic impedance?
A. 2.32 C. 3.0 Y
'B. 0.72 * D. 1.20--*

32. The impedance of a line is 5% on 115 kV and 100 M VA bases. W hat shall be at
120 kV and 10,000 kVA bases?
A. 1.2% C. 0.32%
/ B )0.46% D. 0.72%

33. At a 34.5 kV substation the available fault current is 10 p.u. W hat is the available
fault MVA if the base MVA is 50?
A. 100 r C. 50
B. 200 V ( D) 500 r

34. The available fault duty of a certain point of electrical system is 950 MVA at 230
kV, The Thevenin’s equivalent impedance is 2.63% , what is the available fault
current?
( a) 9 “l kA X C. 6 5 k A v
B. 72 kA / D. 80 kA

35. A simple power system composed of a generator, an ideal transformer(T 1) 1:10,


transmission line of impedance 30+j210 ohms, transformer (T2) 20:1 three phase
load of impedance 7.07+j7.07 ohm per phase at line voltage of 460 volts. Bases
are 10 MVA, 4 6 0 V. W hat is the generator voltage in per unit?
A. 1.086 x 1.043 \e
B. 1.019 D. 1.034 r
36. The per unit reactance of a synchronous generator are Xd=1.1, X d -0 .2 4 , Xd”=
0.15. The generator is operating without load at 5% above rated voltage then a
three phase fault occurs at its terminal, If a generator is rated 500 MVA and 20
K V determine subtransient fault current?
g ) 101 KA A C. 202 kA
B. 10.1 KA X D. 2.02 kA

37. Three 13.2 kv alternators, 30 MVA with 8%X, 90 MVA with 12%X and 30 MVA
with 9% X are connected to a common busbar. Find the MVA short circuit for a
three phase fault near the busbar. Bases are 90 M VA and 13.2 kV.
A. 145.9 * C. 281.7
>1,459 x D. 3 4 5 .9 *

38. In a short circuit study, the positive, negative and zero sequence impedance are
0.25 p.u. 0.25 per and 0.05 p.u.respectively. The base MVA is 100. Determine
the fault current for a three phase fault at tJ^e<115 kV level.
A. 3000 amperes x C. j 2000 amperes.*
B. 2500 am peres t l 3500 amperes ^

39. At a certain location in an electrical system, the available fault MVA is 500 MVA.
A 15 MVA, 34.5/6.24 kV, 5% impedance, wye-wye grounded transformer is
installed at that location. Determine the short circuit MVA at the secondary side
of the transformer. fe.
A. 195 M VA y 188 MVA X.
B. 175 M VA A ' D. 150 MVA

40. A small factory is tapped at 13.8 kV line where the available fault MVA is 150, the
line from the tapping point to the 3-333 kva transformer has an impedance of 0.5
ohm. The impedance of each transformer is 4% and the secondary voltage is
230 volts, what is the approximate three phase fault current at the secondary
side of the transformer bank?
A, 55,000 Amperes C. 65,000 Amperes
B. 50,000 Amperes D. 75,000 Amperes

41 .A utility supplies an industrial plant at 13.2 kV from a 20 MVA transformer whose


impedance is 8 %. The utility short circuit capacity at the primary of the
transformer is 500 MVA. It is desired to add 3-phase current limiting reactors the
secondary of the transformer to limit the initial fault capacity form the utility to 133
MVA. What is the reactance of the reactor required?
A. 0.625 ohm C' 0.265 ohm
B. 0.375 ohm D. 0.25 ohm

42. At a certain point on 34.5kV network the thevenin’s equivalent sequence


Xi=j0.15 per unit at 50 MVA base, Xo=j0.55 per unit at 50 MVA base. Find the
short circuit current for phase to phase fault ^t this point.
A 2800A C. 4800A
B 8000A D 1670A
43. At a certain point of the system network the positive, negative and zero
sequence impedances are 0.25, 0.25, 0.3 p.u. respectively. The base MVA is
100. The voltage level at that point is 34.5 kV. Determine the fault current for a
line to ground fault.
A. 6275 A C. 7516 A
B 8132 A D. 2091 A

44. A generator having a solidly grounded neutral and rated 50 MVA, 30 kV has
X 1=25% , X2=15% and Xo=5% . W hat reactance must be placed in the generator
neutral to limit the fault current for a bolted line to ground fault to that for a bolted
3-phase fault?
A. 2.4 ohms C. 1.5 ohms
B. 1.8 ohms ~ D. 1.2 ohms

45. The positive, negative and zero sequence reactances of a 20 MVA, 13.2 kV
synchronous generator are 0.3 pu, 0.2 pu and 0.1 pu, respectively. The
generator is solidly grounded and is not loaded. A double line to ground fault
occurs at the generator terminals, determine the fault current.
A. 4750 A C. 4500 A
B. 4250 A D. 3750 A

46. The connected electrical load of an office building is 300 kVA. The main circuit
breaker to be installed is 1,600 Amp, 2 poles, 250 Volts. The Meralco distribution
transformer is rated 20,000/230 Volts. It is to serve this load with a rated
connected load of 500 kVA, single-phase, 60 Hz, 3% impedance, 230 Volts.
W hat interrupting rating should be required for the main circuit breaker ?
A. 50,000 A Q. 150,000 A
B. 100,000 A D. 75,000 A

17. A simple power system consist of Generator of reactance j0.26 p.u., transformer
of reactance jO. 16 p.u. and a circuit breaker connected to a system of reactance
jO. 19 p.u. A three phase fault occurs between transformer and circuit breaker.
W hat is the fault current in per unit?
A. 2.38 p.u. C.)7.64 p.u.
B. 3.28 p.u. ' D. 2.88 p.u.

VWhat arrester nominal rating shall be used in a 34.5 kV grounded system?


A. 27 kV C. 22 kV
B'. 42 kV D. 35 kV

A power customer draws power at 220 volts from a transformer on a pole. A


current transformer with ratio 1200/5 and potential transformer with ratio 1000:1
are used to meter the electrical usage. W hat is the power indicated if the
wattmeter reads 800 watts.
A 19.2 M W C. 100 M W
B. 192 M W D. 80 M W
50. The C T ratio and PT ratio are 240 and 2000 respectively. The impedance of the
transmission line is 10 ohms, an impedance relay is installed to protect the line.
W hat is the impedance of the line as seen by the impedance relay?
A. 1.2 ohms C. 2.53 ohms
B. 20 ohms D. 101.6 ohms

51. A three phase wye-delta connected, 345/34.5 kV transformer has an emergency


rating of 60 MVA. Determine the CT ratio and C T connections required for the
protection of the transformer primary & secondary.
A. 200:5 A-con & 1000:5 A-con C. 100:5 Y-con & 1000:5 A-con
B. 200:5 A-con & 1000:5 Y-con D. 100:5A-con & 1000:5 Y-con

52. In a 71 network having Zi=4000+j0, Z2=5000+j0 and Z 3=2000+j0 ohms.


Determine the Y n parameter.
A 450 C. 200
B. -2 0 0 -“ S' D. 7 0 0 /“S

53. For Y-bus system shown, find Y 22


Line (bus to bus) Rpu Xpu
1-2 0.05 0.15
1-3 0.10 0.30
2-3 0.15 0.45
2-4 0.10 0.30
3-4 0.05 0.15
A. 3-j9 C. -2+j6
B. -1+j3 D. 3.666-J11

54. In a two port network, the hybrid parameters are given by hn=1/11, hi2=h2i=5/11
and h22= 3 /1 1. Find the yn parameter.
A . -2 rQ n
B. 5 "TT -5

55. Two bus system has: Zn= jO. 11565 pu , Zi2= Z21=j0.04580 and Z22=j0.13893 pu.
If an impedance Zb= j0.4 pu is connected between busses 1 and 2. Find the new
value of Z 22 ■
A. j0.10698 pu C. jO.05735 pu
B.'jO. 12352 pu D. j0.11565 pu

56. The per unit impedance of a short line is j0.06 pu. A per unit load on the line is
(1+j0.6)pu at a receiving end voltage of 1 Z o pu. Find the average reactive
power flow over the line.
-0.56pu C. 0.56 pu
B. -1.12 pu D. 1.12 pu
Power System Supplementary Problems

1. What is the GMR of seven strands conductor of radius 1 mm of an individual strand ?


A. 2.1 mm C. 1.46 mm
B. 2.304 mm fb '.) 2.177 mm

2. A single-phase transmission iine 90 km is made up of hard drawn copper 500 mils in


diameter. If the resistivity of the hard drawn copper at 20°C is 1 .77 x 1 0 "e ohm-meter.
Determine the loop resistance at 20°C.
A. 0.1394 ohm/km C. 0.546 ohm/km
B. 1.348 ohmkm D. 0.434 ohm/km

3. A transmission line cable consists of 26 strands of identical copper conductor, each 1.255
mm in diameter. The length of the cable is 10 km but because of the twist of the strands,
the actual length of each conductor is increased by 5%. If the resistivity of the copper is
1.72 x 1 0 -8 ohm-meter, what is the resistance of the cable in ohms?
A. 1.08 C. 1.13
B. 5.39 D. 5.66

4. A three phase transmission line is designed to deliver 190.5 MVA at 220 KV over a
distance of 63 km. The total transmission line loss is not to exceed 2.5% of the rated line
MVA. If the resistivity of the conductor material is 2.84x10'® ohm meter, determine the
required conductor diameter. , -% (
A. 1.56 cm C. 1.89 cm
B. 1.85 cm D. 2 . 0 cm

5. A three-phase transposed distribution line is designed with equilateral spacing of 12 ft. It


is decided to build the line with horizontal spacing D31 = 2D 12 = 2 D23 . What should be the /
spacing between adjacent conductors in order to obtain the same inductance as the
original design? ^
A. 9.5246 ft C. 9.5066 ft 2 > •
B. 9.6437 ft D. 9.6198 ft

. A single phase distribution line is installed with two No. 8 copper conductors spaced 15
cm. The line is 4 km long and the GMR with No. 8 is 1.27x10 3 m. What is the inductance
with the line?
A. 7.6 mH C. 4.2 mH
B. 27.5 mH D. 6.1 mH

7. A 69-kV three phase transposed line is composed of one ACSR 336,400 cmil, 36/7
Linnet conductor per phase with a horizontal configuration of D12=5 ft, D 23 = 5 ft and
D3 i = 1 0 ft. The conductors have a diameter of 0.721 inch, resistance of 0,306 ohm per
mile and gmr of 0.0244 ft. What is the reactance for 12 miles line.
A. 6 ohms C. 4 ohms
B. 2 ohms D. 8 ohms

3. A three-phase 60 Hz transmission line is to deliver 20,000 Kw at 6 6 , 0 0 0 volts at a


distance of 30 miles. The load pf is 85 percent lagging, the line conductor are 3/0 AWG
with resistance per mile of 0.326 ohms at 20°C and at 72 inch spacing and reactance per
mile of 0.742 ohms. Find the GMR of conductor.
A. 0.0178 ft C. 0.5146 ft
B. 0.0326 ft (g > 0.01668 ft
9. The three phase untransposed transmission line has 3.6 m horizontal spacing and a
telephone line A.2 m spacing is 4m below phase b and the line are supported on the
same tower. The power line carries a 60 Hz balanced current of 200 A. Find the voltage
per km induced in the telephone line.
A. 5.68cis 45° O: 3.88cis 60°
B. 2.44 cis 30° D. 6.32 cis 90°

10. A double circuit 3 -phase transmission line has a horizontal spacing of 6 . 0 ft and a
conductor vertical spacing of 3.0 ft. Calculate the GMD of the parallel lines.
A. 4.94 ft C. 4.76 ft
B. 9 ft D. 3.78 ft

11. A 115 kV double circuit 3-phase transmission line composed of 336.4 MCM ACSR with
GMR of 0.0244 ft has a horizontal spacing of 18 ft and a conductor vertical spacing of 9.0
ft.. Calculate the GMR of the parallel lines.
A. 0.744 ft C. 0.788 ft
B. 0.0244 ft D. 0.663 ft

12. A single phase secondary line has spacing of 12 cm and with a length of 250 meters.
The conductor is No. 8 copper with a GMR of 1.27x10'3 m and resistance of 2.36 ohm per
km. Determine the inductive reactance of the line.
A. 0.137 ohm C. 0.274 ohm
B. 0.17 ohm D. 0.086 ohm

13. A 5 km long, 3- phase 34.5 kV line has a horizontal configuration of 4 ft. spacing . The
conductor is 336.4 MCM ACSR with GMR of 0.0244 ft. What is the inductance of the line.
A. 5.33 mH C. 10.22 mH
B. 15.12 mH D. 1 2 . 0 2 mH

14. A 115 kV line has vertical configuration with 9 ft spacing. The diameter of the 336.4 MCM
ACSR is 0,721 inch. If the length of the line is 2 0 km what is the total capacitance?
A.-0.282 uf C. 2.2 uf
B. 0.187 uf D. 5.25 uf

15. A 100 km transmission line has a 1,200 ohms shunt reactance. What is the per km shunt
reactance?
A. 1200 ohms C. 120,000 ohms
B. 12 ohms D. 120 ohms

16. The capacitive reactance of a 100 km 34.5 kV line is 200,000 ohms per km, what is the
total capacitive reactance of the line?
A. 2 0 0 0 ohms C. 6750 ohms
B. 1.08 x 1 0 7 ohms D. 3.6 x 1 0 s ohms

17. The capacitive reactance of a 100 km, 23 kV line is 1 2 0 0 ohms, what is the capacitance
per km ?
A. 2 . 2 micro-farad C. 2.2x1 O'8 farad
B. 220 micro-farad D. 12 ohms

18. The inductive reactance of the line is 0.25 cis 35 deg. ohm per km. What is the total
reactance of the line at 2 0 km?
A. 0.0125 cis 35 deg. Q.J 5 cis 35 deg.
B. 0.125 cis 35 deg. D. 0.25 cis 35 deg.
19. A 5 miles three phase line has an equilateral spacing of 4 ft. The conductor has GMR of
0.01688 ft and resistance of 0.303 ohm per mile. What is the impedance?
A. 1.515+ j 3.46 C. 1.515 + j 0.002
B. 1.515 + j3.32 D. 0.505 + j 3.32

20. A 5 km long, three phase line has a horizontal configuration of 4 ft spacing. The
conductor is 336.4 MCM ACSR with GMR of 0.0244 ft and a resistance of 0.306 ohm per
mile. What is its impedance?
A. 2.22 cis 65° C. 6 . 6 6 cis 65°
B. 1.19 cis 46° D. 3.57 cis 46°

21 . A 115-kV three-phase transposed transmission line is composed of two ACSR 336,400


cmil, 36/7 Oricle conductors per phase with horizontal configuration: D 12 = 7 ft; D23 = 7 ft
and D31 = 14 ft. The line spacing as measured from the center of the bundled conductors
is 6 inches. The conductors have a diameter of 0.741 inch and GMR of 0.02265ft. Find
the capacitance per phase for 2 0 km of the line in micro-farads.
A. 0.2889 C. 0.2300
B. 0.2609 D. 0.2719

22. A single circuit, 745 kV, 60Hz 3-phase transposed transmission line is composed of four
acsr per phase, 1272,000 circular mils, 54/19, pheasant conductor has a horizontal
configuration. The distance between the centers of the bundle is 14m and the spacing
between conductor is 45 cm. The conductor has a diameter of 1.382 inches and gmr of
0.5592 inches. Find the capacitance per phase in micro-farad per km.
A. 0.0142 C. 0.0158
B. 0.0123 D. 0.0126

23. A 20 miles 3-phase transmission line is to deliver 2 0 , 0 0 0 kW at 69 kV at 85% power


factor, The line is composed of 300 MCM acsr conductor, resistance is 0.342 ohm per
mile and GMR of 0.023 ft, and spaced horizontally 8 ft apart. What is the sending end
voltage to neutral?
A. 72.4 kV C. 71.7 kV
B. 42.5 kV D. 73.7 kV

24. A-69 kV, 16km, 3-phase transmission line has an impedance of 0.125 +j0.4375 ohm per
km. Determine the transmission efficiency when the line delivers 70 MVA to a load at 64
kV with 80% pf lagging.
A. 98.75% C. 96.47%
B. 95.90% D. 98.2%

25. A three phase transmission line, 15 km long serves a substation rated 15 MVA at 34.5
kV, 60 Hz. The line impedance is 0 . 1 2 0 + j0.457 ohm per km, what should be the sending
end voltage so that the transformer can be fully loaded at 70.7% pf lagging at its rated
voltage?
A. 37, 2 0 0 Volts C. 35,340 Volts
B. 36,500 Volts D. 34,990 Volts

26. The voltage current and power factor at the input of a single phase transmission line are
2400 volts, 30 amp and 75 percent respectively. The entire load is connected at the line
output or line end, 7 miles from the source end. Calculate the voltage at the load end in
volts. The line resistance is 1.1 ohm per mile of wire and the reactance is 0 . 8 ohm per
mile of wire?
A 1927. 'v C. 1827
B. 1682 ^1535
I

: ~ - snge zrase 100 kVA, 14kV/2.4kV distribution transformer, is supplied through a line of
The impedance referred to secondary is 0.01 +J0.04 p.u. A load of 90 KW,
: 3 power factor lagging is connected at the low side. Find the supply voltage when the
. oltage at the load side is 2.3 kV.
A. 14.6 kV C. 14.2 kV
B. 15.2 kV D. 15.6 kV

28. A three phase transmission line has a resistance of 10 ohms and reactance of 80 ohms
per wire. The load current is 90 amperes and the power factor of the load is 80% lagging.
The sending (generator) end voltage in the line is 44,000 volts line to line. What is the
receiving end voltage?
A. 38.4 kv C. 34.3 kv
B. 42.3 kv D. 41.2 kv

29. A short 230 kv transmission line has an impedance of 5 cis 78 ohms. The sending end
power is 100 MW at 230 kv and 85% power factor. What is the voltage at the other end?
A. 225.4 kV C. 223.2 kV
B. 226.3 kV Q. 228.2 kV
30. A short 230 kV transmission line has an impedance of 5 cis 78° ohms. The sending end
power is 100 MW at 230 kV and 85% power factor. What is the percent regulation of the
line?
A. 3.6% C. 2.2%
B. 1.5% D. 0.77%

31. A 230 kV transmission line is sending 100 MW power at 230 kV and 90% power factor.
The impedance is 5 + j20 and its capacitive reactance is 2500 ohms. Determine the
receiving end voltage.
A. 221.72 kV C. 226.15 kV
B. 222.83 kV D. 224.28 kV

32. A 230 kV transmission line has an impedance of 50 cis 78 ohms and a capacity
reactance of 1200 ohms. >lt transmit the power of a base load plant. On a certain dry
season the sending end power is 100 MW at 235 kV and 95% power factor continuously
for a period of one month. If cost of generation is P1.30/kwhr, what is the cost of the line
losses for the one month period?
A. P565.000 C. P5.6 million
B. P12.2 million D. P2.2 million

33. The ABCD constants of a 60 Hz, 3-phase long transmission lines are as follows
A= D= 0.877/1.57° B = 191.62/79.1° C = 0.0012 190.4°
This supplies 100 MW load at 230 kV with 90% power factor. What is the
sending voltage ?
A. 248 kV C. 269 kV
B. 245 kV O’. 238 kV

34. What is the maximum power that can be transmitted over a three phase short
transmission line having a per phase impedance of 0.3 +j 0.4 ohm if the receiving end
voltage is 6351 volts per phase and the voltage regulation of the line is not to exceed 5%.
A. 108.9 MW C. 266.9 MW
B. 165.6 MW D. 312.3 MW
35. A three phase power of 3600 MW is to be transmitted via four identical 60 Hz
transmission lines for a distance of 300 km. From a preliminary line design, the phase
constant and the surge impedance are given by (5 =9.46x1 O'4 rad/km and Zc=343 ohms
respectively. Based on practical line loadability criteria, determine the suitable nominal
voltage level in kV for each line. Assume Vs=1 .Op.u., Vr=0.9 p.u. and power angle
S =36.87°.
A. 300 C. 500
B. 400 D. 800

36. The capacitance between any two conductor and the ground of a three phase, 3-
conductor cable is 2 uF. The cable operates at 11 kV line voltage and 50 Hz. What is the
charging current through cable capacitance?
A.57.98 A C. 6.78A
B. 5.26A D. 6.24A

37. Calculate the sag of an overhead distribution line having a span of 300 ft between level
supports. The conductor is 4/0 copper weighs 2,442 lbs/mile. The ultimate strength of
conductor is 14,050 lbs and safety factor is 5.0.
A. 0.46 ft C. 1.21 ft
B. 1.85 ft D. 1.08 ft

38. A span of 300 m between level supports is expected to have a maximum sag of 12 m
when the wind pressure is 12.2 gm/cm2 of projected area. The circular copper conductor
has an area of 1.29 cm2 and weighs 1.13 kg/m. If the conductor has a breaking strength
of 4,220 kg/cm2, What is the safety factor under these conditions ?
A. 2 C. 3
B. 4 D. 5

39. In transmission lines the cross arms are made of


A. copper C. wood
B. R.C.C. D. steel

40. Transmission line insulators are made of


A. glass C. porcelain
B. iron D. PVC

41. The use of strain type insulator is made where the conductor are
A. deadend C. any of the above
B. at intermediate anchor towers D. none of the above

42. In a certain circuit analyses, the bases chosen are: 34.5 kV and 100 MVA. What is the
impedance base?
A) 11.9 ohms C. 23.8 ohms
B. 7.5 ohms D. 5 ohms

43. A 50 MVA, 33 kV/11kV, three phase, wye-delta, connected transformer has a 3%


impedance. What is the % impedance at 100 MVA and 34.5 kV bases?
A. 1.639% C. 5.74%
B. 5.49% D. 6.56%

44. Find the ohmic value of the impedance 3.8% + j 15.2%. The base values are 100 MVA
and 115 kV respectively.
A. 5 + j 20 C. 3.8 + j15.2
B. 0 .5 + j 2 D 0.005 + j 0 2
45. The impedance of a transmission line is 30fi. What is the per unit impedance of 115 kV
and 100 MVA bases?
A. 0.18 C. 0.03
B. 0.05 V D.J 0.22

46. A 69kV/13.8 kV, 7.5 MVA transformer has 8% impedance. What is its impedance at base
100 MVA?
A. 0.6% C. 800%
B; 8% D 106.7%

47. In a certain line, the positive and zero sequence reactance of 3% & 15% respectively,
what is negative sequence reactance?
A. 3% C. 15%
B. 18% D. 12%

48. The three phase power and line to line ratings of the electric power system as shown in
figure below:

P1 Line w T-2 — Motor


60 MVA 20 kV X=9%
50 MVA 20/200kV X=10%
50 MVA 200/20kV X=10%
43.2MVA 18kV X=8%
200kV Z=120+j200
Choose base values as 100 MVA base and 20 kV base. Determine the internal emf of
the generator in kV if the motor is drawing 45 MVA, 0.8 pf lagging at a line to line voltage
of 18 kV.
A. 24.5 C. 26.5
B. 25.5 D. 27.5

49. A Generator rated 500 MVA, 20 kv has an impedance of 0.20 per unit. If the generator is
supplying 425 kW load at 20 kV, unity power factor and a fault occurs at the load. What
will be the per unit value of the fault current with respect to the bases, 500 MVA and 20
kV.
A. 4.0 C. 5.0
B. 2.0 D. 3.0

50. Behind a certain point in a system network the equivalent Thevenin’s impedance is 0.2
p.u. at 100 MVA base. A 115 kV/34.5 kV, 10 MVA transformer of 5% impedance is
tapped at this point. If a three phase fault should occur at the secondary of the
transformer, what is the maximum fault current?
A. 6692 amperes C^2390 amperes
B. 820 amperes 1590 amperes

51. A 15 MVA, 34.5kV/6.24 kV transformer is connected to infinite bus. The percent


impedance of the transformer is 2.5%. What is the current at 34.5 kV side for a three
phase short at the 6.24 kV side?
A. 10,000 amperes C. 5,000 amperes
B. 55,000 amperes D. 25,000 amperes
52. At a certain location in an electric system, the available fault MVA is 400 MVA. A 15
MVA, 34.5 kV/6.24kV, 2.5% impedance, wye-wye grounded transformer is installed at
that location. Determine the short circuit MVA at the secondary.
A. 195 MVA C. 90 MVA
B. 150 MVA D. 240 MVA

53. There was a 3-phase fault at a certain point in a 13.8 kV network where the Thevenin’s
equivalent impedance is 'A ohm per phase. What is the magnitude of the fault current?
A. 27600 amperes C. 15,900 amperes
B. 7970 amperes D. 13,800 amperes

54. There was a phase to phase fault at 13.8 kV system where the Thevenin’s equivalent
impedance is 2.63% what is the magnitude of the fault current. Base MVA is 10
A. 15,900 amperes C. 7970 amperes
B. 13,800 amperes D. 27,600 amperes

55. A 5 MVA, 13.8 kV / 480 Volts, 5% impedance transformer is tapped at 13.8 kV line where
the Thevenin’s equivalent impedance is % ohm. Determine the fault current at the
primary for a three-phase fault at the secondary.
A. 10,500 amperes C. 42000 amperes
B 3300 amperes D. 6050 amperes

56. A 10MVA, 13.8 kV/480V, 5% impedance, wye grounded-delta secondary . Transformer


serves an industrial customers. The phase a conductor on the secondary side
accidentally touches a grounded point. What is the fault current?
A. 50,000A C. 120.000A
B. 0 D. 80,000A

57. The transformer to serve a customer is rated 5MVA, 13.8 KV/480 V and its impedance is
5%. The cable connecting the breaker to the transformer has an impedance of 0.032 ohm
per phase. What is the fault current if a three phase fault occurs at the breaker?
A. 8000 amperes C. 6200 amperes
B. 5000 amperes D. 1200 amperes

58. The indoor 3-phase power center is to be served at 13.8 kV, the power center will
include a high side (primary) circuit breaker, a 1,500 kVA, 13,800/460 Volts transformer
with 4 % impedance, and a main circuit breaker. If the service point has a short circuit
capacity of 900 MVA, what is the momentary and interrupting duty at 3 cycles of the main
secondary circuit breaker?
A. 73 kA & 59 kA C. 63 kA & 49 kA
B. 83 kA & 69 kA D. 56 kA & 45 kA

59. At a certain point of a power system network the positive, negative and zero sequence
impedances are 0.25 p.u., 0.25 p.u. and 0.30 p.u. respectively. The base MVA is 100.
The voltage level at that point is 34.5 kV. Determine the short circuit current for a double
line to ground fault.
A. 5,578 Amp C. 12,551 Amp
B. 1,969 Amp D?, 5,906 Amp

60. A device which monitor and operates when certain level has been reached.
A. monitor C, relay
B. oscilloscope D, transformer
61. It is a protective relay which compares the sum of incoming currents against the sum of
the out going currents. It operates when there is unbalance.
A. Relay balance C. Differential relay
B. Current relay D. Over current relay

62. A circuit is disconnected by isolators when


A. line is energized C. circuit breaker is not open
B. there is no current in the line D. line is on full load

63. Surge arresters are needed in transmission line for the following purposes. Which is the
important?
A. Regulate the voltage
B. Prevent the lightning from striking the line
C. Protect the system from high voltage transient
D. Protect the line from short circuit current

64. Which one of the following/is not one of the classes of surge arresters?
A. Transmission class C. Station class
B. Distribution class D. Intermediate class

65. What arrester nominal rating shall be used in a 13.8 kV ungrounded system?
A. 11 kV C. 18 kV
B. 15 kV D. 21 kV

66. The distribution system is 34.5 kV ungrounded, which arrester shall be installed to protect
a distribution transformer.
A. 35 kV C. 27 kV
B. 38 kV D. 30 kV

67. For which of the followingcequipment current rating is not necessary


A. circuit breaker C. load break switch
B. isolator D. CB and load break switches

68. What is the relay that can detect overload in the line?
A. fuses C. overcurrent relay
B. differentialrelay D. magnetic switch

69. A three phase, delta-wye connected, 15 MVA, 33/11 KV step-up transformer is protected
by CT’s. Determine the GT ratios for differential protection such that the circulating
current (through the transformer delta) does not exceed 5 A.
A. 167.88 C. 137.56
B. 157.46 D. 262.43

70. A three phase delta-wye connected, 30 MVA, 33/11 kV transformer is protected by a


differential relay. Calculate the relay current setting for faults drawing up to 200% of the
rated current. The CT ratio on the primary is 500:5 and that on the secondary is 2000:5
A. 3.818A C. 3.138A
B. 4.318A D. 5.316A

71. The CT ratio and PT ratio are 240 and 2000 respectively. A reactance relay is installed to
protect the line. What is the reactance of the transmission line if the reactance as seen by
the reactance relay is 1.41 ohms.
A. 12.2 ohms C. 1.41 ohms ‘5 ’.,
B. 0.17 ohms " D. 11.75ohms
72. In transmission lines, the most effective protection against lightning strikes is one of the
following. Which one is this ?
A. Lightning rods C. Lightning arresters
B. Petersen coils D .. Overhead wires

73. Which of the following does not belong to the protection of a transmission line ?
A. distance relay C. reclose relay
B. ground relay D. reverse power relay

74. It is computerized data gathering, monitoring and switching equipment.


A. supervising control C. remote control
B. SCADA D. Control and monitoring system

75. Two bus system has: Z^= j0.11565 pu , Zi2= Z2-i=jQ.04580 and Z22=jO.13893 pu. If an
impedance Zb= j0.4 pu is connected between busses 1 and 2. Find the new value of Zn .
A. jO.10698 pu C. jO.05735 pu
B. jO.12352 pu D. jO.11565 pu

76. The following short circuit currents and voltages were determined experimentally for an
unknown two port network
When V2=0 V,=24 volts l^ m A l2=-0.6mA
When Vi=0 V2=40 volts 1,—lm A l2=12mA
Determine Y22 of the impedance matrix
A. 125/vS C. -25juS
B. 2 5 D. 300 ft S

77. In a two port network, the hybrid parameters are given by hn=1/11, h12=h21=5/11 and
h^=3/11. Find the y22 parameter.
AJ2 C. 11
B. 5 D .-5

78. For Y-bus system shown, find Yn ,.rr—,


Line (bus to bus) Rpu Xpu
1-2 0.05 0.15
1-3 0.10 0.30
2-3 0.15 0.45
2-4 0.10 0.30
3-4 0.05 0.15
A. 3-j9 C. -2+j6
B.-1+j3 D. 3.666-]11

JP JZ ^.C 'T Z C jE: J\JT ctic< zs I/


Power System Supplementary Problems

1. What is the GMR of seven strands conductor of radius 1 mm of an individual strand ?


A. 2.1 mm C. 1.46 mm
B. 2.304 mm fb'.) 2.177 mm
2. A single-phase transmission iine 90 km is made up of hard drawn copper 500 mils in
diameter. If the resistivity of the hard drawn copper at 20°C is 1 .77 x 1 0 "e ohm-meter.
Determine the loop resistance at 20°C.
A. 0.1394 ohm/km C. 0.546 ohm/km
B. 1.348 ohmkm D. 0.434 ohm/km

3. A transmission line cable consists of 26 strands of identical copper conductor, each 1.255
mm in diameter. The length of the cable is 10 km but because of the twist of the strands,
the actual length of each conductor is increased by 5%. If the resistivity of the copper is
1.72 x 1 0 -8 ohm-meter, what is the resistance of the cable in ohms?
A. 1.08 C. 1.13
B. 5.39 D. 5.66

4. A three phase transmission line is designed to deliver 190.5 MVA at 220 KV over a
distance of 63 km. The total transmission line loss is not to exceed 2.5% of the rated line
MVA. If the resistivity of the conductor material is 2.84x10'® ohm meter, determine the
required conductor diameter. , -% (
A. 1.56 cm C. 1.89 cm
B. 1.85 cm D. 2 . 0 cm

5. A three-phase transposed distribution line is designed with equilateral spacing of 12 ft. It


is decided to build the line with horizontal spacing D31 = 2D 12 = 2 D23 . What should be the /
spacing between adjacent conductors in order to obtain the same inductance as the
original design? ^
A. 9.5246 ft C. 9.5066 ft 2 > •
B. 9.6437 ft D. 9.6198 ft

. A single phase distribution line is installed with two No. 8 copper conductors spaced 15
cm. The line is 4 km long and the GMR with No. 8 is 1.27x10 3 m. What is the inductance
with the line?
A. 7.6 mH C. 4.2 mH
B. 27.5 mH D. 6.1 mH

7. A 69-kV three phase transposed line is composed of one ACSR 336,400 cmil, 36/7
Linnet conductor per phase with a horizontal configuration of D12=5 ft, D 23 = 5 ft and
D3 i = 1 0 ft. The conductors have a diameter of 0.721 inch, resistance of 0,306 ohm per
mile and gmr of 0.0244 ft. What is the reactance for 12 miles line.
A. 6 ohms C. 4 ohms
B. 2 ohms D. 8 ohms

3. A three-phase 60 Hz transmission line is to deliver 20,000 Kw at 6 6 , 0 0 0 volts at a


distance of 30 miles. The load pf is 85 percent lagging, the line conductor are 3/0 AWG
with resistance per mile of 0.326 ohms at 20°C and at 72 inch spacing and reactance per
mile of 0.742 ohms. Find the GMR of conductor.
A. 0.0178 ft C. 0.5146 ft
B. 0.0326 ft (g > 0.01668 ft

You might also like